You are on page 1of 172

End of chapter test

1 From patterns to generalizations:


sequences and series
Section A. A calculator is not allowed

1 The nth term of an arithmetic sequence is given by un= 4 + 3n

a Write down the first four terms.

b Find the common difference.

c Find the value of n if the nth term is 109.

d Find the sum of the first 21 terms.

2 In an arithmetic sequence, the 40th term is 144 and the sum of the first 40 terms is 2640. Find

a the first term

b the common difference.

2 2 2
3 Given the sequence ,   ,   ,  2 :
27 9 3

a Find the common ratio.

b Find the eighth term.

4 A school theatre has 24 rows of seats. There are 18 seats in the first row and each subsequent
row had two more seats than the previous row. What is the seating capacity of the theatre?

5 Find the first two terms of an arithmetic sequence where the sixth term is 21 and the sum of
the first 17 terms is zero.

6 a How many terms are there in the expansion of ( a + b ) ?


10

b Show the full expansions of (3x + 2y ) .


4

7 Evaluate
5 6
a ∑ (x
n =1
2
)
+ 2    b ∑2
i =2
i

Section B. A calculator is allowed

8 A geometric sequence has a 2nd term of 6 and a 5th term of 162.

a Find the common ratio.

b Find the 10th term.

c Find the sum of the first 8 terms.

9 A rose bush is 1.67m tall when planted, and each week its height increases by 4%. How tall
will it be after 10 weeks?

© Oxford University Press 2019 End of chapter test 1


End of chapter test

10 The Fibonacci sequence is named after Italian mathematician Leonardo of Pisa, who was known as
Fibonacci.

In this Fibonacci sequence 1, 1, 2, 3, 5, 8, 13, …,

a Find the 10th term of the Fibonacci sequence.

b Write a recursive formula for the Fibonacci sequence.

11 a Explain the condition for a geometric series to be convergent.

10 10
Given the series 10 − + − …,
3 9

b Find the common ratio

c Find the sum to infinity.


9
 2 
12 Find the constant term in the expansion of  x − 2 
 x 

13 Yosef drops a basketball from his bedroom window, which is 3m off the ground. After each bounce,
the basketball comes back to 75% of its previous height. If it keeps on bouncing forever, what
vertical distance, to the nearest metre, will it travel?

14 Tafari starts a job and deposits $500 from the first salary into a bank account producing 4%
interest every month. Each month after that, Tafari deposits an additional $100.

a Calculate the amount Tafari has in the account at the end of each month for the first three
months.

b Write a recursive formula for the amount of money in the account.

c Show that the formula for the amount of money in the account for year n is

(
500 (1.04 ) + 2500 (1.04 ) − 1
n n
)
d Use the formula to find the amount of money in Tafani’s account after 24 months.

© Oxford University Press 2019 2


End of chapter test

Answers

1 a 7, 10, 13, 16 n
d=
Sn
2
(u1 + un )
b d = 10 − 3 = 7
u1 = 7, n = 21, u21 = 4 + 3(21) = 67
c 4 + 3n =
109
21
3n = 105 S21 =
2
(7 + 67) = 777
n = 35

n b un = u1 + ( n − 1) d
2 a=
Sn
2
(u1 + un )
144 =−12 + ( 40 − 1) d
40
2640
=
2
(u1 + 144)
39d = 156

132
= (u1 + 144) d =4

u1 = −12

2 2 2
3 Given the sequence ,   ,   ,  2
27 9 3

2
9  2  7 2 7
a=r = 3 b u8 = 4
 3 = 3  3 =2 × 3 =162
2  27  3 
27

n
4 sn
=
2
(
2u1 + ( n − 1) d )
24
s24
=
2
(
2 (18 ) + (24 − = )
1) 2  984 seats

5 un = u1 + ( n − 1) d 21
= u1 + 5d
n 17
sn
=
2
( ) d 0 2 (2u1 + 16d )
2u1 + ( n − 1= )
This gives the simultaneous equations

u1 + 5d =
21

u1 + 8d =
0

Solve to find d = –7 and u1 = 56.

The first two terms are 56 and 49.

6 a 11

b Using the 4th row of Pascal’s triangle, 1 4 6 4 1,

1 (3x ) + 4 (3x ) (2y ) + 6 (3x ) (2y ) + 4 (3x ) (2y ) + 1 (2y )


4 3 2 2 1 3 4

=81x 4 + 216 x 3y + 216 x 2y 2 + 96 xy 3 + 16y 4

7 a 3 + 6 + 11 + 18 + 27  65
= b 4 +8 + 16 + 32 + 64 =124

© Oxford University Press 2019 3


End of chapter test

8 a 6r3 = 162

r3 = 27

r=3

b = ( u1 ) r n −1
un   

u10 = 2 × 39 = 39366

c S8
=
(
2 38 − 1
= 6560
)
3 −1

9 = ( u1 ) r n −1
un   

= (=
u10   1.67 ) (1.04)10−1 2.38m
10 a 55

b un +=
1    
un + un −1

11 a −1 < r < 1

10 10
Given the series 10 − + −…
3 9

10 1
b r =
− ÷ 10 =

3 3

10 10
c S∞
= = = 7.5
 1 4
1 − −  3
 3

3
9  −2 
12   x 6  2  = −672
3
  x 

13

u1 = 3, r = 0.75

3 3
S∞
= = = 12
1 − 0.75 0.25

To account for upward and downward movement, multiply by 2, but the original drop of 3 only
occurred once, so subtract 3 from the answer.

Vertical distance = (12 × 2) − 3= 21metres

© Oxford University Press 2019 4


End of chapter test

14 a Original investment = $500

Value after 1 month = 1.04 (500) + 100 = $620

Value after 2 months = 1.04 (1.04 (500) + 100) + 100 = $744.80

Value after 3 months = 1.04 (1.04 (1.04 (500) + 100) + 100) + 100 ≈ $874.59

b un = un–1 (1.04) + 100

c Value after n years =


500 (1.04 ) + 100 (1.04 ) + 100 (1.04 ) + 100 (1.04 ) + … + 100 (1.04 ) + 100
n n −1 n −2 n −3

n
(
= 500 (1.04 ) + 100 (1.04 )
n −1
+ (1.04 )
n −2
+ (1.04 )
n −3
+ … + (1.04 ) + 1   )
 (1.04 )n − 1 
=500 (1.04 ) + 100 
n

 1.04 − 1 
=500 (1.04 ) + 2500 (1.04 ) − 1
n n
( )
 

d 500 (1.04 )
24
(
+ 2500 (1.04 )
24
−1 =)
$5190

© Oxford University Press 2019 5


End of chapter test

2 Representing relationships:
introducing functions
Section A. A calculator is not allowed

1 Which of the following sets of ordered pairs are functions?

a {(1, 1), (2, 4), (3, 3), (3, 6), (4, 8)}

b {(−1, 3), (−2, 5), (−3, 7), (−4, 9), (−5, 11)}

c {(−2, −2), (−1, −1), (0, 0), (1, 1), (2, 2)}

2 Which of the following relations are functions?

a b c

3 a Does this mapping diagram represent a function?

b State the domain and range.

4 f (x) =
3x + 5 and g ( x ) =
2 − 2x

Find:

a f ( 4) b f −1 (2 ) c f  g (x) d g  f ( −4 )

Solve:

e f ( x ) = −1

© Oxford University Press 2019 End of chapter test 1


End of chapter test

5 Write down the domain and range of this function:

6 Which of these are one-to-one functions?

a b

x −1
7 If f ( x ) =  , find f−1(x).
x −2

© Oxford University Press 2019 2


End of chapter test

Section B. A calculator is allowed

8 Use your GDC to sketch these graphs. Write down the domain and range of each:

a f ( x ) =  x 2 + 2 x − 3 x)
b g (= x +2

9 If l ( x )   
= x 2 and m ( x )   = 2 x   –  3   :

m (x)
a Find (l   ◦  ) 

m (x)
b Sketch the function (l   ◦  ) 

c State its domain and range.

10 If h ( = ) x + 1 and p ( x )   
x  3 = x3 :

a Find x when h ( x ) = 13

b Find x when p ( x ) = −27

c Use a sketch to solve h ( x ) = p ( x ) .

11 Given f ( x ) =
3x − 5 and g ( x ) =
x − 2,

a Find f −1 ( x )

b Show that g −1  f( ) ( x=) 3x − 3

f (x)
Let h ( x )
= = and i ( x ) x
  
g (x)

c Solve h ( x ) = i ( x ) by sketching the functions.

12 Given f ( x ) = x 3  – 3 :

a Find the inverse function f −1

b If the point ( a, b ) lies on f ( x ) , what is the coordinate of its image on f −1 ( x ) ?

c Sketch them both on the same axes.

d Solve f ( x )   
= f −1 ( x ) .

© Oxford University Press 2019 3


End of chapter test

Answers
1 b and c
2 a and c
3 a Yes
b The domain is 1, 2, 3, 4, 5. The range is 19, 23, 29, 31.

4 a f ( 4=
) 3 ( 4) + =
5 17  c f  g ( x ) = 3 (2 − 2 x ) + 5 = 11 − 6 x

b =
x 3y + 5 d g  f ( −4 )

x −5 =
3y f ( −4 ) =3 ( −4 ) + 5 =−7

x −5 g ( −7 ) = 2 − 2 ( −7 ) = 16
y =
3
e 3x + 5 =−1
x −5
f −1
(x) = 3 3x = −6

x = −2
2−5
f −1 (2 ) = = −1
3
5 The domain is −6 ≤ x ≤ 4 and the range is −3 ≤ y ≤ 4

6 b

y −1
7 x =
y −2

x ( y − 2) = y − 1

xy − 2 x =y − 1

xy − y = 2 x − 1

y ( x − 1) = 2 x − 1

2x − 1
f −1 ( x ) =
x −1

8 a f ( x ) =  x 2 + 2 x − 3 Domain x ∈  Range y ≥ −4

b g (=
x) x +2

Domain x ≥ −2 Range y ≥ 0

© Oxford University Press 2019 4


End of chapter test

( x ) = ( x + 3) = 4x 2 + 12x + 9
2
9 a (l   ◦ m) 2

c Domain x ∈  Range y ≥ 0

10 a 3x + 1 
= 1
  3

3x = 12

 4
x =

b x 3 =  27

x = −3

x =
−1.53,  − 0.347, 1
 .88

11 Given f ( x ) =
3x − 5 and g ( x ) =
x − 2,

a =
x 3y − 5

x +5 =
3y

x +5
f −1 ( x ) =
3

b (g −1
f ) ( x ) = (3x − 5) + 2 = 3x − 3

© Oxford University Press 2019 5


End of chapter test

x = 1.38,  3.61

12 Given f ( x ) = x 3  – 3 ,

a =
x y3 − 3

x +3 =y3

f −1 ( =
x) 3
x +3

b ( b, a)
c

d x = 1.67

© Oxford University Press 2019 6


Mixed review

2 Representing relationships:
introducing functions
ex − 1
1 We consider the function f (x ) = .
ex + 1

a Write down the domain of the above function.

b Work out the inverse of the above function.

c Work out the domain of the inverse function.

d Hence, write down the range of f .

e Prove that f (f −1(x ))


= x= f −1(f (x )) , for every x in the domain of f .

2 We consider the functions f (x ) = 2kx 2 + 3(k + 1)x − 7, k ∈  and g(x=


) x + 2 . Prove that there is
no value for the parameter k ∈  such that the function f (g(x )) is a perfect square.

3 a if f (x ) =x+ x 2 + 1 , x ∈  , prove that f (x ) × f (− x ) =


1 , ∀x ∈  .

ax + 3
b We consider function =
f (x) , ∀x ≠ 2 . Find the value of a ∈  such that
2−x
f (f (x )) = x, ∀x ≠ 2 .

e x − e− x
4 Given the functions f (x ) = ln(x + x 2 + 1) and g(x ) = , prove that f (g(x )) = g(f (x )) .
2

5 We consider the function f (x ) =


x + ln(x ), x > 0 .

a Prove that f is a 1 -1 function.

b Work out the value of f −1(e + 1) .

2λ 2 + 1
c Solve the equation: ln( )= 4 − λ 2
λ2 + 5

Exam-style questions

1
6 It is given that f (=
x) − 1 , x > 0, x ∈  .
x

a Determine the exact value of ff ( 0.01) . (4)


b Find an expression for f −1
(x) . (3)
c State the domain and range of f −1 ( x ) . (2)

© Oxford University Press 2019 1


Mixed review

4
1 
7 a Find the expansion of  + x  in ascending powers of x . (3)
 4 

6
 1
b Find the expansion of  2x −  in ascending powers of x . (3)
 x
c Hence, or otherwise, find the term independent of x in the expansion of
6 4
 1 1 
 2x −   + x  . (3)
 x 4 

8 A convergent geometric series has sum to infinity of 65.

Given that the first term in the series is 52, find the number of terms required for the sum of
the series to exceed 64.99. (7)

4x − 1
9 a Find the value of a , given that f ( x ) = is a self-inverse function. (6)
2x + a

b Determine the largest possible domain for f ( x ) . (2)

1
c Solve the equation fff ( x ) = . (3)
4

x 2 − 36 Ax 2 + Bx + C 2 x 2 − 11x + 12 2 x 2 + x − 5
10 It is given that 2
× × ≡
2 x − 11x − 6 x−4 x 2 + 9 x + 18 2x + 1

Prove that=
A 1,
= B 5 and=
C 6 (7)

© Oxford University Press 2019 2


Mixed review

Answers

1 a Domain = 
ex − 1
b y =
ex + 1
y(e x + 1) = e x − 1
ye x + y = e x − 1
e x (y − 1) =−y − 1
1+ y
ex =
1−y
1+ x
f −1(x ) = ln( )
1− x
1+ x
c The domain of the inverse is Df −1 =
{x ∈  / > 0} .
1− x
We obtain Df −1 = {x ∈  / (1 + x )(1 − x ) > 0}= {x ∈  / −1 < x < 1} .

d We know that if f : Df → Rf , where Df , Rf are domain and range respectively, then


f −1 : Rf → Df .

Hence, Rf = {y ∈  / −1 < y < 1} .

1+ x 1+ x 1+ x −1+ x
ln( ) −1
e 1− x
−1 1− x = 1 − x = 2=x
e −1
f (f (x=
)) = x
ln(
1+ x
) 1+ x 1+ x +1− x 2
e 1− x
+1 +1
1− x 1− x
ex − 1 ex + 1 + ex − 1
1+ x
e= + 1 ) ln( e x + 1= 2e x
f −1(=
f (x )) ln( ) ln(= ) ln(
= ex ) x
ex − 1 ex +1− e +1x
2
1− x
e +1 ex + 1
2 = 2k(x + 2)2 + 3(k + 1)(x + 2) −=
f (g(x )) 7

= 2k(x 2 + 4 x + 4) + 3(kx + 2k + x + 2) −=
7

2kx 2 + x(3 + 11k ) + 14k − 1 .

Expression A is a perfect square if there is another expression B such that B2 = A .


In this case f (g(x )) is a perfect square if the discriminant of it, in terms of the parameter k , is
zero.
We are to prove this by contradiction:
We assume that there is at least one value for the parameter k such that the f (g(x )) is a
perfect square.
We calculate the discriminant of the f (g(x )) in terms of k : (11k + 3)2 − 4 × 2k × (14k − 1) =

= 121k 2 − 38k + 9 .
We equal the above quantity to zero: 121k 2 − 38k + 9 =
0.
Solving the above the equation: D =382 − 4 × 121 × 9 =−2912 < 0 - contradiction.
Therefore, the is no value for real k .

3 a Initially, we find f (− x ) =− x + (− x )2 + 1 =− x + x2 + 1 .

We calculate: f (x ) × f (− x ) = (x + x 2 + 1) × (− x + x 2 + 1) = ( x 2 + 1)2 − x 2 = x 2 + 1 − x 2 = 1 .

© Oxford University Press 2019 3


Mixed review

b We work out the f (f (x )) .

ax + 3 a2 x + 3a + 3(2 − x )
a×( )+3
2−x 2−x a2 x − 3x + 3a + 6
f (f (x )) =
= = .
ax + 3 2(2 − x ) − (ax + 3) x(− a − 2) + 1
2−( )
2−x 2−x
However, f (f (x )) = x .

a2 x − 3x + 3a + 6
Therefore, = x.
x(− a − 2) + 1

We obtain: a2 x − 3x + 3a + 6= x 2 (− a − 2) + x

x 2 (a + 2) + x(a2 − 4) + 3a + 6 =.
0
Hence, a + 2 =
0

a2 − 4 =
0
,
3a + 6 =0
from where we obtain that a = −2 .

e x − e− x (e x − e − x )2 + 4 e x − e− x e2 x + e −2 x + 2 e x − e− x e x + e− x
4 f (g(x ))= ln( + )= ln( + )= ln( + )=
2 4 2 4 2 2
e x − e− x + e x + e− x 2e x
= ln( = ) ln(= ) ln(
= ex ) x
2 2
1 1
2 2 2
ln( ) x+ x2 + 1 − ( )
eln( x + x +1) − e − ln( x + x +1) eln( x + x +1) − e x + x +1
2

=g(f (x )) = = = x + x2 + 1
2 2 2
(x + x 2 + 1)2 − 1 x 2 + 2x x 2 + 1 + x 2 + 1 − 1 x2 + x x2 + 1 x(x + x 2 + 1)
= = = = = x
2(x + x 2 + 1) 2(x + x 2 + 1) x + x2 + 1 x + x2 + 1
Therefore, f (g(x )) = g(f (x )) .

1
5 a We find the derivative of f : f ′(x ) = 1 + > 0, ∀x ∈  .
x
Hence, f is an increasing function. Therefore, f is 1 – 1.
b We know that f (a) = b ⇒ a = f −1(b) .
We observe that f (e) = e +1.
e + ln(e) =

Therefore, f −1(e + 1) =
e.
c By using the Laws of Logarithms, we obtain:
ln(2λ 2 + 1) − ln(λ 2 + 5) = 4 − λ 2 .

ln(2λ 2 + 1) + (2λ 2 + 1) − ln(λ 2 + 5) − (λ 2 + 5) = 4 − λ 2 + (2λ 2 + 1) − (λ 2 + 5)

f (2λ 2 + 1) − f (λ 2 + 5) =
0

f (2λ 2 + 1)= f (λ 2 + 5)
Since f is 1 – 1 function, we can write:
2λ 2 + 1 = λ 2 + 5

λ2 = 4
λ = ±2 .
1
6 a f ( 0.01
= ) =−1 9 M1A1
0.01
1 2
ff ( 0.1) =f ( 9 ) = − 1 =− M1A1
9 3

© Oxford University Press 2019 4


Mixed review

1
b Let=
y −1
x
Attempt to make x the subject M1
1
y +1 =
x
1
x =
y +1

1
x = A1
( y + 1)
2

1
f −1 ( x ) = A1
( x + 1)
2

c Domain of f −1 ( x ) is x > −1, ( x ∈ ) A1

Range of f −1 ( x ) is f −1 ( x ) > 0, (f ( x ) ∈ )
−1
A1
4 r
1  4
1
7 a ∑
 + x =
4  r =0
4
Cr x 4 − r  
4
M1

3x 2 x 1
= x 4 + x3 + + + A1A1
8 16 256
6 r
 1 6
 1
∑ Cr (2 x )
6−r
b  2=
x−  6
−  M1
 x r =0  x
60 12 1
= 64 x 6 − 129 x 4 + 240 x 2 − 160 +
− + A1A1
x2 x 4 x6
 1   3
c Require  −160 ×  +  60 ×  + ( −12 × 1) M1A1
 256   8

79
= =( 9.875) A1
8
52
8 65 = M1A1
1−r
52
1−r =
65
13
r
= = 0.2 A1
65
Let n be the required number of terms for the sum to be equal to 64.99.

64.99 =
(
52 1 − 0.2n ) M1A1
1 − 0.2
51.992
= 1 − 0.2n
52
51.992
0.2n= 1 −
52
n = 5.46 A1
So require 6 terms A1

© Oxford University Press 2019 5


Mixed review

4x − 1
9 a Let y =
2x + a
Attempt to make x the subject M1
2 xy + ay = 4 x − 1 A1
1 + ay = 4 x − 2 xy

1 + ay = x ( 4 − 2y ) A1

1 + ay
x = A1
4 − 2y

1 + ax  − ax − 1 
=f −1 ( x ) =  or f ( x )
−1
 A1
4 − 2x  2x − 4 

4x − 1
Comparison with f ( x ) = yields a = −4 A1
2x + a
4x − 1
So f ( x ) =
2x − 4
b Domain is x ∈ , x ≠ 2 A2

c f is self-inverse, so fff ( x ) = f ( x ) R1

1 4x − 1 1
f (x) = ⇒ = M1
4 2x − 4 4
16 x − 4 = 2 x − 4
14 x = 0
x =0 A1
10 Attempt to factorise relevant expressions M1
( x + 6 ) ( x − 6 ) × Ax 2
+ Bx + C (2 x − 3) ( x − 4 ) (2 x − 3) ( x + 2 )
× ≡ A3
(2x + 1) ( x − 6 ) x−4 ( x + 6 ) ( x + 3) 2x + 1

Attempt to cancel terms: M1


2
Ax + Bx + C
≡ ( x + 2) A1
( x + 3)
Ax 2 + Bx + C ≡ ( x + 2 ) ( x + 3)

Ax 2 + Bx + C ≡ x 2 + 5x + 6
Equating coefficients gives= A 1,
= B 5 and=
C 6 A1

© Oxford University Press 2019 6


Mixed review

3 Modelling relationships: linear


and quadratic functions
1 The terms in an arithmetic sequence are given by un  3   n  1 2 .

a Write down

i the first term in the sequence;

ii the common difference.

b Find the first five terms in the sequence.

c Graph the points  n, un  , for the five terms found in part b.


d Find the gradient-intercept form of the equation of the line that contains the points graphed
in part c.

e Show that un  3   n  1 2 is equivalent to the equation found in part d.

2 A linear function, f, passes through the points 2, 4 and  6,3 .

a Find the gradient-intercept form of the equation of the function.

b Find f 1 .

3 Part of the graph of the quadratic function f is shown in the diagram. The domain of f is all real
numbers.

a Write down the range of f.

b Determine whether the inverse of f exists. Explain your reasoning.

c Write down

i the coordinates of the y-intercept

ii the coordinates of the x-intercepts

iii the vertex of the graph of f

iv the equation of the axis of symmetry of the graph of f.

d Find an equation for the function f.

© Oxford University Press 2019 1


Mixed review

4 The graph of y  f  x  , where 3  x  2 , is shown in the diagram.

a Graph f and f 1 on the same axes.

b Given that g(x)  2f (x  3)  5 , graph f and g on the same axes.

5 Let f (x)  x2  2x  3 and g(x)  x  2 .

a Let h(x)   f g   x  . Show that h(x)  x2  6x  5 .

b Find the equation of the axis of symmetry for the graph of h.

c Find the coordinates of the vertex of the graph of h.

d Find an equation for h in the form h(x)   x  p  x  q  , where p and q are integers.

e Sketch a graph of y  h  x  , where 1  x  5 .

Exam-style questions

6 The first term of an arithmetic sequence is 0 and the common difference is 8 .


a Find the value of the 32nd term in the sequence. (2)

The first term of a geometric sequence is 6. The 11th term of this sequence is equal to the 25th
term of the arithmetic sequence above.
b Find the value of r , the common ratio of the geometric sequence. (4)
c Determine whether the sum to infinity of the geometric sequence exists. (2)

7 In increasing powers of x, the fourth term in the binomial expansion of 3x  a is 145152x3 .
9

Determine the value of a . (5)

8 Find the range of values of x that satisfy both 2x2  3x  9  0 and 2x2  9x  35  0 . (9)

9 a Determine the range of values of p for which the equation px2  px  1  p has two
distinct real solutions. (6)

Suppose that f  x   px2  px  1  p , x  .

b Show that the line of symmetry of the graph of y  f  x  is independent of p . (2)

c In the case when p  4 , express f  x  in the form f  x   a  x  h  k where a, h and


2

k are integers. Hence solve the equation f  x   0 . (6)

© Oxford University Press 2019 2


Mixed review

10 a Describe a series of transformations that maps the graph of y  x 2 onto the graph of

y  3   x  5 .
2
(6)

Consider the function f  x   3   x  5


2

b Find the largest possible domain of f  x  such that f 1 exists, and justify your answer.

On the same axes, sketch the graphs of y  f  x  and y  f 1  x  . (5)

© Oxford University Press 2019 3


Mixed review

Answers
1 a i u1 = 3 ii 2
b 3, 5, 7, 9, 11
c

d y = 2x + 1
e let n = x, y = un
y = 3 + (x – 1)(2)
y = 3 + 2x – 2 = 2x + 1
x 9
2 a y   b f 1(x)  4x  18
4 2
3 a y ≥ −4
b The inverse does not exist, because f is not one-to-one.
c i (0, 5) ii (−5, 0), (−1, 0) iii (−3, −4) iv x = −3
d f (x) = x2 + 6x + 5 or f (x) = (x + 5)(x + 1) or f (x) = (x + 3)2 − 4
4 a

© Oxford University Press 2019 4


Mixed review

5 a (f∙g)(x) = (x – 2)2 – 2(x – 2) – 3


= x2 – 4x + 4 – 2x + 4 – 3
= x2 – 6x + 5
b x=3
c (3, −4)
d h(x) = (x – 1)(x – 5)
e

6 a u32  u1  31d M1

 0  31  8
 248 A1
b 6r 10  24  8 M1A1
10
r  32 A1

r  2 A1

c r  2 1 R1
therefore S does not exist. A1

9 3
  3  a  145152
6
7 M1A1
3
 
84  27  a6  145152 A1
6
a  64 A1
a2 A1
2
8 2x  3x  9  0

2x  3  x  3  0 M1

3
Critical values x  3, x  A1A1
2
3
Solution 3  x  A1
2
2x2  9x  35  0

2x  5  x  7  0 M1

5
Critical values x   , x 7 A1A1
2
5
Solution   x 7 A1
2

© Oxford University Press 2019 5


Mixed review

5 3
So, solution required is  x A1
2 2
9 a px2  px  1  p  0

Require b2  4ac  0 M1
p2  4p 1  p  0 A1

5p2  4p  0

p 5p  4  0 A1

4
Critical values p  0, p  A1
5
4
Solution is p  0 and p  A1A1
5
b
b Line of symmetry is x   M1
2a
p 1
x    , so independent of p . R1
2p 2

c f  x   4x2  4x  5

 5
 4  x 2  x   M1
 4

 1
2
1 5
 4   x      A1
 2 4 4 

 1
2
3
 4   x    
 2 2 

2
 1
 6  4x   A1
 2
2
 1
f x  0  6  4  x    0 M1
 2
2
 1 3
x   
 2 2

1 3
x 
2 2

1 3
x   A2
2 2
10 a Horizontal translation left 5 A1A1
Reflection in the x -axis A1A1
Vertical translation up 3 A1A1

to exist, we require the domain of y  3   x  5 for which the function is one-to-


1 2
b For f
one. R1

y  3   x  5
2

is maximized when x  5 , and hence function has line of symmetry when


x  5 . R1
x   .
1
So function is one-to-one and f exists when x  5, A1

© Oxford University Press 2019 6


Mixed review

Graph of y  f  x  A1 for shape, A1 for domain

Graph of y  f 1  x  A1 for shape, A1 for domain

Note: There is a second possible solution where the domain of f is x  5

© Oxford University Press 2019 7


End of chapter test

Modelling relationships: linear and


quadratic functions
Section A. A calculator is not allowed

1 Find the equation of the line perpendicular to y= 6 − 3x which passes through the point ( 9,10 )

2 The line L1 shown on the set of axes below has equation 2x + 3y = 12. L1 cuts the x-axis at A
and cuts the y-axis at B.

a Find the coordinates of A and B.


M is the midpoint of the line segment [AB].
b Find the coordinates of M.
The line L2 passes through the point M and the point C (0, –1).

c Find the equation of L2.

d Find the equation of the line parallel to L2 that passes through the point A

3 a Factor the expression − x 2 + 4 x + 12

This is the graph of − x 2 + 4 x + 12

b Find the coordinates of A, B and C.

1
4 Consider f ( x ) = x 2 + bx + c which has an axis of symmetry at x = − , where the distance
2
between the zeros is 7 units.
a Find the zeros of f.
b Find the values of b and c.

© Oxford University Press 2019 End of chapter test 1


End of chapter test

5 Copy and complete the table by indicating the correct box for this parabola with equation

y = ax 2 + bx + c

Positive Zero Negative


a
c
b2 – 4ac
b

6 The diagram shows parts of the graphs of f ( x ) =  x 2 and g ( x )   4 ( x   – 5) − 2


2
=

The graph of f may be transformed into the graph of g by these transformations.


A vertical stretch with scale factor s, followed by a horizontal translation of h units and then a
vertical translation of v units.
Write down the values of s, h and v.

Section B. A calculator is allowed

7 Find the value of b for which 3x 2 + bx + 5 =0 will have two different real roots.

Give your answer correct to 2 decimal places.

8 a Draw a sketch showing the functions


= f ( x ) x=
2
g (x)
and 2 x
.

b Label their intersection points

c Write down the solution to the equation x 2 = 2x

© Oxford University Press 2019 2


End of chapter test

9 Farmer Phil has 80 meters of chicken wire available for constructing a chicken enclosure against
an existing stone wall. Phil wants the enclosure to be rectangular as shown in the diagram.

Let the length of the Phil’s enclosure be x metres.

1
a Show that the length of side AB is ( 40 − x)
2
b Find the area of the enclosure in terms of x
c Sketch a graph of the function to find the maximum area that Phil can enclose and state the
length, width and area.
10 The height (h metres) of a ball t seconds after it is thrown is given by the formula

h (t ) =10t − 4.9t 2 + 1.5

a Find the initial height of the ball.


b Sketch the graph of h.
Use your graph to write down the following
c The maximum height
d The time when the ball is at the maximum height.
e The time when the ball will hit the ground.
f Draw a line on your graph and find between what times the ball will be more than 5m off the
ground.

© Oxford University Press 2019 3


End of chapter test

Answers

1
1 If m = −3, the perpendicular slope is
3

y − y1= m ( x − x1 )

1
y − 10 =
3
( x − 9)
1
y − 10 = x −3
3

1
y
= x +7
3

2 a For A, y = 0 therefore
= 2 x  12 and 
= x 6.  A is (6,0)

For B, x = 0 therefore
= 3y  12
=  and  y 4.   B is (0,4)

 x + x1 y2 + y1   6 + 0 0 + 4 
=b M  2 = ,  ,   =  (3,2)
 2 2   2 2 

y mx + b
c =

y mx − 1
=

y2 − y1 2 − −1
=m = = 1 ⇒ y = x −1
x2 − x1 3−0

d y − y1= m ( x − x1 )

y − 0= 1 ( x − 6 )

y= x − 6

3 a − x 2 + 4 x + 12 = ( − x + 6 ) ( x + 2 )

b Find A and C by letting ( − x + 6 ) ( x + 2 ) =0,  which  gives x =−2,  6

A ( −2, 0 ) ,  
C ( 6, 0 )  

b 4
− (2 ) + 4 (2 ) + 12 =
2
Point is the vertex where x =
− =
− 2 and y =
= 16
2a −2

B (2,16 )

4 a The zeroes are equally spread around the axes.

1 7
zeros =− ± =−4 and 3
2 2

b ( x + 4) ( x − 3) =  12
x2 + x − b = 1, 3
c = −

5 Positive Zero Negative


a X
c X
b2 – 4ac X
b X

6 s =  4; h = 5;  
v = −2

© Oxford University Press 2019 4


End of chapter test

7 b2 − 4ac > 0 ⇒ b2 − ( 4 × 3 × 5) > 0 ⇒ b2 > 60 ⇒ b > 7.75, 7.75


b<−

8 a,b

c x = −0.766,  2

9 a 80
AB + BC + CD =
=AB    
CD
= and BC x

2 AB + x =
80

2 AB
= 80 − x

80 − x 1
AB
= = 40 − x
2 2

 1  1
b Area  AB 
= × BC = x  40 − x  = 40 x − x 2
 2  2

Length 40m, width 20m, area 800m

© Oxford University Press 2019 5


End of chapter test

10 h ( t ) =10t − 4.9t 2 + 1.5

a Initial height when t = 0 . h ( 0 )= 10 ( 0 ) − 4.9 ( 0 ) + 1.5= 1.5m


2

c The maximum height is the y value of the vertex which is 6.60m.


d The time when the ball is at the maximum height is the x value of the vertex which is
1.02secs
e The time when the ball will hit the ground is the x intercept of 2.18 secs.
f

The ball will be more than 5m off the ground between 0.449 secs and 1.59 secs.

© Oxford University Press 2019 6


End of chapter test

4 Equivalent representations:
rational functions
Section A. A calculator is not allowed

1 Find the reciprocals of

6 2
a 7 b −3 c   d −4
11 3

3π 2
e f Show that 3.5  and   are reciprocals
2 7

b
2 This function is defined as f ( x )= a +
x −c

a Write down the values of a and c.

b Given that the curve passes through the point (3, 9 ) , find the value of b.

4x − 3
3 Consider the function y = :
2x + 1

a Find the equations of the horizontal and vertical asymptotes.

b State the domain and range of the function.

1
4 Consider the function
= y −3:
x −2

a Write down the equations of the asymptotes of the function.

b Find the axes intercepts.

c Sketch the function −2 ≤ x ≤ 4, 8


− ≤ y ≤ 4.

Show the asymptotes as broken lines.

© Oxford University Press 2019 End of chapter test 1


End of chapter test

Section B. A calculator is allowed

x −1
(x)
5 Let f=
x +1
−3

a Find f −1 ( x )

b Write down the x-intercept and the y-intercept of the graph of f(x).

c Write down the equations of the asymptotes.

d Sketch the graph of f(x) for −4 ≤ x ≤ 4 and −5 ≤ y ≤ 8, including any asymptotes.

6 Travel expert Craig has a drive storing 60 Gb of holiday videos that he takes with him.

a Copy and complete this table, which shows the number of videos (x) and the size (y) that
Craig can store on his drive.

x 1 2 3 4 5 6 8 10 12 15 30 60

y 60 4

b Write down a function for y, the number of videos that Craig can take with him, in terms of
the individual size x.

c Use your GDC to sketch a graph of the function.

Craig wants to have space to take 35 videos with him.

d Show the line x = 35 on your sketch and label the intersection point A.

e What is the average size for Craig to take 35 videos?

3 − 2x
7 (x)
f=
2−x
( x ) e ( x − 3) −
g=
and 2

a Sketch both functions on the same axes with −1 ≤ x ≤ 6 and −7 ≤ y ≤ 10.

b Write down the x intercepts of f and g.

c Write down the solution to the equation f ( x ) = g ( x ) .

d For what value of x is f ( x ) > g ( x ) ?

8 The concentration, C (in mg/dl), of an antibiotic in a patient's bloodstream is modelled by


20t
C (t ) = 2 where t is the time (in hours) after taking the antibiotic.
t +5

a Sketch the graph of C ( t ) for the first 12 hours of medication.

b What is the concentration 4 hours after taking the antibiotic?

c The antibiotic must have 2 or more mg/dl in the bloodstream to be effective. If the first dose
was taken at 8am, at what time must the second dose be taken after 12 noon?

© Oxford University Press 2019 2


End of chapter test

Answers

1 1 11 3
1 a b − c   d −
7 3 6 14

2 7 7 2
e f 3.5 = × =1
3π 2 2 7
2 a=a 4, 2
= c

b
b 9= 4 +
3−2
9= 4 + b

b=5

4
3 a Horizontal asymptote when
= y = , 2
y . Vertical asymptote when 2 x + 1 =0, 1
x =−
2
b x ∈ , x ≠ −1, ,
y ∈  y ≠ 2

4 a x = 2,  3
y = −

b x intercept when y = 0 .

1 1 1 1
−3 =0 = 3 x −2 = x = 2
x −2 x −2 3 3
y intercept when x = 0

1 1 1
y = − 3 =− − 3 =−3
0−2 2 2
c

y −1 y −1
5 a=x −3 ⇒ x +3 = ⇒ ( x + 3) ( y + 1) = ( y − 1) ⇒ xy + x + 3y + 3 = y − 1
y +1 y +1
⇒ xy + 3y − y =−1 − x − 3 ⇒ xy + 2y =− x − 4 ⇒ y ( x + 2 ) =− x − 4

−x − 4
f −1 ( x ) =
x +2

b x intercept at ( −2, 0) and y intercept at ( 0, −4 )

© Oxford University Press 2019 3


End of chapter test

c −2 and y =
x = −1

6 a x 1 2 3 4 5 6 8 10 12 15 30 60
y 60 30 20 15 12 10 7.5 6 5 4 2 1

60
b y =
x
c and d

e 1.71 Gb

© Oxford University Press 2019 4


End of chapter test

7 a

b The x intercepts of f and g are 1.5  and  3.69

c f ( x ) = g ( x ) at the x values of the intercepts. x =  1.73  and  4.48 .

d f ( x ) > g ( x ) where f ( x ) is above g ( x ) . x < 1.73  and  2 < x < 4.48 .

8 a

b Draw the line t = 4 on the calculator and find the intersection point. 3.81mg/dl

b and c

c Draw the line C ( t ) = 2 and find the t value of the second intersection point.

=t   9.72
=  hrs 9hrs   43.2 mins

8am + 9hrs  43mins  makes the next dose at  5 : 43pm


© Oxford University Press 2019 5
Mixed review

4 Equivalent representations:
rational functions
1 In an arithmetic sequence, the first term is 6 and the fifth term is 22. Find the third term.

x +1
2 If f ( x
= ) 3x + 4 and g ( x ) = x −1
, find

a f −1 (2 )

b (g  f ) ( x )
3 Find the equation of the line passing through the points ( −1, 1
− ) and (3,5) .

8x
4 Find the asymptotes, domain and range of y = .
4x − 1

5 Find the term containing x 5   in the expansion of ( x + y )


12
.

2x + 1
6 Solve x 2 + 2 x − 3 = .
x +2

7 Find the equation of the line perpendicular to and passing through the midpoint of AB when A is
the point ( −8, −3) and B is (12, −7 ) .

8 Find the possible values of k in the quadratic equation 4 x 2 + kx =


−9 when there are two
identical solutions for x.

2 4 8 16
9 Find the sum of the infinite geometric series − + − +…
3 9 27 81

5
10 The rational function
= y + b has a vertical asymptote at x = 6 .
x−a

a Write down the value of a.

The function passes through (1,7 ) .

b Find the value of b.

c Write down the equation of the horizontal asymptote.

Exam-style questions

2 − 3x
11 Consider the function given by f ( x ) = .
x+4
a State the equation of the vertical asymptote. (1)
b State the equation of the horizontal asymptote. (1)
c Sketch the graph of y = f ( x ) . On your sketch, show clearly the asymptotes from
parts a and b as dashed lines. State the coordinates of the points where y = f ( x )
intersects the coordinate axes. (5)

© Oxford University Press 2019 1


Mixed review

4 1
12 Consider the functions f ( x )= 3 + ( x ≠ 5, x ∈  ) and g ( x ) = ( x ≠ 0, x ∈ ) .
x −5 x

a The function f ( x ) may be obtained from g ( x ) through a sequence of transformations.


State the required transformations (in order). (6)

b State the range of f ( x ) . (2)

c Find an expression for f −1 ( x ) . (4)


d State the domain and range of f −1
(x) . (2)

1 and goes through the point ( −1, 8 ) .


13 Line L1 is perpendicular to the line 4 x − 12y =

Line L2 goes through the points ( −2, −8 ) and (12,3) .

Find the exact coordinates of the point of intersection of L1 and L2 (9)

1 x
14 Consider the functions f ( x ) = , x ∈ , x ≠ −1 and g ( x=
) −1, x ∈  .
x +1 4

ax + b
a Given p ( x ) = g  f ( x ) , express p ( x ) in the form , where a, b, c, d are integers (3)
cx + d

b Hence find an expression for p−1 ( x ) (3)

c Hence find pppppp (1) , justifying your answer. (2)

15 Find the sum of the integers between 1000 and 2000 that are divisible by 7 . (5)

© Oxford University Press 2019 2


Mixed review

Answers

1 u3 = 14

2 a − 23

b 3 x +5
3 x +3

3 y = 3 x +1
2

4 Asymptote: x = 0.25

Domain: x ∈  : x ≠ 0.25

Range: y ∈  : y ≠ 2

5 792x5y7

6 x =−2, − 23  29
2

7 y = 5x – 15

8 k = ± 12

9 S = 2
5

10 a a = 6

b b=8

c y=8

11 a x = −4 A1

b y = −3 A1

A1A1 for each branch

A1 for both asymptotes

Intercepts ( 2
3
, 0 ) and ( 0, 21 ) A1A1

5
12 a Translation through   A1A1
0

Stretch by a scale factor 4 , parallel to the y -axis. A1A1

0
Translation through   A1A1
3

(Note: The first two transformations may be seen in any order)

© Oxford University Press 2019 3


Mixed review

b f ( x ) ≠ 3, f ( x ) ∈  A2

4
c x= 3 + M1
y −5

4
x −3 = A1
y −5

4
y −5 =
x −3

4
y= 5 + A1
x −3

4
f −1 ( x )= 5 + A1
x −3

d Domain is x ≠ 3, x ∈  A1

Range is f −1 ( x ) ≠ 5, f ( x ) ∈  A1

x 1
13 4 x − 12y = 1 ⇒ y = − M1
3 12

So L1 has gradient of −3 A1

y =−3x + c

Substituting ( −1, 8 ) gives 8= 3 + c , so c = 5 M1A1

L1 : y =−3x + 5

y −3 −8 − 3
Equation of L2 is = (or equivalent) M1
x − 12 −2 − 12

y −3 −11
=
x − 12 −14

−14y + 42 =
−11x + 132

L2 : 11x − 14y − 90 =
0 A1

Attempt to solve simultaneously: M1

11x − 14 (5 − 3x ) − 90 =
0

11x − 70 + 42 x − 90 =
0

53x = 160

160
x = A1
53

480 215
y =
5− =
− A1
53 53

 160 215 
 ,−  AG
 53 53 

© Oxford University Press 2019 4


Mixed review

 1 
 x + 1
p (x) 
14 a =  −1 M1
4

1
= −1 A1
4x + 4

1 − ( 4x + 4)
=
4x + 4

−4 x − 3
= A1
4x + 4

−4y − 3
b x = M1
4y + 4

x ( 4y + 4 ) =−4y − 3 A1

4 xy + 4 x =−4y − 3

4 xy + 4y =−3 − 4 x

4y ( x + 1) =−3 − 4 x

−3 − 4 x
y = A1
4 ( x + 1)

−3 − 4 x
f −1 ( x ) = AG
4x + 4

c p ( x ) is a self-inverse function. R1

Therefore pppppp (1) = 1 A1

15 Require the summation of 1001 + 1008 +  + 1995 which is an AP with first term 1001,
difference 7, and final term 1995. R1

1995 = 1001 + 7 ( n − 1) (or similar method to determine number of terms) M1

So require n = 143 terms A1

143
S143= 2 × 1001 + 142 × 7 M1
2 

= 214 214 A1

© Oxford University Press 2019 5


End of chapter test

5 Measuring change:
differentiation
Section A. A calculator is not allowed

1 Differentiate the following functions:

2
a f ( x ) = x 3 − 4x 2 − 6 x + 2 b g (x) =
x3

c h (x) = x d (x)
i= x2 + 5

2 a Express lim f ( x ) = 5 in a sentence.


x →2

b Write down the limit of the sequence 0.6, 0.66, 0.666, 0.6666, …

3 Differentiate the following:

5x + 3
a f (x) = b g (x) =
(5x + 1) 2x + 1
x2 + 1

1 2
4 Consider the parabola y= x −x−4
2

dy
a Find .
dx

b Find the equation of the tangent to the curve where x = 6.

c Find the equation of the normal where x = 6 in the form


ax + by = c,  where a,  b and c are positive integers .

5 The velocity, in ms–1, of a train approaching a station is given by v (=


t ) 20 − 4t , where t is the
time in seconds.

a What was the initial speed?

b When will the train be at rest in the station?

c What was its acceleration after 1.5secs?

6 Find the intervals for when the function f ( x ) =x 3 − 12 x − 1 is increasing and decreasing.

Section B. A calculator is allowed

7 The equation parabola is f (=


x ) ax 2 + bx. The gradient of the tangent to the curve at point P
(1,3) is 8. Find the values of a and b.

x +2
8 a Sketch the graph =
of y +3.
x −2

b Write down the equations of any asymptotes.

x +2
c Find lim +3.
x →∞ x −2

© Oxford University Press 2019 End of chapter test 1


End of chapter test

9 Consider the function f ( x ) = 3x 4 − 4 x 3 − 2 .

a Find all turning points, and determine their nature (you should justify your answers).

b Find the coordinates and nature of any inflexion points.

c Sketch the graph of f, with −1 ≤ x ≤ 2  and − 4 ≤ y ≤ 8 labelling a horizontal inflexion A,


minimum point B and a non-horizontal inflexion C.

10 ( x − 1) 10 ( x − 2 ) 20 ( x − 3)
10 The function f ( x ) = has f ′ ( x ) = − and f ′′ ( x ) =
x2 x3 x4

a Find the zeros of f(x).

b Find the coordinates of the local maximum point.

c Find the intervals where f(x) is concave up.

Given f(x) has the x and y axes as asymptotes,

d Sketch the function for x ≥ 0 .

© Oxford University Press 2019 2


End of chapter test

Answers
1 a f ′ ( x ) = 3x 2 − 8 x − 6 1
u x2 + 5 ⇒ y =
d = u2
2
b g (=
x) = 2 x −3 dy dy du
= ×
x3 dx du dx
6
g′ ( x ) =
−6 x −4  or − dy 1 − 12
x4 = u × 2x
dx 2
1
h (=
x)
1
dy
c x x2 …
= ( )

= x x2 + 5   2
dx
1
1 − x
h′ ( x ) = x 2
or
2 2
x +5

2 a The limit of f ( x ) , as x approaches positive 2, is 4.

2
b lim un =
x →∞ 3

3 a f (x) =
5x + 3 b g (x) =
(5x + 1) 2x + 1
x2 + 1
Use the chain rule for v ′ ( x )
u=5x + 3,  v =x 2 + 1
u′ 5, 
= = v ′ 2x Let=
y 2x + 1

vu′ − uv ′ u 2x + 1
=
f ′(x) =
v2 1
y = u2

f ′(x) =
(x 2
)
+ 1 (5 ) − (5 x + 3 ) ( 2 x )
dy dy du
= ×
(x )
2
2
+1 dx du dx

5x 2 + 5 − 10 x 2 + 6 x   dy 1 − 12
f ′(x) = = u ×2
(x ) dx 2
2
2
+1
dy 1
1
(2x + 1) 2  or 

−5x 2 + 6 x + 5  =
f ′(x) = dx 2x + 1
(x )
2
2
+1
Using the product rule:

u =(5x + 1)   v = 2 x + 1
1
(2x + 1) 2

u′ 5  
= =v′

g′ (=
x ) uv ′ + vu′

5x + 1
g′ ( x=
)   +5 2 x + 1
2x + 1
dy
4 a = x −1
dx
1
(6 ) − 6 − 4= 8. The coordinate is 6,
( 8)
2
b When x= 6, y=
2
When x 6, the gradient is 6
= = −1 5

y − y1= m ( x − x1 )

y − 8= 5 ( x − 6 )

y − 8 = 5x − 30
y 5x − 22
=

© Oxford University Press 2019 3


End of chapter test

1
=c When x 6, the gradient of the normal is −
5
y − y1= m ( x − x1 )

1
y − 8 =−
5
( x − 6)
1 6
y − 8 =− x+
5 5
1 46
y =
− x+
5 5
1 46
x+y =
5 5
x + 5y =
46
5 a Initial speed = 20 – 4(0) = 20 ms–1
b 20 − 4t = 0, 5
t =
At rest after 5 seconds.
c Acceleration = V’(t) = –4ms–2
6 f ( x ) =x 3 − 12 x − 1

f ′ (=
x ) 3x 2 − 12

( x ) 3(x 2 − 4)
f ′=

f ′ ( x ) =3 ( x + 2 ) ( x − 2 )

f ′ ( x ) = 0 when  x = −2, 2

Choosing a value just below −2 such  as − 2.1

f ′ ( −2.1) =3 ( −2.1) − 12 =1.23


2

Choosing a value between −2 and  2 such  as  0 ,

f ′ (0) =
3 ( 0 ) − 12 =
2
−12

Choosing a value just above 2 such  as 2.1

f ′ (2.1) 3 (2.1) −=
2
= 12 1.23

x x ≤ −2 −2 < x < 2 x ≥2
Sign of f’(x) + − +
f(x) increasing decreasing increasing
f ( x ) is increasing when  2
x ≤ − and  2
x ≥

f ( x ) is decreasing when −2 < x < 2

7 Where x = 1 and y = 3, 3 = a + b,
( x ) 2ax + b
f ′=

8 2a + b
=
Solving simultaneously,
a = 5  and  q = −2

© Oxford University Press 2019 4


End of chapter test

8 a

b=
x 2, 4
= y

x +2
c lim +3 =4
x →∞ x −2

9 a f ′ ( x ) = 12 x 3 − 12 x 2 = 12 x 2 ( x − 1)

f ′ ( x ) 0, 
= = x
when  0, 1.

When x = 0
′′ ( x ) 36 x 2 − 24 x  
f=

f ′′ ( 0 ) 36 ( 0 ) − 24 ( 0 ) = 0
2
=

The y value is  f ( 0 ) =3 ( 0 ) − 4 ( 0 ) − 2 =−2 .
4 3

Horizontal inflexion point at ( 0, 2


− )

When x = 1

f ′′ (1) = 36 (1) − 24 (1) = 12


2

The y value is  f (1) =3 (1) − 4 (1) − 2 =−3 .


4 3

Minimum point at (1, –3).


b f ′′ ( x ) = 0  at an inflexion point.

2
f ′′ ( x ) = 36 x 2 − 24 x = 12 x (3x − 2 ) = 0  when  x = 0, 
3
4 3
2 2 2 70
f =3   − 4   − 2 =− or − 2.59 .
3
  3
  3
  27

2 
There is a non-horizontal point of inflexion at  , 2.59  .
3 

© Oxford University Press 2019 5


End of chapter test

10 a 10 (=
x − 1)   0  when 
= x 1.

b Maximum point where f ′ ( x ) = 0 , –10(x – 2) = 0, x = 2

It is a maximum point when


f ′′ ( x ) < 0
20 (2 − 3) 5
f ′′ (2 ) = = −
24 4
10 (2 − 1)
is f (2 )
The y value= = 2.5 .
22

The local maximum point is (2,2.5) .

c f ( x ) is concave up where f ′′ ( x ) > 0

20 ( x − 3) > 0, 3
x >

© Oxford University Press 2019 6


Mixed review

5 Measuring change: differentiation

1 Let f (x ) = x 2 − 7 x + 6

a Find the equation of the axis of symmetry of the graph of f.

f (x ) can also be expressed in the form f (x ) =(x − h)2 + k for some constants h and k.

b Write down the exact value of h

c Find the value of k.

d By using the second derivative test, prove that (h, k) is a minimum point of f (x ) .

3x − 8
2 Consider the functions f(x) = 2x – 5 and g(x) = ,x ≠ 3
x −3

a Find g−1(2).

b What do you deduce about the function g(x)?

(
c Find x such that f  g −1 ) (x) = 0 .
d State the equation of any asymptotes for .

3 An arithmetic sequence has r as its common difference, and a geometric sequence has r as its
common ratio. For both sequences, a1 = 1.

a Write down the first four terms of both sequences, in terms of r.

b If the sum of the third and fourth terms of the arithmetic sequence is equal to the sum of
the third and four terms of the geometric sequence, find the three possible values of r.

c Find the value of r from part b for which S∞ exists.

d For the value of r that you found in part c, find the sum of the first 20 terms of the
arithmetic sequence.

4 The graph of f (x ) =a(x − b)(x − 2) has axis of symmetry x = −0.5 and y-intercept at (0,-6).

a Find the value of b.

b Find the value of a.

y kx − 10 is tangent to the curve y = f ( x ) .


c Find the value(s) of k such that the line =

5 A function f has its derivative given by f '(x ) = 2 x 2 − 2kx − 7 , where k is constant.

a The function has an inflexion point at x=–1. Find the value of k.

b Find the equation of the tangent to the curve f at (1,–5).

© Oxford University Press 2019 1


Mixed review

8 3 1
6 Consider the function f (x )= x + x 2 − 3x −
3 3

a Find the coordinates of the local maximum and local minimum points of y = f ( x ) .

0 
b The graph of f is translated to the graph of g by the translation   . Find the value(s) of k
k 
such that g(x ) = 0 has exactly one root.

Exam-style questions

Find

a the domain of f (1)

b the zeros of f (1)

c the range of f (1)

d the intervals where the derivative of f is positive (2)

e the coordinates of a point on the graph where the tangent to the graph is horizontal (1)

f the points in the domain of f where the derivative is not defined. (1)

8 a Determine which linear function (1), (2) or (3) is the graph of the derivative of each
quadratic curve (A), (B) and (C). (3)

1 2 3

A B C

b State the zeros of the quadratic function in (A); (1)

c Write down the gradient of the tangent to the quadratic graph in (B) at x = 0 . (1)

© Oxford University Press 2019 2


Mixed review

9 The following table shows the annual profits (in thousands of dollars) for a company during a
5-year period.

Year 2000 2001 2002 2003 2004

Profit 98.5 100.9 101.1 101.2 102.3

a Find

i the average rate at which the profits varied between 2000 and 2002;

ii the average rate at which the profits varied between 2002 and 2004. (3)

b Compare the values obtained in (a) and state their meaning in the context of the
question. (2)

10 Consider the graph of the function defined by f ( x ) = x 3 + 2 x 2 − 4 x − 5 .

a Show that f ′ ( x ) =(3x − 2 ) ( x + 2 ) . (2)

b Find the coordinates of the points A and B on the graph of f where the tangents to the
graph are horizontal. (3)

c Find the equation of the normal to the curve at x = 2 . (6)

d Hence find the exact coordinates of the points at which the normal found in part (c)
meets the tangents found in part (b). (4)

y ax 2 + bx contains the point A (1,5) . The tangent L to=


11 The graph of= y ax 2 + bx at A has
gradient 7.

a Find the values of the real parameters a and b . (6)

b Write down an equation for the tangent L . (1)

c Find the coordinates of the point B on the graph where the tangent to the graph is
horizontal. (3)

d State the geometrical significance of the point B on the graph of=


y ax 2 + bx . (1)

© Oxford University Press 2019 3


Mixed review

Answers

1 a x = −7
2
b h= 7
2
c k = 6.25 d (h, k) is a minimum point.

2 a g−1(x) = 2 b g(x) is self-inverse c x=1 d x=3

3 a Arithmetic: 1, 1 + r, 1 + 2r, 1 + 3r.

Geometric: 1, r, r2, r3

b r = 2, − 23 − 5
2
, − 23 + 5
2

c r = − 23 + 5
2
≈ −0.382

d S20 = −52.57 (2 d.p.)

4 a b = −1

b a=3

c k= 12
3
− 9 at ( 2
3 )
, −2 − 6 3 , k = −12
3
− 9 at ( −2
3
, −2 + 6 3 )
5 a k = 2.5 b y = −10x + 5

6 a ( 1
2
, − 54 ) is a local minimum.

(− 3
4 48 )
, 65 , is local maximum.

b k > 5
4

7 a −2 < x < 3 A1

b −2 and 0 A1

c 0≤y ≤2 A1

d −2 < x < −1, 0 < x < 2 A1A1

e Any point of the form ( x,2 ) where 2 ≤ x ≤ 3 A1

f −2, −1, 0, 2 and 3 A1

8 a 1C, 2A and 3B A1A1A1

b −3 and 0 A1

c −3 A1

101.1 − 98.5
9 a i = 1.3 M1A1
2002 − 2000

102.3 − 101.1
ii = 0.6 A1
2004 − 2002

b The average increase in annual profits was greater between 2000 and 2002 ($1300 per
year) than it was between 2002 and 2004 ($600 per year). R1A1

10 a f ′ ( x ) = 3x 2 + 4 x − 4 M1A1

=(3x − 2 ) ( x + 2 ) AG

© Oxford University Press 2019 4


Mixed review

b f ′(x) = (3x − 2 ) ( x + 2 ) = 0 for horizontal tangent M1

2 175 2 175 
x = ⇒y =− so point is  , −  A1
3 27 3 27 

x =−2 ⇒ y =3 so point is ( −2, 3) A1

c f ′ (2 ) = 16 A1

1
mtangent mnormal =−1 ⇒ Gradient of normal: − M1A1
16

f (2 ) = 3 A1

1
y − 3 =−
16
( x − 2) ⇒ x + 16y =
50 (or equivalent) M1A1

175 4150
Tangent y = − meets normal x + 16y =
50 when x = M1
27 27

Tangent y = 3 meets normal x + 16y =


50 when x = 2 M1

 175 4150 
So points are  − ,  and (2,3) A2
 27 27 

y ′ 2ax + b
11 a = A1

Attempt to form two simultaneous equations in a and b M1

7 2a + b
=
 A1A1
 5= a + b

Solving gives a = 2 and b = 3 A1A1

b y − 5 = 7 ( x − 1) ⇒ y = 7 x − 2 A1

3
c y ′ ( x ) =4 x + 3 =0 ⇒ x =− M1
4

 3 9
y −  =− M1
 4  8

 3 9
So coordinates are B  − , −  A1
 4 8

d B is the minimum point (accept ‘vertex’) of the parabola. R1

© Oxford University Press 2019 5


Exam practice

Exam practice: chapters 1 – 5

1 P2: Give your answers to parts a to e to the nearest dollar.

When Maria turned 18 her grandparents gave her three options of how she might receive
her birthday present.

Option A: She receives $50 each month for three years.

Option B: She receives $1 in the first month, $4 in the second month, $7 in the third
month, increasing by $3 each month for three years.

Option C: $20 in the first month and increasing by 5% each month for three years.

a If Maria chooses Option A, calculate the total value of her present. (2)

b If Maria chooses Option B, calculate

i the amount of money she will receive in the 12th month

ii the total value of her present at the end of the three-year period. (4)

c If Maria chooses Option C, calculate

i the amount of money she would receive in the 12th month;

ii the total value of her present at the end of the three-year period. (4)

d State which of options A, B or C Maria should choose to give her the greatest total
value of her present. (1)

2 P2: On 1 Jan 2018, a company purchased a vehicle costing $26 000. The company expects the
vehicle to be operational for 4 years, at the end of which it can be sold for $6 000.

a Calculate the total depreciation of the vehicle at the end of 2021. (1)

b Hence find the average annual depreciation. (2)

c Assuming that the vehicle depreciates at a constant rate during the 4-year period,
sketch a graph to represent the value of the vehicle against time for 0 ≤ t ≤ 4 years.

3 P1: Let f ( x ) = a ( x − b ) + c . The vertex of the graph of f is at (1, −4 ) .


2

a State the value of b and the value of c . (2)

The graph passes through (3, 4 ) .

b Find the value of a . (3)

4 P2: In an arithmetic sequence, the third term is 15 and the fifth term is 21.

a Find the common difference. (2)

b Find the first term. (2)

c Find the sum of the first 50 terms of the sequence. (3)

© Oxford University Press 2019 1


Exam practice

5 P1: Without expanding any brackets, find the derivative of the following functions.

a ( )
f ( x ) = 3x 2 + 1 (1 − 2 x ) (3)

(x )
2
b g (=
x) 2
+ 3x (2)

4x − 1
c h (x) = (3)
2−x

6 P2: A rock is thrown vertically upward from the surface of the moon at a velocity of v0 m s–1.
After t seconds its height is given by=
h 24t − 0.8t 2 metres.

a Find expressions for the velocity and acceleration of the rock after t seconds. (3)

b Hence state

i the value of v0

ii the acceleration due to gravity on the moon. (2)

c Find the maximum height the rock reaches and state the value of t when this
occurs. (3)

d State how long it takes for the rock to fall back to the moon’s surface. (2)

e Find the values of t for which the rock is at half of its maximum height. (3)

7 P2: The sum of the first n terms of an arithmetic sequence is given by =


Sn 2n2 + 4n .

a Write down the value of

i S1

ii S2 . (2)

The nth term of the arithmetic sequence is given by un .

b Find the value of u2 . (2)

c Find the common difference of the sequence. (2)

d Hence show that u


=n 4n + 2 for all n ∈  . (2)

e Find the least value of n for which Sn > 100un . (4)

8 P1: Three consecutive terms of an infinite geometric sequence are a + 2, 6 and a + 7 ,


where a ∈  .

a i Write down two expression for the common ratio, r , in term of a .

ii Hence, show that a satisfies the equation a2 + 9a − 22 =


0. (4)

b i Find the possible values of a .

ii Find the possible values of r . (4)

The sum of all the terms of sequence is 9.

c State which value of r leads to this sum. Justify your answer. (2)

d Find the first term of the sequence. (2)

© Oxford University Press 2019 2


Exam practice

9 P1: The following diagram shows the graph of a function f defined for x ≠ 1 .

a State

i the range of f

ii the equations of the asymptotes to the graph of f . (3)

b On the same axes, sketch and label

i the graph of g ( x )= f ( x − 2 ) + 1

1
ii the graph of h ( x ) = . (3)
f (x)

10 P1: The diagram below shows two consecutive rows of Pascal Triangle.

1 4 a b 1

1 c 10 d e 1

a Write down the values of a, b, c, d and e . (5)

b Find the coefficient of x3 in the expansion (2 x − 1) .


5
(3)

x +2 3x − 1
11 P2: Let f ( x ) = and g ( x ) = .
5 x −2

a State the largest possible domain of g . (1)

b Find an expression for the inverse of f . (3)

c Hence

i (
determine g f −1 ( x ) ) ii state the domain of g  f −1 . (3)

The line y = k is an asymptote of the graph of g .

d Find the value of k . (2)

3x − 1 x +2
e Solve the inequality ≥ (6)
x −2 5

© Oxford University Press 2019 3


Exam practice

f (x ) kx ( x − 6 ) , where k > 0 .
2
12 P1: Consider the function =

a Find an expression for f (2 ) in terms of k . (2)

b Find an expression for f ′ ( x ) in terms of k . (3)

c Hence show that the graph of the function f has a local maximum at x = 2 . (2)

Given that f (3) = 54 ,

d i Find the value of k

ii Write down the coordinates of the local maximum of f . (3)

e Sketch the graph of the function f for 0 ≤ x ≤ 7 . (2)

Let T be the tangent to the graph of the function f at x = 3 .

f Find the gradient of T . (2)

The line L passes through the point (2, 10) and is perpendicular to T.

g Find an equation for L in the form ax + by


= + c 0, a, b, c ∈  . (3)

13 P1: A rectangle ABCD has dimensions 16 cm by 12 cm. In a Mathematics class, students are
instructed to construct a sequence of right-angled triangles by successively dividing the
sides of quadrilaterals as shown in the diagram below.

Stage 1 Stage 2 Stage 3

The midpoints of the sides of ABCD are joined to form a quadrilateral EFGH and four
triangles, one of which is shown in red (Stage 1). Then the process is repeated twice, as
shown in stages 2 and 3, to obtain right-angled triangles that are shown in red.

a**Explain why the quadrilateral EFGH is a rhombus. (2)

b***Find the values of the areas A1, A2 and A3 of the right-angled triangles shown in the
diagram. (4)

c**Suppose that the process of obtaining these right-angled triangles is repeated


indefinitely. Find an expression for the area An. (2)

d**Show that the sum to infinity of A1 +A2 +…. =32 cm2 (2)

14 P1: Let h = f  g for functions f and g, where f (3) = 5 , f ′ (3) = −2 , g (2 ) = 3 and g′ (2 ) = 4 .

a Show that h is decreasing at x = 2 . (3)

b Find the equation of the tangent to the graph of h at x = 2 . (3)

© Oxford University Press 2019 4


Exam practice

2kx
15 P1: Consider the family of rational functions defined by rk ( x ) = , k ≠ 0.
x−k

a Find an expression for the inverse of rk . (4)

The horizontal asymptote of the graph of rk ( x ) is y = 4 .

b Find the value of k . (2)

c For the value of k found in part b, state the domain of the inverse of rk . (1)

16 P2: A particle, A, is moving along a straight line. The displacement, s metres, of A after
1 4
t seconds since its motion began is given by s(t ) = t − 2t 3 + 3t 2 + t .
4

a Sketch the graph of s = s(t ) , for 0 ≤ t ≤ 6.5 with s on the vertical axis and t on the
horizontal. Mark on your sketch the local maximum and minimum points, and the
intercepts with the t -axis. (4)

b State the range of s = s(t ) for 0 ≤ t ≤ 6.5 . (2)

c Find an expression for the velocity v = v(t ) of A. (2)

d Write down the intervals of time for which the magnitude of the velocity of the particle
is increasing. (2)

e Hence state, with a reason, the time at which the velocity of A is at a local
maximum. (2)

© Oxford University Press 2019 5


Exam practice

Answers
1 a 50 × 12 × 3 =$1800 M1A1
b i 1 + 11 × 3 =$34 M1A1

2 × 1 + 35 × 3
ii × 36 =
$1926 M1A1
2
c i 20 × 1.0511 =
$34 (nearest dollar) M1A1
36
1.05 − 1
ii $1917 (nearest dollar)
× 20 = M1A1
1.05 − 1
d Option B A1
2 a 26 000 − 6000 =
$20 000 A1
20000
b = $5000 M1A1
4
c y − intercept at 26 000; A1 Straight line with end-point at (4,6000)

A1
3 a b = 1 and c = −4 A1A1

4 = a (3 − 1) − 4
2
b M1A1

a=2 A1
4 a 21
= 15 + 2d M1
d=3 A1
b 15 = u1 + 2 × 3 M1

u1 = 9 A1

2 × 9 + 49 × 3
c
= S50 × 50 M1A1
2
S50 = 4125 A1

5 a f ′(x) = (3x 2
) ( )
+ 1 ′ (1 − 2 x ) + 3x 2 + 1 (1 − 2 x )′ M1

(
f ′ ( x ) = 6 x (1 − 2 x ) − 2 3x 2 + 1 ) A1

f ′(x) =
−18 x 2 + 6 x − 2 A1

b (
g′ ( x ) = 2 ( 2 x + 3 ) x 2 + 3 x ) M1A1

c h'(x) =
( 4x − 1)′ (2 − x ) − ( 4x − 1) (2 − x )′ M1
(2 − x )
2

4 (2 − x ) + ( 4 x − 1)
h'(x) = A1
(2 − x )
2

7
h'(x) = A1
(2 − x )
2

© Oxford University Press 2019 6


Exam practice

6 a v 24 − 1.6t m s–1, a = −1.6 m s–1


= M1A1A1

b i v0 = 24 m s−1 A1

ii 1.6 m s−2 A1
c Use of GDC to find maximum M1
180 m after 15 seconds A1A1
d Use of GDC to find zero, or double result from c M1
30 seconds A1
e Find intersection with line v = 90 M1
4.39 s and 25.6 s A1A1
7 a i S1 = 6 A1

ii S2 = 16 A1
b u=
2 S2 − S1 M1

u2 = 10 A1

c u=
1 S=
1 6 R1

d = u2 − u1 = 4 A1

d un =6 + 4 ( n − 1) M1A1

u
=n 4n + 2 AG

e Sn =6n + 2 ( n − 1) n A1

Use GDC to solve 6n + 2 ( n − 1) n > 400n + 200 M1A1

199 A1
6 a+7
8 a i and A2
a+2 6
6 a+7
ii = M1
a+2 6

( a + 7) ( a + 2) =
36 A1

a2 + 9a − 22 =
0 AG
b i Attempt to solve a2 + 9a − 22 =
0 M1
a = 2, a = −11 A1

2 3
ii r =
− ,r = A1A1
3 2
2
c r = − , the common ratio must have absolute value less than 1. R1
3
u1
d =9 M1
2
1+
3
u1 = 15 A1

9 a i y ≠1 A1
ii =
x 1,
= y 1 A1A1

© Oxford University Press 2019 7


Exam practice

b Attempt to transform the graph of f A1

10 a a= 6, b= 4, c= e= 5, d= 10 A1A1A1A1A1

10 × 23 × ( −1) =80
2
b M1A1A1

11 a x ≠2 A1
y +2
b x = M1
5
Solve for y M1
f −1
( x=) 5x − 2 A1

3 (5 x − 2 ) − 1
c i ( ) (5 x − 2 ) − 2
g f −1 ( x ) = M1

15x − 7
( )
g f −1 ( x ) =
5x − 4
A1

4
ii x ≠ A1
5
3x − 1
d lim g ( x )
= lim
= 3 M1
x →+∞ x −2
x →+∞

k =3 A1
e

Labelled axes with appropriate scale A1


Correct graphs A1A1
x ≤ −0.071, 0.660 ≤ x < 2, 2 < x ≤ 14.1 A1A1A1

© Oxford University Press 2019 8


Exam practice

f (2 )= 2k (2 − 6 ) = 32k
2
12 a M1A1

f ′(x ) = k ( x − 6 ) + 2kx ( x − 6 )
2
b M1A1

f ′ ( x ) =k ( x − 6 ) (3x − 6 ) A1

c f ′ (2 ) = 0 A1

( ) ( )
f ′ 2− > 0 and f ′ 2+ < 0 (or considers second derivative) R1

f has a local maximum at x = 2 AG


i 3k (3 − 6 ) = 54 ⇒ k = 2
2
d M1A1

ii (2, 64) A1
e

Correct zeros A1
Correct shape with one maximum at x = 2 A1
f f ′ ( 3=
) 2 (3 − 6 ) (3 × 3 − 6 ) M1

f ′ (3) = −18 A1

1
g y − 10
=
18
( x − 2) M1A1

x − 18y + 178 =
0 A1

13 a All the sides are equal as they are hypotenuses of equal right-angled triangles; R1
the diagonals are parallel to the sides of the rectangle ABCD and are therefore
perpendicular. R1
So EFGH is a rhombus. AG
8×6
b =
A1 = 24 ; A1
2
1 1 3
A2 = 24 × = 6 ; A3 = 6 × = M1A1A1
4 4 2
n −1
1 96
c An =×
24   = M1A1
4 4n

24
d=S = 32 M1A1AG
1
1−
4

© Oxford University Press 2019 9


Exam practice

14 a (
h′ (2 ) = f ′ g (2 ) g′ (2 )) M1

=f ′ (3) × 4 =−8 A1

<0 R1
h is decreasing at x = 2 AG
b (2 ) f g =
h= (2) f=(
(3) 5 ) A1

y − 5 =−8 ( x − 2 ) M1

y =−8 x + 21 A1

2ky
15 a x = M1
y −k

kx
xy − kx= 2ky ⇒ y= M1A1
x − 2k
kx
rk −1 ( x ) = A1
x − 2k
b lim rk ( x ) =4 ⇒ 2k =4 M1
x →∞

k =2 A1
c x ≠4 A1

16 a Max (1.47,2.77) Min (4.67,-14.7) A1A1


Zeros: 0 and 2.46 A1
shape and correct domain A1

b −14.7 ≤ t ≤ 66.0 M1A1


c v(t ) = t 3 − 6t 2 + 6t + 1 M1A1
d 0 < t < 0.586 and 3.41 < t < 6.5 M1A1
e

the accelaration changes sign from positive to negative at 0.586 seconds, so the velocity
is at a local maximum then. M1A1

© Oxford University Press 2019 10


End of chapter test

6 Representing data: statistics for


univariate data
Section A. A calculator is not allowed

1 Classify the following data sets using the words

Qualitative. Quantitative. Discrete. Continuous.

a The number of children in your family.

b Your favourite color.

c The height of the students in your class.

d The time that it takes to go home from school.

2 Joao is studying the average weight of fish caught at a port.

Choose from convenience, simple random, systematic, stratified or quota to classify each of the
following sampling techniques that Joao might use.

a Joao selects any 100 fish at random.

b A random fish is chosen and then every 20th fish after that until Joao has 100 fish.

c The fish consist of 80% round fish and 20% flat fish. Joao selects 80 round and 20 flat fish.

d A sample of 100 fish is taken by organizing the fish by 5 species and then taking 20 from
each species

3 This table shows the ages of my relatives and the number of coffee drinkers.

Ages (x) Number of coffee drinkers (f)

20 ≤ x < 30 5

30 ≤ x < 40 4

40 ≤ x < 50 3

50 ≤ x < 60 2

60 ≤ x < 70 3

a Find the mean coffee drinking age.

b Represent this data on a histogram.

© Oxford University Press 2019 End of chapter test 1


End of chapter test

4 Bilbo Air records the amount of time, in minutes. that its planes are delayed.

Delay time Number of flights

0 ≤ x < 30 5

30 ≤ x < 60 15

60 ≤ x < 90 33

90 ≤ x < 120 21

120 ≤ x < 150 11

150 ≤ x < 180 7

180 ≤ x < 210 5

210 ≤ x < 240 3

a Construct a cumulative frequency table for Bilbo Air’s data.

b Draw a cumulative frequency graph to represent the data.

Use your cumulative frequency graph to estimate

c The median delay time.

d The interquartile range of the delay times.

5 This diagram is a box plot for a data set.

a Write down the median.

b If the range is 42 and the interquartile range is 16, find the values of p and q.

6 The cumulative frequency diagram shows the


number of push-ups done in one minute by a
group of 80 athletes.

a Write down the median.

b Find the interquartile range

c The top 10% of athletes completed at least


how many push-ups?

© Oxford University Press 2019 2


End of chapter test

Section B. A calculator is allowed

7 Yuyu has taken 5 class tests and her average is 88. 6%. How much must she score in the next
test to finish with an average of 90%?

8 The table shows the numbers of bicycles that students have owned.

Bicycles 0 1 2 3 4

Students 28 24 20 17 11

a Is the data discrete or continuous?

b Write down the mode.

c Find the mean number of bicycles per student.

d Find the median.

9 Khabib has a papaya orchard. On a Sunday he harvests 72 green papaya and 28 yellow. The
mean weight of a green papaya is 1.79kg and the mean weight of a yellow papaya is 1.62kg.

Find the mean weight of Khabib’s 100 papayas.

10 Here is a group of university friends and their ages

Age (yrs) 19 20 21 22 23

f 6 8 7 11 9

Find

a the mean

b the standard deviation

c the variance

If the friends meet up for an alumni fund raiser after x years, what will be their

d mean

e standard deviation

© Oxford University Press 2019 3


End of chapter test

Answers

1 a Quantitative. Discrete. b Qualitative.

c Quantitative. Continuous. d Quantitative. Continuous.

2 a Simple random b Systematic

c Quota d Stratified.

3 a=x
∑ fm
=
(25 × 5) + (35 × 4) + ( 45 × 3) + (55 × 2) + (65 ×=
3) 705
= 41.5
∑f 5+4+3+2+3 17

4 a Delay time Cumulative frequency

x < 30 5

x < 60 20

x < 90 53

x < 120 74

x < 150 85

x < 180 92

x < 210 97

x < 240 100

© Oxford University Press 2019 4


End of chapter test

c The median delay time is 87 mins.

d IQR = Q3 − Q1 = 123 − 65 = 58 mins

5 a 16

b p = 45 – 42 = 3, q = 28 – 16 = 12

6 a The median is 32

b IQR = Q3 − Q1 = 40 − 22 = 18

c 10% of 80 = 8.

The 72nd athlete completed 48 push-ups.

7 Yuyu′ s total
= 88.6 ×=
5 443

To finish with 90%, Yuyu needs a total of 90 × 6 =


540

Yuyu must score 540 − 443 =


97% in the next test.

8 a Discrete

b 1 bicycle.

Total number of bicycles 163
c Mean
= = = 1.63 bicycles
Total number of students 100

th th
 n + 1  100 + 1 
d Median
=  =   =  50.5
= th
1b
 icycle
 2   2 

=9 Mean
(=
72 × 1.79 ) + (28 × 1.62 )
1.74kg
100

∑ fx 870
10 a=x = = 21.2
∑f 41

b 1.37

c the variance 
= (1.370636
= …)2 1.88

d 21.2 + x

e 1.37

© Oxford University Press 2019 5


Mixed review

6 Representing data: statistics for


univariate data
2
f (x)
1 Consider the curve = +1.
x −3

a Write down the equation of the vertical asymptote.

b Find the value at which the curve crosses the x-axis.

2 If f ( x ) = 2 x 2 − 8 x + 5 ,

a Find the values of a, h and k, when f ( x ) is written in the form a ( x − h ) + k


2

b Write down the equation of the axis of symmetry.

3 Find the coefficient of x 3 in the expansion of (2 − x ) .


5

4 Find three positive integers a, b and c that have a mean of 9, a median of 11 and a range
of 10.

5 Solve e=
x
x2 − 2

6 a Write an expression for the nth term of the arithmetic sequence 14, 11, 8, 5,

b Find the 50th term.

7 Find the coordinates of the maximum point of the curve f ( x ) =x 3 − 10 x 2 + 12 x − 2

1
x ) 4 x − 4 and g ( x )= 3 −
8 If f ( = x
2

a Find g −1.

b Solve the equation ( f  g ) ( x ) = −20

9 If Jo invests $1000 at 15% interest per annum, compounded monthly, how many months will it
take for Jo’s investment to exceed $3000?

10 This diagram shows the graph of f ( x ) . Make a copy of and sketch the graph of f ′ ( x ) on the
same axes.

© Oxford University Press 2019 1


Mixed review

Exam-style questions

11 A maths textbook states that if data is skewed to the right then it has the following properties.

i The mean is always greater than the mode.

ii The median is always greater than the mode.

iii The mean is usually greater than the median.

a For the following data:

1, 3, 4, 4, 6, 8, 10, 12

Find the mode, the median and mean. (3)

b Determine which of the statements i, ii and iii hold for this data. Hence, deduce
whether this book would classify this data as skewed to the right. (2)

12 A canned food manufacturer wants to cut production costs and reduce the amount of
aluminium used to make the cans. The company requires cylindrical cans with volume 500 cm3.
Let h represent the height of a can, and x represent the radius of the base, as shown.

500
a Show that h = . (2)
π x2

1000
( x ) 2π x 2 +
b Show that the total surface area of the can is given by S=
x
. (2)

c Hence find the values of radius and height that minimize the total surface area of the can.
(4)

13 a A set of data is made up of the first seven terms of an Arithmetic Progression, with first
term 4 and common difference of 3.

For this data, find

i the median

ii the mean

iii the interquartile range. (6)

b A set of data is made up of the first seven terms of a Geometric Progression, with first
term 1 and common ratio 2.

For this data, find

i the median

ii the mean (giving your answer as a fraction)

iii the interquartile range. (6)

c State which of these two data sets has mean equal to its median. Suggest a reason for this.
(2)

© Oxford University Press 2019 2


Mixed review

14 A set of data has a mean of 16, a mode of 14, a median of 18, an interquartile range of 20 and
a variance of 36.

a All the values in this data set have 5 added to them. Write down the new value of the

i mean ii mode iii median iv interquartile range


v variance. (5)

b A particular value x was an outlier in the original set of data. Determine whether the
new value of x + 5 will still be an outlier in the new set. Justify your answer. (3)

c All the values in the original set of data are multiplied by −2 . Write down the new value
of the

i mean ii mode iii median iv interquartile range


v variance vi standard deviation. (6)

15 For a set containing data for an entire population, the mean is defined by µ =
∑x and the
n

variance by
= σ2 ∑x 2

− µ2 .
n

A population data set with five values is: 1, 6, 8, x, y . For this data,

y >x

the mean is 4.4

the variance is 6.64.

Use the definitions of mean and variance to find the values of x and y.

© Oxford University Press 2019 3


Mixed review

Answers

1 a x =3 b 1

2 a a = 2,  2,
h=  
k = −3 b x =2

3 -40

4 3, 11 and 13

5 −1.49

6 a 17 − 3n b −133

 2 50 
7  , 
 3 27 

8 a g −1 ( x )= 6 − 2 x b 14

9 89 months

10

48
11 a mode = 4, median = 5, mean
= = 6 A1A1A1
8

b All three statements hold for this data, so the book would classify this data as being skewed
to the right. A1R1

500
12 a V= 500 ⇒ π hx 2= 500 ⇒ h= M1A1AG
π x2

b=S 2π x 2 + 2π xh M1

500
( x ) 2π x 2 + 2π x
S=
π x2
M1

1000
( x ) 2π x 2 +
S=
x
AG

c Use of GDC to maximize S M1

x = 4.30 cm A1

h = 8.60 cm M1A1

© Oxford University Press 2019 4


Mixed review

13 a i The median is the middle term which is u4 = 4 + (4 − 1)3 =13 M1A1

∑x i 7
(2 × 4 + (7 −=
1)3)
ii=x =
i =1 2
13 M1A1
n 7

iii IQR= Q3 − Q1 = u6 − u2 = 4d = 12 M1A1

b i The median is the middle term which is u4 =


1 × 24 −1 =
8 M1A1

7

(
1 × 27 − 1 

)
∑x i  (2 − 1)  127
 
ii=x =
i =1
= M1A1
n 7 7

iii IQR= Q3 − Q1 = u6 − u2 = 25 − 2 = 30 M1A1

c The arithmetic progression A1

For this data set, a graph of n against un would yield a straight line (or equivalent) R1

14 a i 21 ii 19 iii 23 iv 20
v 36 A1A1A1A1A1

b Q1 and Q3 both increase by 5 R1

1.5 × IQR remains the same R1

Distance from new point to a new quartile will be the same as from the old point to the old
quartile, so it will remain as an outlier. R1

OR

Recognition that the box and whisker plot is just moved 5 units to the right so

outliers will be unaffected.

c i −32 ii −28 iii −36 iv 40


v 36 × (−2) =144 2
vi 12 A1A1A1A1A1A1

1+6+8+ x + y
15 = 4.4 ⇒ 15 + x + y = 22 ⇒ x + y = 7 M1A1
5

1 + 36 + 64 + x 2 + y 2
− 19.36 = 6.64 ⇒ 101 + x 2 + y 2 = 130 ⇒ x 2 + y 2 = 29 M1A1
5

Solving simultaneous equations, e.g.

x 2 + (7 − x ) =
2
29 M1

2 x 2 − 14 x + 49 = 29 ⇒ x 2 − 7 x + 10 = 0 A1

( x − 2 ) ( x − 5) =
0 M1

x = 2 or x = 5 , x =2 ⇒ y =5 and x =5 ⇒ y =2

So=x 2=
,y 5 R1A1A1

© Oxford University Press 2019 5


End of chapter test

7 Modelling relationships between two


data sets: statistics for bivariate data

Section A. A calculator is not allowed


1 Match the diagram with the correlation coefficient
A B C D

i −1 ii 0 iii 0.8 iv 1

2 Describe the correlation for each diagram using the words


Strong, moderate weak, positive, negative, linear, non linear, strong, moderate weak, no
correlation.
A B C

D E F

3 The scatter diagram shows the age of a


family of pet rabbits and their weights in kg.
a Describe the correlation.
b What should you do with the outlier?
Explain your reasons.

© Oxford University Press 2019 End of chapter test 1


End of chapter test

4 The production cost at Luigi’s pizzeria is modelled by

c (p
= ) 6 p + 50
where c is the production cost in dollars and p is the number of pizzas made.
a How much does it cost Luigi if the shop does not produce any pizzas?
b Interpret the meaning of the value of the gradient, 6.
5 A basketball coach recorded the average number of training hours per week for him team and
the average change in the number of points scored in a season.
Hours (x) 0 1 2 3 4 5
Points change (y) -5 0 1 10 15 18

a Show the data on a scatter plot


b Find the mean point and indicate this on your scatter plot by the label M.
c Draw a line of best fit through your data.
d Describe the correlation.
e What can you say about the number of hours trained and the number of points scored?

Section B. A calculator is allowed


6 Ten students recorded how many hours of exercise they have at the weekend and their weights
in kg.
Hours (x) 6 2 7 1 0 3 10 8 9 4
Weight (y) 80 60 70 50 90 80 70 100 55 60
Write down:
a the equation of the regression line
b the r value.
c Is it appropriate to estimate the weight of a student who does 5 hrs of exercise at the
weekend? Give reasons.
7 Here are the scores for 10 of the 12 students in my class for mathematics and science.
Mathematics (x) 90 66 84 75 90 88 69 95 73 81
Science (y) 73 60 79 67 78 67 55 82 59 80
a Write down the r value.
b What does this tell you about the correlation?
c Write down the equation of the y on x regression line.
Sara was absent for the science test but scored 80 in mathematics.
d Estimate her score using the equation of the regression line.
e Is this a valid estimate? Give reasons.
f Can the regression line be used to estimate the score of a student who scored 10% in
mathematics? Give reasons.
g Find the equation of the regression line of x on y.
h What can this equation be used for?

© Oxford University Press 2019 2


End of chapter test

8 I asked the first ten people that I saw this morning “How many pairs of shoes do you own”
Age (x) 15 24 42 13 56 16 14 20 6 12
Pairs of shoes (y) 2 7 5 4 6 8 4 8 2 6
a What type of sampling method did I use?
b Write down the r value.
c Explain what a positive value for the coefficient of correlation indicates.
d Write down the linear regression equation of y on x in the form y = ax + b
e Use your equation to determine the number of pairs of shoes that an 18 yr old would have.
f Can your answer in part e be considered reliable? Give a reason for your answer.
9 Vin thinks that price of a car is related to its age and collects the following data where the age
of the car is in years and the cost is in thousands of dollars.
Age of the car (x) 0 5 10 15 20 25 30 35
Cost ($1000) (y) 20 15 12 8 1 6 13 18
a Show this data on a scatter plot.
b Can you draw a single line of best fit to represent this data? Explain your answer.
c Find two piecewise linear functions that best represent the data.

© Oxford University Press 2019 3


End of chapter test

Answers
1 A1 B −1 C 0 D 0.8

2 A Strong negative linear B Weak positive linear


C No correlation D Moderate positive linear
E Non linear F Weak, negative linear
3 a Moderate positive correlation
b Discard the outlier. It appears to be an error.
4a $50
b Each pizza costs $6 to produce.
5a, b, c on the graph.

 0 + 1 + 2 + 3 + 4 + 5 −5 + 0 + 1 + 10 + 15 + 18 
b ( x,y ) =
 6
,
6


(2.5, 6.5)

d There is a strong, positive linear correlation.


e As the number of hours of practice increases the number of points scored increases.
6=
a y 0.364 x + 69.7

b 0.0795
c Is it not appropriate to estimate the weight of a student who does 5 hrs of exercise at the
weekend because the r value is weak.
7 a 0.812
b Strong, positive, linear correlation?
c y 0.801x + 5.02
=

d=y 0.801 ( 80 ) + 5.02


= 69%

e This is a valid estimate as it has a strong r value and we are using interpolation.
f The regression line cannot be used to estimate the score of a student who scored 10% in
mathematics as the score is outside of the given domain and extrapolation is unreliable.
g x 0.823y + 23.5
=

h This equation be used to estimate a mathematics score, given a science score.


8 a Random sampling
b 0.295
c As the age of a person increases, so does the number of shoes they have.

© Oxford University Press 2019 4


End of chapter test

d y 0.0421x + 4.28
=

e
= y 0.0421 (18 ) + =
4.28 4.62 or  5p
  airs of shoes

f No. The r value is weak.


9 a

b No. The data cannot be represented by one line but two lines would be appropriate.
c Using the calculator to find a function from
= = 20 gives y =
x 0  to x −0.9 x + 20.2

and then using the calculator to find a function from


= = 35 gives
x 20  to x
=y 1.16 x − 22.4

© Oxford University Press 2019 5


Mixed review

7 Modelling data between two data


sets: statistics for bivariate data
x 2 + 1,  and g ( x ) =
1 If f ( x ) = x + 2 , find

a f ( −3)

b (f  g) ( x )

c Solve (f  g) ( x ) = 2

2 The price of fried rice at 15 stalls in Hong Kong was


$3.00,$2.82,$2.75,$2.55,$2.98,$2.53,$2.40,$2.80,$2.50,$2.65,$2.48,$2.57,$2.30,
$2.79,$2.25.

Find

a the median

b the interquartile range.

3 The graph of the function f (x) = 2x − 8 intersects the x-axis at A and the y-axis at B.

Find the coordinates of

a A

b B

c Find the area of triangle OAB, where O is the origin.

4 The equation ax 2   3 = has exactly one solution. Find the value of a.
+ x   +1  0

5 Find the coefficient of x 4 in the expansion of (2 x − 3) .


12

6 The mass M kg of a substance at time t years is given by M = 8e−0.2t

a Write down the initial mass.

b How long does it take for the mass to be reduced to 6 kg?

7 Gardener George notices that the more birds in his garden, the more bees appear. He counts
the number of birds and bees in his garden over a seven-hour period and tabulates the data.

Birds 2 3 4 5 6 7 8

Bees 1 7 18 14 19 24 25

a Find a linear regression line in the form y = mx + b for his data.

b Find Pearson’s product-moment correlation coefficient (r) for this data.

c How many bees can he expect when there are 9 birds?

d Explain whether George could use this model to estimate the number of bees when there
are 1000 birds.

© Oxford University Press 2019 1


Mixed review

2x − 3
8 f (x) =
3x − 4

a Find the equations of the horizontal and vertical asymptotes.

b State the domain and range of f.

c Solve f ( x ) = g ( x ) where g ( x )= 3 − 2 x .

ln a
9 Solve 6 x = 3x +2 giving your answer in the form , where a and b are integers.
ln b

1
10 If f ( x ) =
1 + x2,

a Write down the equation of the horizontal asymptote of f.

b Find f ′ ( x )

c Find the coordinates of the maximum point of f.

d Find f ′′ ( x )

e Find the x-coordinate of the point on the curve of f where the gradient of the tangent

is a maximum

Exam-style questions

11 A swimming pool is 25 m long. The depth, d m, of water x m away from the edge at the
shallow end is given by

 0.5 + 0.1x 0 ≤ x ≤ 15
d( x ) = 
−1 + 0.2 x 15 ≤ x ≤ 25

a Write down the depth of water at the shallow end, when x = 0 . (1)

b Find the depth when x = 15. (1)

c Find the maximum depth of the pool. (1)

d Swimmers are only allowed to dive into the pool when the depth is greater than, or equal
to, 3 m. Calculate the minimum distance from the shallow end at which diving is allowed.
(2)

e Josie can only stand up in the pool if the depth is less than 1 m. Calculate the maximum
distance from the shallow end that Josie can be, if she is still able to stand up. (2)

f i Sketch a cross-section of the pool showing x against d ( x ) for 0 ≤ x ≤ 25 .

The width of the pool is 10 m.

ii Hence calculate the volume of water in the pool. Give your answer in m3 correct to 1
decimal place. (6)

© Oxford University Press 2019 2


Mixed review

12 Ten pairs of paired bivariate data, ( x, y ) , are given in the table below.

x 1 2 2.5 3 4 4.5 5 6 6.5 8

y 0.2 0.4 0.5 0.8 1.6 2.2 3.3 6.3 9 26

a For this data

i calculate the Pearson product-moment correlation coefficient

ii write down the equation of the y on x regression line. (4)

b For each data pair, list the value of x against the value of log y . Construct a table like
the one below to show your results.

log y

(2)

c For the paired bivariate data showing x against the value of log y ,

i calculate the Pearson product-moment correlation coefficient, giving your answer to 4


decimal places.

ii write down the equation of the log y on x regression line. (4)

d For x = 5.5 , find an estimate for y using

i the y on x regression line found in a ii

ii the log y on x regression line found in c ii. (4)

e Suggest, with a reason, which of the two estimates found in part d is the better one. (2)

13 Thirteen pairs of paired bivariate data are given in the table below

x 1 2 2 3 4 5 6 7 8 9 10 11 12

y 1 1 2 4 5 5.5 5 4.5 4 4 3.5 3 2.5

a Draw a scatter diagram to represent this data. (3)

b i Calculate the Pearson product-moment correlation coefficient for the above data.

ii Using two words, describe the linear correlation between x and y . (2)

c i For the six data pairs where x ≤ 5 , calculate the Pearson product-moment correlation
coefficient.

ii Using two words, describe the linear correlation between x and y for the six data pairs
from part i.

iii Find the equation of the y on x regression line for these six data pairs. (4)

d i For the seven data pairs where x ≥ 6 , calculate the Pearson product-moment correlation
coefficient.

© Oxford University Press 2019 3


Mixed review

ii Using two words, describe the linear correlation between x and y for the seven data
pairs from part i.

iii Find the equation of the y on x regression line for these seven data pairs. (4)

e Find the intersection point of the two lines found in parts c iii and d iii. (2)

f State two reasons why it is better to split the data into two halves in order to model it. (2)

14 a i Two values of paired bivariate data x and y are measured and recorded as
( x1, y1 ) and ( x2 , y2 ) . Explain why the Pearson product-moment correlation coefficient, r ,
for this data must be either +1 or –1.

ii Suppose ( x1, y1 ) = (1, 2) and ( x2 , y2 ) = (3, 6). Determine whether r = 1 or r = −1 .

iii Comment on whether it would be appropriate to draw a line of best fit for the data given
in part ii to help calculate the strength of correlation between x and y . (4)

b 100 pairs of paired bivariate data have perfect correlation (either positive or negative).
If ( x0 , y0 ) is one of these data pairs, where x0 is an outlier for the x -values, state (with a
brief geometrical explanation) whether or not y0 will be an outlier for the y -values. (3)

15 a For paired bivariate data ( x, y ) , which satisfies

y =−3x + 2, − 4 ≤ x ≤ 4 , x, y ∈ 

i describe geometrically the shape of the scatter diagram

ii sketch (very roughly) the scatter diagram

iii from the list below, select the description that would best describe the linear correlation:

Perfect positive, strong positive, weak positive, no correlation, weak negative, strong
negative, perfect negative. (4)

b Repeat all of part (a) but this time for paired data which satisfies

x 2 + y 2 ≤ 16 x, y ∈  . (4)

c Repeat all of part a but this time for paired data which satisfies

y − 2x ≤ 1 , − 4 ≤ x ≤ 4 , x, y ∈  . (5)

© Oxford University Press 2019 4


Mixed review

Answers

1 a 10 b x 2 + 4x + 5 c −3,  1

2 a $2.57 b $0.32

3 a ( 4, 0) b (0, −8) c 16 units2

4 2.25
5 51963120
6 a 8kg b 1.44yrs
a y 3.82 x − 3.68
7 = b 0.937 c 31 d No. 1000 is too far outside of the domain.

2 3 3 2
8 a=y = , x b x ∈ ,x ≠ y ∈ ,y ≠
,  c x = 1, 1
  .5
3 4 4 3

9=a 9,
=  2
b

2x 6x2 − 2
10 a y =0 b f ′(x) = − c (0, 1 ) d f ′′ ( x ) = e −0.577
(1 + x ) (1 + x )
2 3
2 2

11 a 0.5 m A1
b 2m A1
c 4m A1
d d ≥ 3 ⇒ −1 + 0.2 x ≥ 3 ⇒ x ≥ 20 , so minimum distance is 20 m M1A1

e d < 1 ⇒ 0.5 + 0.1x < 1 ⇒ x < 5 , so maximum distance is 5 m M1A1

f i

A2
ii Cross-sectional area is 2 rectangles plus two triangles M1

0.5 × 15 + 2 × 10 + 1
2
× 15 × 1.5 + 1
2
× 10 × 2 =48.75 M1A1

487.5 m3
Volume is 10 × 48.75 = A1

12 a i r = 0.821(3sf ) A2

ii y 2.97 x − 7.58(3sf )
= A1A1

b x 1 2 2.5 3 4 4.5 5 6 6.5 8


log -0.699 -0.398 -0.301 -0.0969 0.204 0.342 0.519 0.799 0.954 1.42
y
A2
c i r = 0.9996 (4dp) A2

ii =
log y 0.304 x − 1.02(3sf ) A1A1

d i y = 2.97 × 5.5 − 7.58= 8.76(3sf ) M1A1

ii log=
y 0.304 × 5.5 − 1.02
= 0.652 ⇒=
y 4.49(3sf ) M1A1

e The estimate using the log y on x regression line will be the better estimate A1

because there was almost perfect correlation between log y and x . R1

© Oxford University Press 2019 5


Mixed review

13 a

A3
b i r = 0.262(3sf ) A1

ii weak, positive A1
c i r = 0.952(3sf ) A1

ii strong, positive A1
iii
= y 1.3x − 0.6 A1A1

d i r = −0.988(3sf ) A1

ii strong, negative A1
iii y =
−0.393x + 7.32(3sf ) A1A1

e intersect at ( 4.7,5.5) (1dp) A1A1

f There was only minor linear correlation for the whole data. R1
The scatter diagram suggests that a better model is two lines of best fit, each showing
strong linear correlation by their PMCC values. R1
14 a i As the two points must lie on a straight line, there will be perfect correlation. R1R1
Perfect correlation in Pearson’s PMCC is denoted by r = ±1 . AG
ii Since the straight line has positive gradient, r = 1 . R1
iii Since the 2 points must lie on a straight line, it is impossible to determine the strength of
correlation between x and y using only these values. Hence, a line of best fit would not
be appropriate. R1
b Perfect correlation ⇒ All the points lie on a straight line. R1

Since x0 is an outlier, ( x0 , y0 ) must be sufficiently separated from the majority of the other
points and thus y0 is also an outlier. R1A1

15 a i all points lie on a straight line A1


ii

A2
iii Perfect negative A1
© Oxford University Press 2019 6
Mixed review

b i Points lie on or inside a circle of radius 4. A1


ii

A2
iii no correlation A1

c i y − 2 x ≤ 1 ⇒ −1 ≤ y − 2 x ≤ 1 ⇒ y − 2 x =−1, 0 or 1 R1

y 2x − 1 , y = 2x , =
So points lie on 3 lines: = y 2x + 1 A1

ii

A2
iii strong positive A1

© Oxford University Press 2019 7


End of chapter test

8 Quantifying randomness:
probability
A calculator is allowed

1
1 If the probability of rain on any day this week is , find the probability that,
3

a It will not rain today.

b It will rain on Monday and Tuesday

c It will rain on Friday and Saturday, but not Sunday.

d What is the expected number of dry days in a six-day period?

2 In a class of 20 students, each student chooses a favorite colour from red, blue and purple.

The results are shown in this table.

Red Blue Purple

Female 5 3 3

Male 4 2 3

Find the probability that the student was:

a a male who chose red

b a male or a student who chose red, but not both.

c Given that a student is female, calculate the probability that she did not choose red.

d Two students are chosen at random Find the probability that neither chose red.

3 If = P ( B ) 0.5 and  P ( A=
P ( A ) 0.6, = ∩ B ) 0.2,

a Show this data on a Venn diagram like the one shown.

b Find the values of a, b and c.

c Find P ( A′ ∩ B ) .

© Oxford University Press 2019 End of chapter test 1


End of chapter test

4 In a class of 16 students, 12 speak Spanish, 8 speak French and one speaks neither Spanish
nor French. The class is shown on the Venn diagram where p, q, r and s represent the number
of students.

a Write down the value of s.

b Find the value of q.

c Write down the values of p and r.

A student is selected at random from the class.

d Given that the student speaks French, write down the probability that the student speaks
Spanish.

e Hence, show that speaking Spanish and speaking French are not independent events.

One student is selected at random from the class and asked to stand outside, then another is
selected.

f Find the probability of choosing two students where the first student speaks only French and
the second speaks only Spanish.

5 There are 120 students in an IB group. 40 take physics (P), 35 take chemistry (C) and 30 take
biology (B). This is shown in the Venn diagram.

Find the probability that a student chosen at random will

a Take all three of these sciences.

b Study exactly two of these sciences.

c Study only chemistry.

d Not study physics, chemistry or biology.

© Oxford University Press 2019 2


End of chapter test

6 Roz recorded the number of siblings that students in his class had in school.

a Copy the table and complete the relative frequency column.

Siblings Frequency Relative frequency

0 8

1 4

2 3

3 3

4 2

What is the probability that a student:

b had 3 siblings in school

c had at least 2 siblings in school.

7
7 The probability that Serene wins her first tennis match is .
8

3
If she wins the first match, the probability that she wins the second is .
4

3
If she loses the first match, the probability that she loses the second match is .
5

a Show the possible results for Serene’s first two matches on a tree diagram.

Calculate the probability that Serene will

b Win her first two matches.

c Not lose her first two matches.

d Win only one of her first two matches.

e Given that Serene loses her second match, what is the probability that she won her first?

8 The probability of two events is given


= as P ( A ) 0.2 and 
= P ( B ) 0.5.

Find P ( A ∪ B ) when

a A and B are Independent.

b A and B are mutually exclusive.

c Find P( A | B) when P ( A ∪ B ) =
0.6

1 1 5
( A)
9 When P=
6
(B) 3
,  and 
P= P ( A ∪=
B)
12
,

a Find P ( A ∩ B )

The sets A and B can be represented in a Venn diagram as shown.

b Find the values of p, q, r, s.

c Find P ( A′|B′ ) .

© Oxford University Press 2019 3


End of chapter test

Answers

1 2 1 1 1 1 1 2 2 2
1 a 1− = b × = c × × = d ×6 =4
3 3 3 3 9 3 3 3 27 3

4 1
2 a =
20 5

5 + 3 + 2 10 1
b = =
20 20 2

Number of females who did not choose red 6
c =
Number of females 11

11 10 11
d × =
20 19 38

3 a

b a = 0.6 − 0.2 = 0.4

b = 0.5 − 0.2 = 0.3

1 ( 0.4 + 0.3 + 0.2 ) =


c =− 0.1

c P ( A′ ∩ B ) =
0.3

4 a s =1

b q = 21 − 16 = 5

p 7, 3
c = = r

5
d
8

e For independent events

P ( S ) × P ( F ) =P ( S ∩ F )

12 8 5
× ≠
16 16 16

3 7 7
f × =
16 15 80

5 1
5 a =
120 24

8+4+7 19
b =
120 120

35 − ( 8 + 5 + 7 ) 15 1
c = =
120 120 8

© Oxford University Press 2019 4


End of chapter test

d Only physics = 40 − ( 8 + 5 + 4 ) = 23

Only biology = 30 − (7 + 5 + 4 ) = 14

) 44
Not studying science= 120 − (15 + 23 + 14 + 8 + 5 + 4 + 7=

44 11
P (no science
= ) =
120 30

6 a Siblings Frequency Relative frequency

0 8 0.4

1 4 0.2

2 3 0.15

3 3 0.15

4 2 0.1

b 0.15

c 0.15 + 0.15 + 0.1 =


0.4

7 a

7 3 21
b × =
8 4 32

 1 3  37
c 1− ×  =
 8 5  40

7 1 1 2 7 1 43
d  × + ×  = + =
 8 4   8 5  32 20 160

7 1
P (W ∩ L ) ×
e P (W
= | L) = 8
= 4 35
P (L) 47 47
160

8 a P ( A ∪ B=
) P ( A) + P ( B ) − P ( A ∩ B )
) P ( A) + P ( B ) − P ( A) (P ( B )
P ( A ∪ B=

P ( A ∪ B ) = 0.2 + 0.5 − 0.1 = 0.4

© Oxford University Press 2019 5


End of chapter test

b P ( A ∪ B ) = P ( A ) + P ( B ) = 0.2 + 0.5 = 0.7

c P ( A ∪ B=
) P ( A) + P ( B ) − P ( A ∩ B ) 
P ( A ∩ B=
) P ( A) + P ( B ) − P ( A ∪ B )
P ( A ∩ B ) = 0.2 + 0.5 − 0.6 = 0.1

P ( A ∩ B ) 0.1 1
P ( A=
|B ) = =
P (B) 0.5 5

9 a P ( A ∪ B=
) P ( A) + P ( B ) − P ( A ∩ B ) 
1 1 5 1
P ( A ∩ B ) = P ( A) + P ( B ) − P ( A ∪ B ) = + − =
6 3 12 12

1
b q=
12

1 1 1
p= − =
6 12 12

1 1 1
r = − =
3 12 4

 1 1 1 7
s=1− + + =
 12 12 4  12

7
P ( A′ ∩ B′ ) 12 7
c P ( A′|B′ ) = = =
P ( B′ ) 8 8
12

© Oxford University Press 2019 6


Mixed review

8 Quantifying randomness:
probability
1 a I pick 2 cards from a well-shuffled pack. Work out the probability that the 2nd card is an ace
given that the first card is an ace.

b In a box there are 12 white cards and 18 black cards. I pick up two cards randomly. Find the
probability that I pick a black card as a second card.

2 a I pick two cards from a well-shuffled pack at random. Find the probability they are both
aces.

b There are 6 boys and 8 girls. I pick randomly three people. What is the probability that all
three people are girls?

3 In a town, 15% of the houses do not have a smart TV, 40% of the houses do not have a video
game console and 10% of the houses have neither a smart TV nor a video game console. I
choose one house randomly. Find the probability that it has smart TV and a video game
console.

P( A) 7
4 If = , for an event A , find P( A) and P( A′) .
P( A′) 8

5 We consider Ω
= {w ∈  / 10 ≤ w ≤ 20} , A the subset of Ω which contains the multiples of 3
and B the subset of Ω which contains the multiples of 4. I choose one number from Ω
randomly. Work out the following probabilities:

a P( A) b P(B′) c P( A ∩ B)

d P( A ∪ B) e P( A | B) f P(B | A)

Exam-style questions

6 Eight cards have the following numbers written on them:

22
0, 1, π, 2, 1.5, −3, , 1000.
7

A card is chosen at random. Find the probability that the number written on it is

a a natural number

b an integer

c a rational number

d an irrational number

e greater than 3

f a real number

g smaller than zero. (7)

© Oxford University Press 2019 1


Mixed review

7 a If one of the following quadratics is chosen at random, find the probability that it is
concave upwards. Justify your answer.

y = x2 , y = 2 x 2 − 3x + 4 , −3x 2 + 4 , =
y = y 4(x − 1)2 , y = π x 2 + 2x + 4 (2)

b If the derivative of the following functions is evaluated at x = 0 , find the probability that
the answer is zero. You must show all your working to gain full marks.

y = x2 , y = sin x , y = cos x , y = ex , y 4(x − 1)2


= (4)

8 Rachel, a librarian, has 10 books on her desk. These 10 books consist of 6 thrillers, 3
autobiographies and 1 science fiction book. She chooses 5 books at random to place back on
the shelves. Calculate the probability that

a she chooses all thrillers

b her choice includes the science fiction book.

c her choice does not include an autobiography. (6)

9 Martin and Rob are fighting a duel to the death with pistols (over the affections of a lady).

They fire at each other at the same time. The probability that Martin’s bullet hits and kills Rob
2 3
is always . The probability that Rob’s bullet hits and kills Martin is always . All probabilities
3 5
are independent. If at the end of the first round they are both alive they continue the process.
They continue in this way until at least one of them is dead.

a Find the probability that they are both alive at the end of the first round. (2)

b Find the probability that they are both dead at the end of the first round. (2)

c Find the probability that Martin is alive and Rob is dead at the end of the first round. (2)

d Find the probability that Martin is alive and Rob is dead after the second round. (3)

e Find the probability that Martin is alive and Rob is dead at the end of the whole duel. (6)

10 Consider the graph of the quadratic function defined by f ( x=


) x 2 − 3x and the point A ( 4, −1) .
Let P be a point on the graph of f with x-coordinate x.

a State the coordinates of P in terms of x . (1)

b Find an expression for the distance between

i the origin O and the point P

ii the point A and the point P. (4)


c Determine the coordinates of P such that the perimeter of triangle OAP is minimum, and
find the length of the perimeter in this case. (6)
© Oxford University Press 2019 2
Mixed review

Answers
1 a We denote as A the event that the 1st pick is an ace and as B the event that the 2nd pick is
an ace.
Since these two events are independent we obtain that P( A ∩ B)= P( A) × P(B)
4 3
Obviously, we have that
= P( A) = , P(B)
52 51
4 3 1
P( A ∩ B) = × =
52 51 221
We are required to find P(B | A) .
P(B ∩ A)
We know that P(B | A) = .
P( A)
1
3
Hence, we obtain that P(B | =
A) =
221
.
4
52
51

b We denote as w1, b1, b2 the events of picking white as a first card, black as a first card and
black as a second card.
12 18 18
Then obviously,
= P(w1 ) = , P(b2 ) = , P(b1 )
30 29 30
By denoting as X the required event, we get that:
12 18 18 17 3
P( X ) = × + × =
30 29 30 29 5
2 a We denote by A the event of picking one ace and by X the required event of picking two
aces.
4
Obviously, P( A) = .
52
The events of picking one ace first and then one other ace at the second time are
4 3 1
independent. Hence, we can say that P( X ) = × = .
52 51 221
b We denote by A the event of picking the first person to be a girl and by X the required
event of picking three girls
8
Obviously, P( A) = .
14
The events of picking three people to be all girls are independent. Hence, we can say that
8 7 6 2
P( X ) = × × = .
14 13 12 13
3 We denote T , V the event of having a smart TV and the event of having a video game console
respectively.
We are required to find P(T ∩ V ) .
15 85
Hence, P(T ) =
1 − 15% =
1− =
100 100
40 60
P(V ) =
1 − 40% =
1− =
100 100
10
P(T ′ ∩ V ′) =
100
We know from theory that P(T ∪ V ) + P(T ∩ V )= P(T ) + P(V ) .
We also know that P(T ′ ∩ V ′) =P((T ∪ V )′ ) =1 − P(T ∪ V ) .
10 90
Hence, P(T ∪ V ) =1 − = .
100 100
85 60 90 55
Therefore, P(T ∩ V )= + − = .
100 100 100 100

© Oxford University Press 2019 3


Mixed review

4 We know that P( A) + P( A′) =


1.
So, P( A′)= 1 − P( A) .
P( A) 7
In the given equation, we obtain: =
1 − P( A′) 8
8P(=
A) 7(1 − P( A))
15P( A) = 7
7
P( A) =
15
7 8
P( A′) =1− =
15 15
5 a Let’s analyse the subsets A and B .
Hence, A = {12,15,18} and B = {12,16,20} .
Note that Ω ={10,11,12,13,14,15,16,17,18,19,20} .
3
Obviously then, P( A) =
11
b We know that P(B′)= 1 − P(B) .
3
But, similarly as above, we have that P(B) = .
11
8
Therefore, P(B′) = .
11
c Intersection of two sets means the common elements.
1
Hence, P( A ∩ B) = , as A, B just have “12” in common.
11
d We know that P( A ∪ B) + P( A ∩ B)= P( A) + P(B) .
3 3 1 5
Therefore, P( A ∪ B) = + − = .
11 11 11 11
P( A ∩ B)
e We also know that P( A | B) = .
P(B)
1
1
Therefore, P( A | B=
) =
11
.
3
11
3

P(B ∩ A)
f Similarly, we get that P(B | A) = .
P( A)
1
1
Therefore, P(B | A=
) =
11
3
11
3

3
6 a A1
8
4 1
b = A1
8 2
6 3
c = A1
8 4
2 1
d = A1
8 4
3
e A1
8
f 1 A1
1
g A1
8

© Oxford University Press 2019 4


Mixed review

7 a ax 2 + bx + c is concave up if a > 0 R1
4
So probability is A1
5
b Derivatives are
2 x, cos x, − sin x, e x , 8 ( x − 1) M1A1

Derivatives evaluated at x = 0 are 0, 1, 0, 1 and 8 A1


2
So probability is . A1
5
C56 6 1
8 a = = M1A1
C510 252 42

C49 126 1
b = = M1A1
C510 252 2

C57 21 1
c = = M1A1
C510 252 12

1 2 2
9 a × = M1A1
3 5 15
2 3 2
b × = M1A1
3 5 5
2 2 4
c × = M1A1
2 5 15
d must both miss 1st round; then in 2nd round Martin hits and Rob misses R1
2 2 2 8
× × = M1A1
15 3 5 225
e Martin could win in the 1st round, or the 2nd or the 3rd... R1
2
4 2 4  2  4
+ × +  × + ... M1A1
15 15 15  15  15

4 2
Infinite GP with a = and r = R1
15 15
4
15 4
=S∞ = M1A1
2 13
1−
15
10 a (
P x, x 2 − 3x ) A1

( )
2
b i x 2 + x 2 − 3x M1A1

( )
2
( x − 4)
2
ii + x 2 − 3x + 1 M1A1

c perimeter OAP = OP + AP + OA

( ) ( )
2 2
( x − 4)
2
= x 2 + x 2 − 3x + + x 2 − 3x + 1 + 42 + 1 M1A1

Use GDC to find minimum M1


x = 1.5 A1

(1.5) − 3 (1.5) =−2.25


2
⇒y = A1

Perimeter = 6.35 A1

© Oxford University Press 2019 5


End of chapter test

9 Representing equivalent quantities:


exponentials and logarithms
Section A. A calculator is not allowed
x
x x 1 x
1 Match the curves f, g, h and p with the functions
= y e= y
, 1= y
,  = y
, 2
2

p
h

2 Copy the graph below with the function f ( x ) = 3x and add the curves

g (x) =
1.5x , h ( x ) =
3x + 2,  p ( x ) =+
3− x 2 and  q ( x ) =
3x +2.

3 Evaluate the log of each number

1
a 10000 b c 0.001 d 102.5
100

e 10 f 10 3 10 g 0

4 Solve for x

2 1
a 3x +2 x
= 27 b 75 − x = c 53 x = 25x −1 d 22 −3 x = 82 x −3
49

5=   and  logx 5 q   ,  find expressions in terms of p and q for


If logx 2 p=

a logx 2.5  b logx 40  c log2 5 

x
If a ln
6= = x, b ln y , a=
nd  c ln z,  write ln 2
 in terms of a, b and c.
y z

© Oxford University Press 2019 End of chapter test 1


End of chapter test

Section B. A calculator is allowed

7 Solve the following equations to find the value of x to 3 significant figures

a 4x = 9 b e3 x − 51− x =
0

ln a
c Solve 6 x = 3x +1 giving your answer in the form where a, b are integers
ln b

1 − log2 ( x − 6 )
8 Solve log2 x =

9 Find the equation of the tangent to the curve f ( x ) = e2 x  at the point where x = 1 .

(
f ( x ) xln 4 − x 2
10 Given = )
a Find f ′ ( x )

b Sketch the curve within the domain −2 ≤ x ≤ 2 .

c Hence write down the solutions of f ′ ( x ) = 0 .

© Oxford University Press 2019 2


End of chapter test

Answers
x
1
1 f is y = 1x , g is y = e x , h is y = 2x and p is y = .
2
2

1
3 a log1000
= log10
= 4
4log10
= 4 b log = log10−2 = −2
100

c log0.001 = log10−3 = −3 d log102.5 = 2.5


4
4
e log 10 log10
= = 0.5
0.5 f log10
= 3
10 log10
= 3
g log0 = 1
3

4 a 3x
2
+2 x
= 27 1 c 53 x = 25x −1 d 22 −3 x = 82 x −3
b 75 − x =
49
( ) ( )
2 x −1 2 x −3
3x +2 x
= 33 53 x = 52
  22 −3 x = 23
5− x −2
7 =7
x 2 + 2x =
3  3=
x 2x − 2 2 − 3x = 6 x − 9
5−x =−2
x 2 + 2x − 3 =0  2
x = − 9 x = 11
x =7
( x + 3) ( x − 1) =
0
x =
11
9
x = −3,1

5 a logx 2.5 = q − p 
logx 5 −  logx 2 =

b logx 40 =
 logx 8 log
+ x 5 =
 logx 23  log
+ x 5 =
 
3 logx 2 log
+ x 5 =
 
3p + q

logx 5 q
c log2 5
= =
logx 2 p

x
6 ln 2
=lnx − lny 2 − ln z
y z

x 1 1
ln =lnx − 2lny − lnz =a − 2b − c
y 2
z 2 2

7 a log 4x = log9 b e3 x − 51− x =


0 c ln 6 x = ln3x +1
 log
x 4 = log9 e3 x = 51− x x ln6
= (x + 1)ln3
x=ln6 x ln3 + ln3
log9 lne3 x = ln51− x
= 1.58
x = x ln6 − x ln3 = ln3
log 4 3xlne
= (1 − x ) ln5 x(ln6 − ln3) = ln3
3=
x ln5 − xln5 ln3
x =
3x + xln5 =
ln5 (ln6 − ln3)
ln3
x (3 + ln5) =
ln5 x =
ln2
ln5
=x = 0.349
3 + ln5

© Oxford University Press 2019 3


End of chapter test

8 log2 x + log2 ( x − 6 ) =
log2 2

(
log2 x 2 − 6 x =
2 )
x2 − 6x =
2

x2 − 6x − 2 =
0
x = −0.316, 6.32
x = −0.316 is not a valid answer as it leaves the log of a negative.
Therefore   6.32
x =
9 f (1) = e2  or  7.39

It is better to keep your answer as e2 for accuracy and simplicity.


f ′ ( x ) = 2e2 x

f ′ (1) = 2e2  
or 1
 4.8

y − y1= m ( x − x1 )
2
y − e= 2e2 ( x − 1)

=y 2e2 x − e2

f ( x ) xln 4 − x 2
10 Given = ( )
a f ′ (=
x ) uv ′ + vu′

Differentiate ln 4 − x 2 ( ) using the chain rule

u= x2 y =
4 −     lnu

du dy 1 1
−2 x,  
= = =
dx du u 4 − x 2
dy dy du
= ×
dx du dx
dy −2 x
=
dx 4 − x2

  v ln 4 − x 2
u x=
= ( )
−2 x
u′ 1 
= =v′
4 − x2
 −2 x 
f ′ ( x ) = x ×

 + ln 4 − x
4 − x2 
2
( )
 −2 x 2 
f ′ (=
x) 
4 − x 2 
+ ln 4 − x 2 ( )
 

 −2 x 2 
b Graph of y =  2 
+ ln 4 − x 2 ( )
4 − x 
c x = −1.15,1
 .15

© Oxford University Press 2019 4


Mixed review

9 Representing equivalent quantities:


exponentials and logarithms

1 The graph of y= 3 − 
x 2 intersects the graph of y   = 2 
x at the point (1, 2) and at one other point
A. Find the coordinates of A.

2 A sum of $4000 is invested at a compound interest rate of 3.5% per annum. To the nearest

dollar, find the total value of the investment at the end of five years.

3 The graph of y = a sin bx, for constants a and b, is shown below.

Find the values of a and b.

4 A data set has a mean of 10 and a standard deviation of 3.

Each value in the data set has 6 added to it. Write down the values of

a the new mean

b the new standard deviation.

Each value in the original data set is then multiplied by 5. Find

c the new mean

d the new variance.

5 Solve e x= 5 − 2 x .

6 A cinema has 40 rows of seats. There are 25 seats in the first row, 30 seats in the second
row, and each successive row of seats has two more seats in it than the previous row. Find
the number of seats

a in the 20th row

b the cinema has in total.

© Oxford University Press 2019 1


Mixed review

7 A cricket team has 11 players. The number of runs scored, per player, over three games is
summarized by this box plot.

a Find the median number of runs scored per player.

b Find the percentage of players that scored between 20 and 60 runs.

c State the interquartile range for this data.

d Explain why 160 is an outlier.

8 The amount of a drug, A mg, which remains in a patient’s bloodstream at time t hours after the
drug was injected is modelled by ( t )  
=5
  00e− kt .

a Determine the amount of drug that was injected into the patient.

It is found that 250 mg of the drug remains after 5 hours have passed.

b Determine the value of k.

c Find the value of t at which 50 mg of the drug remains in the patient’s bloodstream. Give
your answer correct to 3 s.f.

9 Of the final year students at a school, 36 students study Biology and 18 students study
Spanish. There are 53 students in the final year, and 10 do not study either Biology or Spanish.

a Determine how many students study both Biology and Spanish.

If a student is chosen at random, find the probability that they:

b study Biology but do not study Spanish

c study Spanish, given that they study Biology.

10 A stone is dropped vertically from the top of a cliff. Its velocity, v ms−1, is modelled by

V (=
t ) 50 − 50e−0.2t , where t is the number of seconds since the stone was dropped.

a Find the velocity of the stone when

i t   0
=

ii t 
=1
  0.

b Find an expression for the acceleration of the stone, a ms−2 , as a function of t.

c State the value of a when t   0.


=

d Determine the value which v approaches as t becomes very large.

e Determine the value which a approaches as t becomes very large.

f Explain what your answers to parts d and e tell you about the motion of the stone.

© Oxford University Press 2019 2


Mixed review

Exam-style questions

11 A consumer report suggests that the probability of a six-year-old car having faulty lights is
0.32, and that the probability of a six-year-old car having faulty brakes is 0.21.

Find the probability that

a A six-year-old car chosen at random has faulty lights and faulty brakes. (2)

b The car has neither faulty lights, nor faulty brakes. (2)

c The car has either faulty lights, or faulty brakes, but not both. (2)

12 Katharina and Carolina go to a swimming pool. They both swim the first length of the pool in 2
minutes 6 seconds.

The time that Katharina takes to swim each subsequent length is 5 seconds more than the time
she took to swim the previous length.

The time Carolina takes to swim a length is 1.04 multiplied by the time that she took to swim
the previous length.

a i State the time Katharina takes to swim the third length.

ii Show that Carolina takes 2 min 16 seconds, to the nearest second, to swim the third
length. (5)

Katharina and Carolina both swim a total of 10 lengths of the pool. They start at exactly at the
same time.

b Show that Katharina completes the 10 lengths before Carolina, clearly showing your
calculations and reasoning. (6)

c Hence, state the time that passes between Katharina finishing and Carolina finishing. Give
your answer correct to the nearest second. (2)

13 Consider the function defined by f ( x


= ) a ln ( x − b ) + c , where a, b and c are parameters to be
determined.

The graph of f has a vertical asymptote x = 2 , intercepts the x − axis at x = 3 , and the
tangent to its graph at x = 1 has gradient 2.

a Determine the value of each parameter a, b and c . (6)

b Hence sketch the graph of f . (2)

14 The velocity v , in metres per second, of a particle after t seconds is given by

v ( t )= 2 ( 0.2t + 0.3) − 1 , 0 ≤ t ≤ 4 .
t

a Sketch the graph of v = v ( t ) , showing clearly the axes intercepts. (3)

b State the value(s) of t when the particle is at rest. (2)

c Find the value of t when the acceleration of the particle is 0. (3)

15 Consider the four numbers a, b, c, d ∈  with a ≤ b ≤ c ≤ d .

The mean of these four numbers is 8.

The mode and the median are both equal to 6.

The range is 10.

Find the values of a, b, c and d , clearly showing your reasoning. (8)

© Oxford University Press 2019 3


Mixed review

Answers

1 ( −3, −6 )
2 $4751

3=a 4,
=  3
b

4 a 16 b 3 c 50 d 225

5 1.06

6 a 66 b 490

7 a 48 b 50% c 40

d An outlier is greater than 1.5 IQR above Q3.

Q3 + 1.5IQR = 60 + 1.5 ( 60 − 20 ) = 60 + 60 = 120

160>120

8 a 500 b 0.139 c 16.6

25 11
9 a 11 b c
53 36

10 a i 0 ii 43.2

b 10e−0.2t c 10 d 50 e 0

f The terminal velocity of the stone is 50ms −1

11 a P ( L ∩ B )= 0.32 × 0.21= 0.0672 M1A1

b P ( L´∩B´) = (1 − 0.32 ) × (1 − 0.21) = 0.5372 M1A1

c P ( L ∪ B ) − P ( L ∩ B ) =1 − P ( L´∩B´)  − P ( L ∩ B ) M1
=1 − 0.5372 − 0.0672

= 0.3956 A1

12 a i 2 min 16 sec A1

ii 2 minutes 6 seconds = 126 seconds A1

1.042 136.28...sec
126 ×= = 2 min16 sec M1A1A1

b Katharina´s times form an AP. R1

K’s total time = (126 × 2 + 9 ×=


5) × 5 1485
= sec 24 min 45 sec M1A1

Carolina´s times form a GP. R1

1.0410 − 1
C´s total time= 126 × = 1512.77 sec= 25min13 sec M1A1
1.04 − 1

c 1513 – 1485 = 28 seconds M1A1

© Oxford University Press 2019 4


Mixed review

13 a vertical asymptote x = 2 ⇒ b =
2 R1

f (3) = 0 ⇒ a ln (3 − 2 ) + c = 0 ⇒ c = 0 M1A1

a
f ′(x) = M1
x −2

a
f ′ (1) = 2 ⇒ = 2 ⇒ a = −2 M1A1
1−2

A1 Shape; A1 Domain and zero

14 a

Shape A1, domain A1, Intercepts A1

b b v ( t ) = 0 ⇒ t = 1, t = 2.09 seconds A1A1

c c a ( t ) = 0 ⇒ t = 1.52 seconds M1A1

15 Mode & median = 6 ⇒ b= c= 6 R1A1A1

Range = d − a = 10 ⇒ d = a + 10 M1A1

a + 6 + 6 + ( a + 10 )
Mean = 8 ⇒ =8⇒ a =5 M1A1
4

d = a + 10 ⇒ d = 15 A1

© Oxford University Press 2019 5


End of chapter test

10 From approximation to
generalization: integration
Section A. A calculator is not allowed
1 Integrate the following

3 4 x 5 − x 2 − 10 x
a 4x 7 − 9x 2 − 4 b x2 c
2x

4
( )
5
d 3x 2 x 3 + 3 e x x2 + 6 f + 5e x
x

3 1
2 (3x − 7 )
6
g h i
(2x + 1)
2
5x − 1

2 Evaluate
1 5
 3
a ∫(4x − 4x + 1)dx b ∫  4x + x  dx
2

0 1

3 This diagram shows part of the graph of y = x 2 . Find the area of region R.

4 4 4
4 When
= ∫f ( x ) dx 9 and 
0
= ∫g ( x ) 4,  evaluate
0
∫ (3f ( x ) + g ( x ) ) dx
0

5 Given that f ′ ( x ) =
10 x + 6 x  and  f (1) =
4
7, find  f ( x )

1
6 The gradient of f ( x ) is e −3 x +  , x < 1 and f ( x ) has a y intercept of 4.
1− x

Find f ( x ) .

© Oxford University Press 2019 End of chapter test 1


End of chapter test

Section B. A calculator is allowed

f ( x ) 6 x 2 (1 − x ) .
7 This is the graph pf =

a Find the value of the x-intercepts.


b Write down an integral which represents the shaded area.
c Write down the area of the shaded region.

1
8 The shaded region under the curve of y = has an area of 2 square units, find the value of a.
x

9 The diagram shows f ( x )= x 2  and  g ( x )= 6 − x

a Find the x values of their intersection points.


b Write down an integral for the area of the shaded region.
c Write down the area of the shaded region.

10 a Sketch the graph of f ( x ) = x 3 − x 2 − 6 x with −3 ≤ x ≤ 4 and −10 ≤ y ≤ 5

Write down
b the values of the x-intercepts.

c An integral to represent the area between f ( x ) and the x axis.

d The area between the curve and the x axis.

11 The acceleration of a particle is given modelled by 12t 2 − 6. 

a Find the velocity after 10 seconds if the initial velocity was 2 ms−1.

b Find the displacement after 3 seconds if the displacement after 1 second was -3 m.

© Oxford University Press 2019 2


End of chapter test

Answers

1 8 3 53 2 5 x2
1 a x − 3x 3 − 4 x + c b x c x − − 5x + c
2 5 5 4
3
2 3 1
( ) ( )
5
d x +3 2
+c e x2 + 6 +c f 4ln x + 5 e x + c
3 10

2 3 1
(3x − 7 ) + c
7
g h ln 5x − 1 + c i −
6 (2 x + 1)
3
21 5

1
4 1 4 3  4  1
− 4 x + 1)dx=  x 3 − 2 x 2 + x  =  (1) − 2 (1) + (1)  −  ( 0 ) − 2 ( 0 ) + ( 0 ) =
2 3 2
2 a ∫(4x
2
  
0  3  0  3   3  3
2
 3 2
b ∫  4x + x  dx = 2x
2
+ 3lnx  = ( 8 + 3ln2 ) − (2 + 3ln1) = 6 + 3ln2
1 1
2
1  2  1   1  8 1 7
3 Area R = ∫x 2dx =  x 3  =  23  −  13  = − =  square  units
1 3  1  3   3  3 3 3
4
4 ∫ (3f ( x ) + g ( x ) ) dx=
0
3 (9) + 4
= 31

5 ∫ f ′ ( x ) dx = f ( x )
∫ (10x )
4
+ 6 x dx = 2 x 5 + 3x 2 + c

f (1) = 2 (1) + 3 (1) + c = 5 + c = 7, 2


5 2
c =

f ( x ) = 2 x 5 + 3x 2 + 2

 1  1
6 ∫  e
−3 x
+  dx =− e −3 x − ln (1 − x ) + c
1− x 3

1 −2 0
At  ( 0, 4 )  4 =
− e ( ) − ln 1 − ( 0 ) + c
3
( )
1
4 =− − ln1 + c
3

1
c =4 
3

1 1
f (x) =
=− e−3 x − ln (1 − x ) + 4
3 3

7 a f ( x=
) 6 x 2 (1 − x=) 0 when =
x 0,1

The x intercepts are 0 and 1


1
b x ) dx
∫6 x (1 −  
2

c 0.5 square units


a
1 a
8 ∫ x dx = ln x  2 = ln a − ln2 = 2
2

ln a= 2 + ln2
2 + ln 2
=a e= 14.8

© Oxford University Press 2019 3


End of chapter test

9 a x 2= 6 − x

x2 + x − 6 =
0

( x + 3) ( x − 2 ) =
0

x = −3,2
2
b ∫ (6 − 
x − x ) dx 2

−3

125
c ≈ 20.8 square units
6
10 a

b −2, 0,1
 
0 0
c ∫ (x )
dx + ∫ x 3 − x 2 − 6 x   ( )
3
− x2 − 6x   dx
3 −2

253
d ≈ 21.1 square units
12

v (t ) = ∫ (12t )
2
11 a − 6 dt = 4t 3 − 6t + c

An initial velocity of 2 ms−1 gives ( 0,2 ) which makes

2= 4 ( 0 ) − 6 ( 0 ) + c, 2
3
c=

v ( t ) = 4t 3 − 6t + 2

At 10 secs, t = 10

v (10 ) 4 (3) − 6 (3) +=


3
= 2 92 ms−1

s (t ) = ∫ ( 4t )
3
b − 6t + 2 dt = t 4 − 3t 2 + 2t + c

A displacement after 1 second of −3m gives (1, −3) which makes

−3 =(1) − 3 (1) + 2 (1) + c,  c =−3


4 2

s ( t ) = t 4 − 3t 2 + 2t − 3

After 3 secs, t = 3

s ( 3=
) (3) − 3 (3) + 2 (3) − 3= 57 m
4 2

© Oxford University Press 2019 4


Mixed review

10 From approximation to
generalization: integration
  2 x  5 dx , using the fact that it is of the form   ax  b  dx .
2 n
1 a Find

  2 x  5 dx by first expanding 2x  5 .


2 2
b Find

c Use the binomial theorem to expand your answer to part a.

d Explain why the answers to parts a and b are both correct.

2 A particle moves along a horizontal line with velocity v t   t 3  2t , for t  0 , where v is

in centimetres per second and t is in seconds.

a Find the values of t for which the particle is moving left.

b Find the acceleration of the particle when t  3 .

c When t  2 , the displacement, s, of the particle is 5 centimetres.

Find an expression for s in terms of t.

3 Part of the graph of f  x   x2 ln  x  4 is shown in the following diagram.

The x-intercepts are at x  0 and x  a .

a Find the value of a.

The graph has a maximum when x  b .

b Use a GDC to find the value of b.

Let R be the region under the graph of f between x  a and x  b .

c Use a GDC to find the area of R.

1
4 Let f  x   and g  x   5x  3 .
x

5
a Given that h  x    f g   x  , for x  find h.
3

b Find  h  x  dx .

© Oxford University Press 2019 1


Mixed review

5 Consider the function f with second derivative f (x)  3x  4 . The graph of f has a minimum
 1 358 
point at A   ,   and a maximum point at B 3, 4 .
 3 27 

a Use the second derivative to justify that A is a minimum point.

3 2
b Given that f   x    x  4x  p , find p.
2

c Find f  x  .

Exam-style questions

6 Consider the functions f and g defined by f  x   x2  x  1 and g  x   2x  1 .

a Find expressions for

i g f  x

ii f  x . (4)

2g  x 
b Find  f x dx . (2)

7 A geometric sequence has first term e and third term e2 .

a Find the common ratio. (2)

b Hence show that all the terms of the sequence lie on the graph of f  x   ex . (3)

c Show that the area of the region enclosed by the graph y  f  x  , the x -axis and the
1
lines x  1 and x  3 is given by 2 e2  e 1 . (3)

8 Consider the function f defined by f  x   ln  x  1 .

a State the largest possible domain D of f . (1)

b Find an expression for f   x  . (2)

c Hence show that f is increasing on D. (2)

d Find an expression for the inverse function f 1  x  . (3)

e Show that the graphs of f and f 1 have a common tangent t at x  0 . (3)

f State the equation of t . (1)

g Hence determine the area of the region enclosed by the tangent t , the graph of f 1 and
the line x  1 (4)

© Oxford University Press 2019 2


Mixed review

9 A particle moves along a straight line passing through the origin.

At time t s the particle’s velocity, measured in cm s–1, is given by v t   0.1t  0.2  3 , for
t

0  t  10 . Use a GDC to answer the following.

a Find the value of t for which

i the acceleration of the article is zero (2)

ii the particle is at rest. (2)

b Find the total distance travelled by the particle in the first 10 seconds. (2)

10 The magnitudes of earthquakes are measured on a logarithmic scale.

The magnitudes of n earthquakes were recorded as: ln k,ln k 2,ln k 3,ln k 4,...,ln k n

where k is a constant greater than one.

(1  n)ln k
Show that the mean of this data is . (7)
2

© Oxford University Press 2019 3


Mixed review

Answers
1
 2 x  5  C
3
1 a
6
4 3
  4x 
2
b  20x  25 dx  x  10x 2  25x  C
3
4 3 125
c x  10x 2  25x  C
3 6
4 3 4  125  125
d x  10x 2  25x  C1  x 3  10x 2  25x    C2  , where C1   C2 .
3 3  6  6

1 4
2 a 0t  2 b 25 cm s−2 c s t   t  t2  5
4
3 a 3 b 2.18 c 1.65
1 1
4 a h x  b ln 5x  3  C
5x  3 5
 1
5 a f      5  0 implies that A is a minimum point
 3
3
b p
2
1 3 3
c f x   x  2x 2  x  13
2 2

6 a i  
g f  x   g x2  x  1   M1

 
2 x 2  x  1  1  2x 2  2x  3 A1

ii f   x   2x  1 M1A1

2g  x  2x  1
b  f x dx  2 2
x  x 1
dx M1


= 2ln x2  x  1  C  A1

e2
7 a r2  er  e M1A1
e

 e  e
n 1 n 1
b Any term un can be written un  e    en1 M1A1

So un  en1  f  n  1 and hence lies on graph of f  x   ex R1


3
3 3 3
x
 x  3 1

c  f  x  dx   ex dx   e 2 dx  2 e 2   2  e2  e2  M1A1A1
1 1 1
 1  
1
 2e2  e  1 AG

8 a x  1 A1
1
b f  x  M1A1
x 1
1
c x  1  f   x   0 M1R1
x 1
Therefore f is increasing in D. AG

© Oxford University Press 2019 4


Mixed review

d x  ln  y  1 M1

ex  y  1 M1
x
y  e 1

f 1  x   ex  1 A1

e  f   x   e
1 x
M1

 f  0  1  f  0 so gradient of tangent to f  x 


1
at x  0 is equal to gradient of tangent to

f   x  at x  0 . R1

Also, f 0  0  f 1 0 , so tangents pass through same point, and are hence the same line.
R1
f y x A1
1

e
x
g  1  x dx M1A1
0

1
 x2  5
 e x  x   e A1A1
 2 0 2

dv
9 a i Use GDC to solve 0 M1
dt
t  3.35 seconds A1
ii Use GDC to solve v t   0 M1

t  9.26 seconds A1
10
b Use GDC to calculate  v t 
0
dt M1

10

 v t 
0
dt  24.1 cm. A1

ln k  ln k 2  ...  ln k n
10 x  M1
n
ln k  2ln k  ...  n ln k
 M1A1
n
ln k
=
n
1  2  ...  n A1

ln k n  n  1
=  M1A1
n 2
 n  1 ln k. A1
2

© Oxford University Press 2019 5


Exam practice

Exam practice: chapters 1 – 10

1 P2: Ben practises playing the Oboe daily.


The time (in minutes) he spends on daily practice over 28 days is as follows.

10, 15, 30, 35, 40, 40, 45, 55, 60, 62, 64, 64, 66, 68,
70, 70, 72, 75, 75, 80, 82, 84, 90, 90, 105, 110, 120, 180

a Find the median time (2)


b Find the lower quartile (2)

c Find the upper quartile (2)


d Find the range (2)

e Determine whether there are any outliers in the data (4)


f Draw a box and whisker diagram for the above data, marking any outliers as required.

2 P1: Find the range of values of k for which the equation 3kx 2 + k 3x + 3 =has
0
two real roots. (6)

1 2 1
3 P1: Consider the quadratic functions f ( x ) = x 2 and g ( x )= x − 3x + .
2 2

a Express g ( x ) in the form g ( x ) = a ( x − h ) + k .


2
(4)

b Find the coordinates of the vertex of the graph of y = g ( x ) . (1)

c The graph of y = f ( x ) is transformed into the graph of y = g ( x ) by a sequence of


three transformations. Describe each transformation in turn. (3)

1
4 P1: Consider the functions f (=
x) x + 12 and g ( x ) = x 2 .
3

(
a Find the value of ( f  g ) 2 3 . ) (2)

b Find an expression for f −1 ( x ) . (2)

c Solve the equation f −1  g ( x ) = 0 . (3)

5 P2: Jake and Elisa are given a mathematics problem.


The probability that Jake can solve it is 0.35.

If Jake has solved it, the probability that Elisa can solve it is 0.6, otherwise it is 0.45.
a Draw a tree diagram to illustrate the above situation, showing clearly the probabilities
on each branch. (3)

b Find the probability that neither student solves the problem. (2)
c Find the probability that at least one of the students can solve the problem. (2)

d Find the probability that Jake solves the problem, given than Elisa has. (4)

© Oxford University Press 2019 1


Exam practice

8
 1
6 P1: In the binomial expansion of  2x 3 −  , find the coefficient of the term containing x12 .
 x
(6)

2 1
7 P1: Consider the function f ( x ) = , x ≠ , x∈.
3x − 1 3
a State the range of f . (1)

b Sketch the graph of y = f ( x ) , stating clearly the equations of any asymptotes.

State also the coordinates of any points of intersection with the x and y axes. (5)

c Find the inverse function f −1 ( x ) . (3)

5 − 8x 3
8 P1:Consider the function f ( x ) = x ≠ − , x∈.
4x + 3 4

a Write down the equations of the two asymptotes on the graph of y = f ( x ) . (2)

b State the range of f . (1)

ax + b
c Find an expression for f  f ( x ) , giving your answer in the form f  f ( x ) = .
cx + d
State also the domain of f  f ( x ) . (5)

9 P1: A population of ferrets has mean age 5.25 years and standard deviation 1.2 years.

a Find the mean age of the same ferrets 3 years later. (2)
b Find the standard deviation of the same ferrets 2 years later. (2)

10 P2: The following raw data is a list the height of flowers (in cm) in Eve’s garden:
26.5, 53.2, 27.5, 33.6, 44.6, 39.5, 24.9, 45.1, 47.8, 39.3, 33.1, 38.7, 44.1, 22.3, 44.1,
30.5, 25.5, 35.9, 37.1, 40.2, 23.3, 36.2, 34.8, 37.3
a Copy and complete the following grouped frequency table:

height, (x cm) frequency

20 ≤ x < 25

25 ≤ x < 30

30 ≤ x < 35

35 ≤ x < 40

40 ≤ x < 45

45 ≤ x < 50

50 ≤ x < 55

(3)
b Find an estimate for the mean height, using the frequency table. (2)

c Find an estimate for the variance, using the frequency table. (2)
d Find an estimate for the standard deviation, using the frequency table . (2)

Eve’s neighbour’s garden was also surveyed. It was found that the flowers in the
neighbour’s garden had a mean height of 32.1 cm and standard deviation 7.83 cm.

e Compare the heights of the flowers in both gardens, drawing specific conclusions.

© Oxford University Press 2019 2


Exam practice

11 P1: ‘Icicles creamery’ decided to analyse their ice-cream sales to see if there was any
correlation between sales and the average outdoor temperature for that particular
month.
The following data was collected:

Month Mean temperature ( C)



Sales ($ )

January 3 350

February 4 650

March 9 900

April 11 920

May 17 1080

June 22 1200

July 25 1260

August 29 1390

September 19 1220

October 11 880

November 8 770

December 6 500

a Draw a scatter diagram to represent the data. (2)


b Describe the correlation between mean outdoor temperature and sales of ice-cream.(1)

c Comment on whether you can conclude, from this data, that outdoor temperature
affects ice-cream sales. (2)
12 P2: Neeve conducts a test to determine if there is any correlation between a person’s age
and the number of hours they watch television per week.

Age 8 42 17 81 45 14 39 42 31 40 28 24

No. of
20 15 30 2 25 28 19 14 16 21 26 20
hours

a Find Pearson’s product-moment correlation coefficient ( r ) for this data. (2)

b Interpret the value of Pearson’s product-moment correlation coefficient ( r ) in the


context of the question. (1)

c Find the equation of the y on x regression line. (2)

d Using your result from part c), determine the number hours per week that a
60-year-old might be expected to watch television. (2)
From her data, Neeve concludes that a person’s age affects the number of hours per week
that they tend to watch television.

e Explain whether or not this is a valid conclusion. If not, suggest what conclusion Neeve
could draw from these results. (3)

1
13 P1: a Show that log16 4 = . (3)
2

1
b Hence or otherwise, solve the equation log16 ( x − 4 ) − log16 ( x − 12 ) =. Give your
2
answer as an exact value. (4)
© Oxford University Press 2019 3
Exam practice

6
dx
14 P1: Find the value of the definite integral ∫
1 3x − 2
. (5)

x
15 P2: a Sketch a graph of the region bounded by the curves y = e 2 and =
y 10 − x 2 . (3)

b Write down a definite integral representing the area of the region drawn. (3)
c Find the size of the bounded area using a GDC. (1)

( x − 6)
2

16 P1: Find the indefinite integral ∫ x2


dx . (6)

17 P1: Consider the function f ( x ) =


x − 4 x, x ≥ 0 .

a Find an expression for f ′ ( x ) . (2)

b Find an expression for f ′′ ( x ) . (1)

c Find the coordinates of any turning point(s) on the curve and determine their nature.
(5)

d Sketch the graph of y = f ( x ) , showing clearly the coordinates of any turning points
and intersections with axes. (6)

x2
by f ( x )
18 P2: Consider the function defined= ,x ≠ 3 1 .
2x 3 − 1 2

a Find an expression for f ′ ( x ) . (3)

b Find the equation of the tangent to the curve y = f ( x ) at the point where x = 1 .
(4)

c Find the coordinates of the points on the curve y = f ( x ) where the gradient is zero.
(6)

d Without the use of technology, determine the range of values of x for which f ( x ) is an
increasing function. To gain full marks, you must show all your working. (4)

19 P1: A and B are events such that P( A) = 0.3 , P(B) = 0.65 and P ( A ∪ B ) =
0.7 .

By drawing a Venn diagram (or otherwise), find:


a P ( A ∩ B) (2)

b P ( A ∪ B′ ) (2)

c P ( A′ ∩ B )′ (2)

20 P2: The first four terms of an arithmetic sequence are 80, 99, 118, 137.
a Find the 30th term in the sequence. (3)

b Find the sum to 13 terms. (2)


c Find the least number of terms required so that the sum exceeds 50 000.
To gain full marks, you must show all of your working. (5)

21 P1: Solve the equation 52 x − 5x +2 + 100 =


0 . Give all solutions in exact form. (7)

© Oxford University Press 2019 4


Exam practice

Answers
1 a 69 minutes M1A1
b 50 minutes M1A1
c 83 minutes M1A1
d 170 minutes M1A1
e Interquartile range is 83 − 50 =
33 A1
Q3 + 1.5 × IQ range = 83 + 1.5 × 33 M1

= 132.5
Q1 − 1.5 × IQ range = 50 − 1.5 × 33 M1

= 0.5
Therefore 180 is an outlier A1
f

M1A1A1
2 Two real roots implies b2 − 4ac > 0 M1

(k 3 )
2
− 4 (3k ) (3) > 0 A1

3k 2 − 36k > 0

k 2 − 12k > 0

k ( k − 12 ) > 0 M1

Critical values=
are k 0=
and k 12 A1
Solution is k < 0 or k > 12 A1A1
1 2 1 1 2
3 a g ( x )= x − 3x + =  x − 6 x + 1 M1
2 2 2
1
( x − 3) − 9 + 1
2
=  M1A1
2  

1 1
( x − 3) − 8 = 2 ( x − 3) − 4
2 2
= A1
2 
b (3, −4) A1

c Horizontal translation 3 units to the right A1


1
Vertical stretch scale factor A1
2
Vertical translation down 4 units A1

4 a (f ( )
 g ) 2 3 = f (12 ) = 16 M1A1

1
b y= x + 12 ⇒ x = 3 ( y − 12 ) M1
3

( x ) 3 ( x − 12)
So f −1 = A1

c ( x ) 3 x 2 − 12
f −1  g= ( ) M1A1

f −1  g ( x ) =0⇒x =±2 3 A1

© Oxford University Press 2019 5


Exam practice

5 a

M1A1A1
b 0.65 × 0.55 =
0.3575 M1A1

c 1 − ( 0.65 × 0.55) =
0.6425 M1A1
P ( Jake and Elisa solve ) 0.35 × 0.6
d = M1A1A1
P (Elisa solve ) (0.35 × 0.6 ) + (0.65 × 0.45)
= 0.418 A1
8−r
 1
( )
r
6 The general term in the expansion is 8Cr 2 x 3 −  M1
 x

 2r x 3 r 
= ( −1)
8−r 8
Cr  8 − r = ( −1)
8−r 8
(
C r 2r x 4 r − 8 ) A1
 x 
Therefore 4r − 8 =
12 M1
So r = 5 A1
3
 1
( )
5
So the term required is 8C5 2 x 3 −  =−56 × 25 × x12
 x

The coefficient is therefore −56 × 25 A1


= −1792 A1

7 a (
Range is f ( x ) ≠ 0 , f ( x ) ∈  ) A1

M1A1A1
1
Asymptotes x = and y = 0 A1
3
Intersects y − axis at ( 0, −2 ) A1

© Oxford University Press 2019 6


Exam practice

2
c y =
3x − 1
y (3x − 1) =
2 M1

3xy − y =
2
3xy= 2 + y

2+y
x =
3y

2+x  1 2
f −1 ( x=
) 3x
) 3 + 3x 
 or f ( x=
−1
A1A1
 
3
8 a x = − A1
4
y = −2 A1

b (
Range is f ( x ) ≠ −2 , f ( x ) ∈  ) A1

 5 − 8x 
5 − 8 
c f  f (x) =  4x + 3  M1A1
 5 − 8x 
4 +3
 4x + 3 
5 ( 4 x + 3 ) − 8 (5 − 8 x )
= A1
4 (5 − 8 x ) + 3 ( 4 x + 3 )

20 x + 15 − 40 + 64 x 84 x − 25
= = A1
20 − 32 x + 12 x + 9 29 − 20 x
29
x ≠
20
, ( x ∈ ) A1

9 a 8.25 years
5.25 + 3 = M1A1
b 1.2 years M1A1
10 a
height, (x cm) frequency
20 ≤ x < 25 4
25 ≤ x < 30 2
30 ≤ x < 35 4
35 ≤ x < 40 7
40 ≤ x < 45 4
45 ≤ x < 50 2
50 ≤ x < 55 1
M1A1A1
b Using GDC, mean height = 35.6 cm M1A1
c Using GDC, variance = 68.3 cm2 M1A1
d Using GDC, SD = 8.27 cm M1A1
e On average, the neighbour’s garden’s flowers had a slightly lower height compared
to Eve’s. R1

The neighbour’s flowers also had a smaller standard deviation, indicating they were grown
to a more consistent length. R1R1

© Oxford University Press 2019 7


Exam practice

11 a

M1A1
b (strong) positive correlation A1
c You cannot be sure that outdoor temperature affects ice-cream sales, as correlation does
not imply causation. R1
The graph demonstrates that there is a link between outdoor temperature and ice-cream
sales, but you cannot say that one causes the other as there may be other factors
involved. R1
12 a Use of GDC to give r = −0.775 M1A1
b This is a strong negative correlation. i.e. as a person’s age increases, the number of hours
they watch TV decreases. R1
c Use of GDC to give y =
−0.306 x + 30.1 M1A1
d y =
−0.306 × 60 + 30.1 M1
= 11.7 hours A1
e Neeve is incorrect. A1
A value of r = −0.775 indicates a strong negative correlation between a person’s age and
the hours per week they watch TV. R1
However, you cannot say this is causal. R1
(i.e. You cannot conclude that your age affects the amount of TV you watch.)
1
13 a log16 4 = M1
log4 16

1
= M1
log4 42

1 1
= = A1
2log4 4 2

1
b log16 ( x − 4 ) − log16 ( x − 12 ) =
2
log16 ( x − 4 ) − log16 ( x − 12 ) =
log16 4 M1

 x−4 
log16   = log16 4 M1
 x − 12 
x−4
=4 A1
x − 12
x − 4 = 4 x − 48
3x = 44
44
x = A1
3

© Oxford University Press 2019 8


Exam practice

6 6
dx
∫ (3x − 2 )
− 12
14 ∫
1
=
3x − 2 1
dx M1

2 1 6

= ( 3x − 2 ) 2  M1A1
3  1
2
=
3
( 4 − 1) M1

=2 A1
15 a

M1A1A1

(10 − x )
2.538
x
b ∫
2
− e 2 dx
−3.129

Correct limits A1
Correct expression A1
All correct A1
c 34.3 A1

( x − 6)
2
x 2 − 12 x + 36
16 ∫ x2
dx = ∫ x2
dx M1A1

12
= ∫1 − + 36 x −2 dx M1A1
x
36
=x − 12ln x − +c M1A1
x

f ( x=
) x − 4x 2
1
17 a

f ′ ( x )= 1 − 2 x
− 12
M1A1

f ′′ ( x ) = x
− 23
b A1

c Attempting to solve f ′ ( x ) = 0 M1
− 12
1 − 2x 0
=
2
1− 0
=
x
x =4 A1

f ( 4) =
4−4 4 ( )=
−4
1
2
A1

© Oxford University Press 2019 9


Exam practice

Therefore ( 4, −4 ) is the only turning point

1 1
f ′′ (=
4 ) 4=
−3
2
= A1
8
3
4 2

1
> 0 . Therefore a minimum point R1
8
d Intersects y − axis at ( 0, 0 ) A1
1
Intersects x − axis where x − 4 x 2 =
0 M1

(
x 1 − 4x
− 12
)=
0 M1

− 12
1 − 4x 0 or x = 0
=
4
1− 0
=
x
x = 16 A1

So crosses x − axis at (16, 0 )

A1 shape; A1 minimum; A1intercepts


18 a Attempt to use quotient rule M1

=f ′(x)
( 3
2x 2x − 1 − x 6 x
=
) 2
−2 x 4 − 2 x( 2
) A1A1
( ) ( )
2 2
2x 3 − 1 2x 3 − 1

b At x = 1 , y = 1 A1

−4
f ′ (1) = = −4 A1
1
Equation is y − 1 =−4 ( x − 1) M1A1

Or y= 5 − 4 x

c Setting f ′ ( x ) = 0 M1

−2 x 4 − 2 x
=0
(2 x )
2
3
−1

−2 x 4 − 2 x =
0 M1

2x 4 + 2x =
0

(
2x x 3 + 1 =
0 ) M1

So x = 0 or x = −1 A1

Coordinates are ( 0, 0 ) and ( −1, − 1


3 ) A1A1

© Oxford University Press 2019 10


Exam practice

d Setting f ′ ( x ) > 0 M1

−2 x 4 − 2 x > 0

2x 4 + 2x < 0

( )
2x x 3 + 1 < 0

Critical values are x =


−1, x =
0 A1

(or similar method to consider sign of f ( x ) either side of critical values) M1

So solution is −1 < x < 0 A1

19 a P ( A ∩ B ) = P ( A ) + P ( B ) − P ( A ∪ B ) = 0.3 + 0.65 − 0.7 = 0.25 M1A1

b ( )
P ( A ∪ B′ ) = P ( A ) + 1 − P ( A ∪ B ) = 0.3 + (1 − 0.7 ) = 0.6 M1A1

c This is the same region as described in part b R1


so has probability 0.6. A1
20 a u30 = u1 + ( n − 1) d M1

= 80 + 29 × 19 A1
= 631 A1
n
b S= 2u1 + ( n − 1) d  M1
2
13

13
= 160 + 12 × 19
2 
= 2522 A1
c Require Sn > 50 000

n
Consider 160 + 19 ( n − 1)  =
50 000 M1
2
19n2 + 141n − 100 000 =
0 A1
Valid attempt to solve three term quadratic M1
−141 ± 2760.4
n=
38
−141 + 2760.4
n > 0 , so n
= = 68.9 A1
38
So require 69 terms. A1
21 52 x − 25 × 5x + 100 =
0 M1

Substitute u = 5 x
(or recognise as a quadratic in 5 ) x
M1
u2 − 25u + 100 =
0 A1

(u − 20) (u − 5) =
0 or other valid attempt to solve M1

=u 20
= or u 5 A1
ln20
5x = 20 ⇒ x = log5 20 (or x = or similar) A1
ln5
5x = 5 ⇒ x = 1 A1

© Oxford University Press 2019 11


End of chapter test

11 Relationships in space: geometry


and trigonometry in 2D and 3D
A calculator is allowed

1 Find the midpoint of and the distance between the following points

a (2, 6 ) and ( −3, −5)


b (3,5 , 4)  and  (9, 6, 2) .
2 The shape shown is a cuboid.

a Find the volume of the cuboid.

b Find the length AC.

c Find the length AG.

d Find the angle that AG makes with the plane ABCD.

3 Coco is standing 20m away from a tree at the end of its shadow and measures the angle of
elevation of the sun as 37° . How tall is the tree?

4 The body of a juice carton is the shape of a cuboid with a square base. The top of the carton is
a square based pyramid.

a The size of the angle between TF and FG.

b The total surface area of the four triangular sections of the roof.

c The volume of juice in the carton.

© Oxford University Press 2019 End of chapter test 1


End of chapter test

5 A fishing weight is made by combining a hemisphere of radius 3 cm and a right circular cone of
height 4cm as shown on the diagram.

a Find the volume of the fishing weight

b Find the total surface area.

6 Find the smallest angle in a triangle of sides 6cm, 7cm, 8cm.

7 Grumpy and Happy have a garden each that is joined by fence BD

a Find the length of the fence.

b Find two possible values for angle CBD.

c If angle CBD is acute, find the perimeter, ABCDA, of their gardens.

8 An airplane leaves Manpool on a bearing of 030° to Doondee, a distance of 250km. They take
off from Doondee and head to Bernich on a bearing of 100° for 400km.

a Show this on a diagram.

b What is the bearing and distance that the pilot must use to return home to Manpool?

© Oxford University Press 2019 2


End of chapter test

Answers

 x1 + x2 y1 + y2   2 − 3 6 − 5   1 1 
1 a Midpoint =  ,
 =  , =  − , 
 2 2   2 2   2 2

( x2 − x1 ) + ( y2 − y1 ) = ( −3 − 2) + ( −5 − 6 ) =
2 2 2 2
Distance = 146 = 12.1

 x + x2 y1 + y2 z1 + z2   3 + 9 5 + 6 4 + 2   11 
b Midpoint    1 =
= , ,  = , ,   6, , 3 
 2 2 2   2 2 2   2 

( x2 − x1 ) + ( y2 − y1 ) + ( z2 − z1 ) = ( 9 − 3) + ( 6 − 5) + (2 − 4 ) =
2 2 2 2 2 2
Distance = 41 = 6.40

2 a V = lbh = 3 × 5 × 10 = 150 m3

b AC 2 =52 + 102 =125

AC
= 125 11.2 m
=

Remember to use 125 and not 11.2 in any further calculations!

( )
2
c 32 + 125
AG = 134
=

AG
= 134 11.6 m
=

CG 3
d sin CAG
= =
AG 134

= sin−1 15.0°
∠CAG

h
3 tan 37° =
20

=h 20tan37
= ° 15.1m

3
4 a cos TFG
= = 0.3
10

∠TFG = cos−1 0.3 = 72.5°

1
b Surface area = 4 × × TF × FG × sin TFG = 2 × 10 × 6 × sin72.5° = 114 cm2
2

c The volume of juice in the carton=Volume of the cuboid+ volume of the pyramid

1 
V =
lbh +  base area × height 
3 

The height of the pyramid


Find EG

EG 2 = 62 + 62 = 72

EG = 72
2
1 
Height =102 −  72  =
9.06 cm
2 

1
V = (6 × 6 × 40) +  3 × 6 × 6 × 9.06  = 1550 cm3
 

© Oxford University Press 2019 3


End of chapter test

5 a Total volume = volume of the hemisphere + volume of the cone

2 1 2 1
V = π r 3 + π r 2h = π ( 3 ) + π ( 3 ) ( 4 ) =
3 2
30π =94.2 cm3
3 3 3 3
b The slant height (l) of the cone is 5cm
Total surface area (SA) = SA of the hemisphere + Curved SA of the cone

1
π rl = π (3) + (π × 3 × 5) = 33π = 104 cm2
2
SA = × 4π r 2 +  2
2

6 Find the smallest angle in a triangle of sides 6cm, 7cm, 8cm.

72 + 82 − 62 77
=cos θ =
2×7×8 112

77
=θ cos−1 = 46.6°
112

7 a BD2 = 42 + 82 − (2 × 4 × 8 × cos 40°)

BD = 5.57

sin CBD sin25°


b =
12 5.57

12sin25°
CBD =sin−1 =65.7° or 1
 14°
5.57

c ∠BDC
= (180 − 65.7 − 25=
)° 89.3°

BC 5.57
=
sin 89.3 sin25

BC = 13.2

Perimeter
= 13.2 + 4 + 8 + 12
= 37.2
8 a

b ∠MDB = 360 − 100 − 150 = 110°

= 2502 + 4002 − (2 × 250 × 400 × cos110)


BM

BM = 539km

4002 + 2502 − 5392


cos BMD =
2 × 400 × 250

∠BMD = 44.3°
Bearing M to B = 074
Bearing B to M = 360 − 74 = 286°

© Oxford University Press 2019 4


Mixed review

11 Relationships in space: geometry


and trigonometry in 2D and 3D
1 If the first term in an arithmetic series is 2 and the 4th term is 14, find:

a the common difference

b the sum of the first 14 terms.

2 The function f is given by f ( =


x) x − 4.   Sketch the function showing 2 points and state its
domain and range.

2x − 1
3 a Write down the asymptotes of f ( x ) =
x −1

2x − 1
b Solve = x3 + 2
x −1

4 Match the following statistics with the histograms.

a Mean 49. Standard deviation 20

b Mean 52. Standard deviation 40

c Mean 68. Standard deviation 16

Histogram 1 Histogram 2 Histogram 3

4
 p2 
5 If
= log9 p x=
,  log9 q y   and log
= 9 r z   , express log9  3   
in terms of x, y and z.
q r 

1
6 Consider the functions f ( x ) = 4 x and g ( x ) =
x −3

a Calculate ( f  g ) (5)

b Find g −1 ( x )

c Write down the domain of g −1.

7 The rate of change of the number of fish (f ) in a pond is modelled by the equation
df
= 2t + 5t 1.5 , where t is the time in months. If there are 100 fish originally, how many will
dt
there be after 4 months? Round your answer to the nearest fish.

© Oxford University Press 2019 Mixed review 1


Mixed review

8 The diagram shows a rectangular play area ABCD


enclosed on three sides by 120 m of fencing, and on
the fourth by a wall AB.

Find the maximum play area.

Exam-style questions

9 Mrs Smith´s greenhouse has a maximum height of 2.8 metres.

The two vertical, rectangular sides are 2 metres high and 3 metres long.

The roof consists of two equal rectangles, each of which has an area of 6 m2.

a Find the dimensions of the rectangles that make the roof. (2)

b Hence find the width of the greenhouse. (3)

c Determine the total surface area of all the vertical sides of the greenhouse. (3)

d Calculate the volume of the greenhouse. (3)

Mrs Smith spent $2350 on the construction of the greenhouse. She borrowed this amount from
the local bank that offered her a special deal. Mrs Smith will pay a fixed amount of $50 per
month for 5 years.

e Calculate the annual interest rate the bank is charging Mrs Smith. (4)

10 John draws a rectangle and a square. Let x represent the length of the side of the square in
centimetres. The length of the rectangle is 3 cm longer than the side of the square and the
width of the rectangle is double the length of the side of the square.

a Write down an expression for the sum of the areas of the two shapes. (2)

The sum of the areas of the rectangle and square is 24 cm2.

b Show that x is solution of the equation x 2 + 2 x − 8 =


0. (2)

c Hence find the dimensions of the two shapes. (4)

11 A rock-climber slips off a rock-face and falls vertically. At first, he falls freely under gravity, but
after 3 seconds a safety rope slows his fall.

The height, h metres, of the rock-climber t seconds after he falls is given by:

 150 − 5t 2 , 0 ≤ t ≤ 3
h (t ) =  2
t − 33t + 195,3 < t ≤ 5

a Find the height of the rock-climber when t = 2 s. (2)

© Oxford University Press 2019 2


Mixed review

b Sketch a graph of h against t for 0 ≤ t ≤ 5 . On your sketch, label the coordinates of the
points at which=
t 0, =t 3 and= t 5. (5)

dh
c Find for
dt

i 0<t <3 ii 3 < t < 5. (3)

d Find the velocity of the rock-climber when t = 2. (2)

e Show that the velocity of the rock-climber is increasing in the interval 3 < t < 5 . (2)

f Gianni is 100 metres away from the base of the vertical rock when the climber reaches his
minimum height whilst falling during the interval 0 ≤ t ≤ 5 . By assuming that Gianni is
looking from ground level, estimate the angle of elevation, in degrees, at which Gianni looks
at the climber. (3)

12 When the top T of a mountain is


viewed from point A, which is 2000 m
above the ground, the angle of
depression is 15°.

When T is viewed from point B on the


ground, the angle of elevation is 10°.

Given that the points A, B and T lie in


the same vertical plane, find

a the size of the angle ATB (2)

b the height of the mountain, giving the answer correct to 1 decimal place. (4)

a+b
13 The arithmetic mean of two positive real numbers a and b is .
2

Their geometric mean is ab.

a For the numbers 1 and 4, find

i their arithmetic mean ii their geometric mean. (2)

( )
2
b Use the fact that, for any real numbers a and b, a− b ≥ 0 to prove that the
arithmetic mean of two real numbers is greater than, or equal to, the geometric mean. (4)

c i Given that a and b are positive real numbers, apply the result from part b to a2 and
b2 to prove that a2 + b2 − 2ab cos C ≥ 0 for any angle C . You must fully justify your
proof.

ii State the geometrical rule which is associated with the result you proved in part i.
Explain how this geometrical rule, and the proved inequality, are consistent with one
another. (7)

© Oxford University Press 2019 3


Mixed review

Answers
1 a 4 b 392
2

Domain x ≥ 4
Range y ≥ 0
3 a=x 1,
=  2
y b −0.819, 1
 .38
4 a2, c1, b3.
5 8 x − 12y − 4 z
1
6 a 2 b g −1 ( x=
) +3 c x ≠0
x
7 181
8 1800 m2
9 a 6 ÷3 =
2
2 metres by 3 metres A1A1
b 2 22 − 0.82 =
3.67 M1A1A1
  3.66.. × 0.8  
c 2  (3 × 2 ) +   + 3.66.. × 2  M1A1
  2  
= 29.6 m2 A1
3.66.. × 0.8
d 3× + 2 × 3.66.. × 3 M1A1
2
= 26.4 m3 A1
5
 R 
e 2350 1 + 50 60
 =× M1A1A1
 100 
5% A1
10 a x + 2 x ( x + 3)
2
A1A1

b x 2 + 2 x ( x + 3) =
24 M1

3x 2 + 6 x − 24 =
0 A1
2
x + 2x − 8 =
0 AG
c ( x + 4) ( x − 2) =
0 M1

x =2 A1
Square has dimensions 2 × 2 A1
Rectangle has dimensions 5 × 4 A1

© Oxford University Press 2019 4


Mixed review

11 a h (2=
) 150 − 5 × 2=
2
130 metres M1A1

(Three points correctly labelled) A3


(correct shape of each branch of the graph) A1A1
c i h′ ( t ) =−10t , 0 < t < 3 M1A1

ii h′ ( t ) = 2t − 33, 3<t <5 A1

d h′ (2 ) = −20 m s−1 M1A1

e h′′ ( t ) = 2 > 0, 3<t <5 A1R1

Hence the velocity is increasing AG


55
f tan θ = M1A1
100
−1  55 
=θ tan
=   28.8º A1
 100 
12 a 10 + 15 =
25º M1A1
sin25º sin75º
b First find length BT: = M1A1
2000 BT
BT = 4571.15... metres A1
Now find
= = 794 metres
height of T: h 4571.15...sin10º A1
13 a i 2.5 ii 2 A1A1

( )
2
b a− b ≥0

⇒ a−2 a b +b ≥ 0 M1A1
a+b
⇒ a+b ≥2 a b ⇒ ≥ ab M1A1
2
So the arithmetic mean is greater or equal to the geometric mean. AG
2 2
a +b
c i Part b ⇒ ≥ a2b2 M1
2
⇒ a2 + b2 ≥ 2ab A1
2 2
⇒ a + b ≥ 2ab cos C A1
for any angle C, since −1 ≤ cos C ≤ 1 and a, b > 0 R2
So a2 + b2 − 2ab cos C ≥ 0 AG
ii For a triangle with sides a, b, c and opposite angles A, B, C , the cosine rule states that
c 2 = a2 + b2 − 2ab cos C A1
As c ≥ 0 this is consistent with the inequality in c i
2
R1
© Oxford University Press 2019 5
End of chapter test

12 Periodic relationships:
trigonometric functions
Section A. A calculator is not allowed

3
1 If sin θ =   and 0 < θ < 90° , find the exact value of cos θ .
8

2 The circle with centre O has a radius of 4cm.

If ∠AOB =
1.5 radians , find

a the length of arc ACB b the shaded area.

3 Solve, giving your answer in radians as an exact value with 0 ≤ x ≤ 2π .

a sin x + 4 =
5 b 3 tan 2
x+ =
1 c 10 cos x − 5 2 =
0

4 Solve sin2 x = 2 cos x , for 0 ≤ x ≤ 3π

5 a Write sin x + 2 cos2 x in terms of sin .


x

b Hence solve sin x + 2 cos2 x − 2 =


0 in the range 0 ≤ x ≤ π

sin2 x
6 Show that = tan x
1 + cos 2 x

of f ( x ) acos ( bx )  +c . Find the values of a,  and 


7 This is the graph = b c.

© Oxford University Press 2019 End of chapter test 1


End of chapter test

Section B. A calculator is allowed

8 The diagram shows a sector of a circle centre O, with radii OA and OB.

=
Find the shaded area when OA 15
= cm and  θ 2 radians

9 a Write 4 cos x + 3 sin2 x in the form acos2 x + bcos  x + c

π
b Hence solve 4 cos x + 3 sin2 x =
4 , with 0 ≤ x ≤
2

10 The depth (d) of water at the end of a pier on Tuesday is modelled by

π 
d ( h ) 4 sin h   + 8
=
6

where h is the number of hours after midnight.

a Find the maximum depth of water.

b Sketch the graph showing the height of water for Tuesday and draw a line on the sketch to
show the times between which there is 10m or more of water at the end of the pier. State
these times (in the form am and/or pm) under your sketch.

© Oxford University Press 2019 2


End of chapter test

Answers

1 sin2 θ + cos2 θ =
1

2
3 55
cos2 θ =
1 − sin2 θ =
1−  =
8 64

55
cos θ =
8

2 a s=rθ =4 × 1.5 =6 cm

1 1
b A = r 2θ = × 16 × 1.5 =12 cm2
2 2

π
3 a sin
= x 1,   
= x
2

1 5π 11π
b tan x =
− ,    x = , 
3 6 6

2 π 7π
c =
cos x = ,   x ,   
2 4 4

4 2sinxcosx = 2cosx

2 sin xcos 2
x − cos x =
0

2cosx ( sinx − 1) =
0

cosx 0   
= = sinx 1

π 3π 5π
x = , , 
2 2 2

5 a sin x + 2 cos2 x =
sin x + 2(1 − sin2 x ) =
−2 sin2 x + sin x + 2

b sin x + 2 cos2 x − 2 =
0

−2 sin2 x + sin x =
0

sin x(1 − 2 sin x ) =


0

1
sinx 0, sin
= = x
2

π 5π
π
x = 0,   , , 
6 6

sin2 x 2 sin x cos x sin x


6 = = = tan x
1 + cos 2 x 1 + 2 cos2 x − 1 cos x

1
7 (=
a)
amplitude 5
2
( − −=
1) 3

2π 2π
Period
= = 3b=
b 3

Vertical shift ( c ) = 5 − 3 = 2

© Oxford University Press 2019 3


End of chapter test

1 2 1
8 Shaded area
= r θ − × OA × OB × sin θ
2 2

1 1
(15) (2) − 2 × 15 × 15 ×=
2
Shaded area
= sin2 123 cm2
2

9 a 4 cos x + 3 sin2 x =
4 cos x + 3(1 − cos2 x ) =
−3 cos2 x + 4 cos x + 3

b 4 cos x + 3 sin2 x =
4

−3 cos2 x + 4 cos x + 3 =4

3 cos2 x − 4 cos x + 1 =0

(3 cos x − 1)(cos x − 1) + 0

1
cos x = ,1
3

x = 0, 1.23

π 
10 a Maximum depth when h   = 1 , d ( h=
) 4 (1) + 8= 12m
6

1am to 5am and 1pm to 5pm

© Oxford University Press 2019 4


Mixed review

12 Periodic relationships:
trigonometric functions
1 Convert each angle measure from degrees to radians. Give exact answers in terms of π.

a 270° b 135° c 240°

2 Convert each angle measure from radians to degrees.

π 8π 7π
a b c
2 3 2

3 Without using your GDC, solve each equation for −π ≤ θ ≤ 2π.

θ 
a sin   = 0 b sin2 θ = cos2 θ
2

4 The velocity (v) of a ball in a science experiment is modelled by v (=


t ) 25 − 4t 2 where t is the
time measured in seconds.

a If the displacement (s) after 3 secs is 10m, find an expression for s in terms of t.

b At what time is the displacement a maximum?

c Find the distance travelled in the first second.

5 f ( x ) x=
=  and  g ( x ) 2 x

a Sketch f and g on the same axes

b Write down an expression for the area contained between f and g.

c Find the area.

d The line x = a divides the area in half. Find the value of a.

Exam-style questions

6 The diagram shows a circle with centre O and radius r . The points A, B
and C lie on the circumference of the circle. The area of the sector OABC is
4 cm2 and the length of the arc ABC is 3 cm.

^
Let θ = A O C , where θ is measured in radians.

a Determine the exact values of

i r

ii θ (4)

b Hence find the area of the triangle AOC, giving your answer correct to three significant
figures. (2)

© Oxford University Press 2019 1


Mixed review

7 The diagram shows a circle with centre O and radius r .

The points P and Q lie on the circumference of the circle.

^
π
Let P O Q = 2α with 0 < α < and l be the length of the arc PQ.
2

a Show that PQ = 2r sin α . (4)

π
Consider the function defined by f ( x
= ) 2x − 3 sin x , 0 < x < .
2

b Sketch the graph of f , showing clearly its zero. (2)

Given that l = 1.5PQ ,

c Use the graph of f to find the value of α . (3)

d State the values of α for which l < 1.5PQ (3)

The area shaded is 4.

e Find the value of r . (3)

8 The following diagram shows a triangle ABC, where a > 0 is a parameter to be determined.

^
π
AB = 10a , AC = 8a and B A C =
3

12
a Show that a = . (3)
7

b Hence calculate the lengths of AB and AC, giving your answers as decimals to 3 significant
figures. (2)

Let D be a point on the side AB such that AD = x .

c Find an expression for d ( x ) , the length of the line DC , given as a function of x. (2)

d Sketch the graph of y = d ( x ) , indicating clearly the coordinates of its minimum point. (3)

e For the value of x that minimizes d ( x ) ,

i ˆ , showing your working or justifying your reasoning


calculate the size of angle ADC

ii determine the perimeter of the triangle ADC. (4)

9 A quadratic sequence has an nth term of n2 – 4n + 21.

a Find the 6th term of the sequence. (1)

b Prove that every term in the sequence is positive. (4)

© Oxford University Press 2019 2


Mixed review

10 The diagram shows the movement of a bicycle pedal. The position of the pedal is represented
by the point P that rotates counter-clockwise about the point A, which is located 26 cm above
the ground.

Let θ represent the angle which the line segment AP makes with the horizontal line through A,
as shown in the diagram.

AP = 15 cm and θ = 2.1t , where t represents the time, in seconds, after the start of the
movement.

a State the coordinates of the initial position of the points A and P. (2)

b Show that the height of the pedal above the ground t seconds after the start of the
movement is given by h (=
t ) 26 − 15 sin (2.1t ) , t ≥ 0 . (3)

c Sketch the graph of h against t for 0 ≤ t ≤ 5 , indicating clearly the coordinates of its
maximum and minimum points, correct to three significant figures. (3)

d Determine how long it takes for the pedal to complete a full turn about the point A. (2)

e Describe a sequence of transformations which are required to obtain the graph of h from
the graph of s ( t ) = sin t , indicating clearly the order in which transformations need to be
performed. (4)

© Oxford University Press 2019 3


Mixed review

Answers
 π  270π 3π  π  135π 3π  π  240π 4π
1 a 270°  = = b 135°  = = c 240°  = =
 180°  180 2  180°  180 4  180°  180 3

π  180°  180° 8π  180°  1440° 7π  180°  1260°


2 a  = = 90° b  =  = 480° c  =  = 630°
2 π  2 3  π  3 2  π  2

θ  θ
3 a sin  = 0 ⇒ = 0, π ⇒=
θ 0, 2π
2
  2

sin2 θ
b sin2 θ = cos2 θ ⇒ = 1 ⇒ tan2 θ = 1 ⇒ tan θ = ±1 ⇒ θ = ±45°, ±135°, ±225°, ±315°
cos2 θ
4 2 2
4 a s ( t ) = 25t − t − 29 b 2.5secs c 23 m
3 3
5 a

4
b ∫ (2
0
x − x dx ) c 2.67 d a = 1.51

6 a i rθ = 3 A1
θ
r2 =4 A1
2
Solving simultaneously using an appropriate method M1
8
r = cm A1
3
9
ii θ = radians A1
8
2
1 8 9
=b A =   sin   3.21 cm
2
M1A1
2 3 8
 
7 a Cosine rule gives

PQ = r 2 + r 2 − 2r 2 cos 2α M1A1

(
= 2r 2 − 2r 2 1 − 2 sin2 α ) M1

= 4r 2 sin2 α M1
= 2r sin α AG
b A1 for shape and domain, A1 for zero at 1.50 (3 sf)

© Oxford University Press 2019 4


Mixed review

c = α 3r sin α ⇒ 2α − 3 sin=
l 1.5PQ ⇒ 2r= α 0 M1

So f (α ) =
2α − 3 sin α =
0 R1

From part b, the zero of f is 1.50, so α = 1.50 A1


d This is equivalent to values of α for which f ( x ) < 0 , R1

that is when 0 < α < 1.50 A1A1


1 2 1 2 
e  × 2 (1.496...) × r  −  r sin (2 × 1.496...)  =
4 A1
2  2 
Solve for r > 0 using GDC M1
r = 1.68 A1
π
(10a) + ( 8a) − 2 ⋅ 10a ⋅ 8a cos
2 2
8 a 122
= M1A1
3
12
84a2 = 144 ⇒ a2 = A1
7

12
a= AG
7

12 12
b
= = 13.1 3 s.f. and=
AB 10 AC 8= 10.5 A1A1
7 7
c Cosine rule gives
2
2  12  12 π
d ( x )  = x +  8 7  − 2 × 8 7 × x cos 3
2
M1
 

768 12
d ( x ) = x2 + −8 x A1
7 7
d

A1 for correct shape A1 for domain A1 for minimum point

© Oxford University Press 2019 5


Mixed review

π
sin ˆ
e i 3 = sin ADC R1
9.07 12
8
7
(OR shortest distance from the point C to the line AB meets at right angles)

ˆ π
So ADC= A1
2
ii AC + CD + =
AD 10.47... + 9.071... + 5.237...
= 24.8 cm M1A1
9 a 62 – 4(6) + 21 = 33 A1
b Completing the square, n2 – 4n + 21= (n – 2)2 + 17 M1A2
Since (n – 2) ≥ 0 for all n, (n –
2 2)2 + 17 > 0 for all n. R1
10 a A ( 0,26 ) and P ( −15,26 ) A1A1

b E.g. Consider a right-angled triangle PQA M1

=PQ 15
= sin θ 15 sin2.1t A1
h 26 − PQ
= M1

h (=
t ) 26 − 15 sin2.1t AG

A1 for shape and domain


A1 for maximum
A1 for minima
Period = time of full turn = 3.740 − 0.748 = 2.99 R1A1
1
Horizontal stretch with factor ; A1
2.1
Vertical stretch with factor 15; A1
Reflection in the x − axis ; A1
Vertical translation 26 units upwards A1

© Oxford University Press 2019 6


End of chapter test

13 Modelling change: more


calculus
Section A. A calculator is not allowed

1 Find ∫ sin5x cos 5x  dx


2 Find the value of a for the curve f ( x
= ) 5x + a sin x , which has a gradient of 7 at the point
π
where x = .
3

3x
3 Find f ′ (π ) when f ( x ) = .
cos x

4 Let f ( x  ) = 2e x   
cos x. Find the gradient of the normal to the curve where the curve crosses the y
axis.
π
3
sin x
5 Find ∫
0 cos x
 dx .

Section B. A calculator is allowed

6 a Given
= f ( x )       
e x ( cos x + sin x ) , find f ′ ( x )

b The curve has a maximum point between 0 and 3. Find the coordinates of the maximum
point.

c Sketch f (x) = ex (cos x + sin x) with −2 ≤ x ≤ 3 and shade the area bounded by the curve,
the positive x axis and the y axis.

d Write down an integral for the shaded area.

e Find the shaded area.

7 The velocity, v ms−1, of a particle at time t seconds, is given by

v ( t )   2
= t   
+ cos2t ,   for 0    2.
≤t ≤

Find

a The initial velocity of the particle.

b An expression for the acceleration of the particle.

c The time when the acceleration is zero.

d An integral to represent the distance travelled during the first second.

e Write down the distance travelled in the first second.

© Oxford University Press 2019 End of chapter test 1


End of chapter test

Answers

1 Let u = sin5x

du
= 5 cos 5x
dx

1 1 u2
5 ∫ u  du = ×
5 2
+c

1
∫ sin5x cos
= 5x  dx
10
sin2  3x + C

2 f ′ ( x )= 5 + acosx

π
5 + acos 7
=
3

1
a=2 a=4
2

3 cos x − (3x ) (− sin x )


3 f ′(x) =
cos2 x

3 cos x + 3x sin x
f ′(x) =
cos2 x

3 cos (π ) + 3 (π ) sin (π )
f ′ (π ) =
cos2 (π )

−3
f ′ (π ) = = −3
1

4 (
f ′ ( x ) = 2e  x × (–sin     ) (
x ) + cos x ×2e x  2 )
= e x     – e x   sin x = 2e x (cos x − sin x)
cos x 2

f ′ ( 0= (
) e0 cos (0) − sin (0)= 1 )
Gradient of the normal is −1

5 Let u   cos 
= x
du
= − sin x
dx

1
sin x − 1

∫ cos x
dx= ∫ −u 2 du = −2u 2 =
    2
− cosx

π
π 
π
( )
3
sin x 2
∫ dx =  − cosx  3 =  −2 cos  − −2 cos0 = 2 −
 2
cos x    3  2
0
0  

f ( x )   e x ( cos   sin 
6 a= x+ x)

f ′ ( x=)   
e x ( cos    x ) + e x (–sin   
x + sin    x)
x + cos 

= e x   cos  x
  2

© Oxford University Press 2019 2


End of chapter test

b 2e x cos x =  0

π
 cos   0
x =    
⇒ x=
2

π π
 π π 2
y   
= e 2  cos   
+ sin   
= e
 2 2

 π π2 
The coordinate of the maximum point is  , e  
2 

2.36
d ∫ e ( cos   sin x ) dx
x
x +
0

e 7.46 square units

7 a v (0) =
 2 ( 0 ) +  cos2 ( 0 ) =
1ms−1

b Acc ( t )= v ' ( t )=  2 − 2 sin2t

c 2 − 2 sin2t =
0

sin2t = 1

π
t =
4

1
d ∫ (2t + cos 2t ) dt
0

e 1.45m

© Oxford University Press 2019 3


Mixed review

13 Modelling change: more


calculus
1 A curve with equation y  f  x  passes through the point  0,3 . Its gradient function is
f   x   x  cos x . Find the equation of the curve.

2 The graph represents the function f  x   p cos  qx   r , for 0  x   , where p, q, r  .

a Find the values of p, of q, and of r.

b Find the area of the shaded region.

3 A particle moves along a line such that its displacement from a fixed origin O is given by
s t   2t  10ln t  1 , for t  0 .

a Find the velocity function for s.

b Find when the particle is moving left.

c Show that the velocity of the particle is always increasing.

4 A rectangular plot of land is enclosed by 180 m of fencing material on three sides. The fourth
side of the plot is bordered by a stone wall. Find the dimensions of the plot that encloses the
maximum area. Find the maximum area.

5 A particle’s velocity, in metres, is given by v t   0.4t cos t , for 0  t  6 , where t is time in


seconds. Use a GDC to answer the following.

a Find the displacement of the particle after 6 seconds.

b Find the distance the particle travels in 6 seconds.

200
6 Part of the graph of f  x   is shown.
1  30e0.4 x

a Find the range of f.

2400e0.4 x
b Show that f   x   .
1  30e 
2
0.4 x

c Use a GDC to find the maximum rate of change of f.

© Oxford University Press 2019 1


Mixed review

Exam-style questions

1 
7 The point A  , a  lies on the graph of the curve of y = sin(2x – 1) .
2 

a Determine the value of a . (2)

b Find the gradient of the tangent to the curve at any point x on the domain. (2)

c Hence find the equation of the normal to the curve at A. (3)

 
8 Consider the functions defined by f  x   e2 x and g  x   sin  x   .
 3

a Find expressions for

i f  x

ii g  x  . (2)

Consider the function h  x   f  x   g  x  .

 
b Show that h    0 . (3)
3

9 The diagram shows an isosceles triangle with one of its vertices at the origin O and AB parallel
to the x  axis. Points A and B both lie on the curve y  25  3x2 .

Let the x-coordinate of B be x.

a State the coordinates of A and B in terms of x . (2)

b Show that an expression for the area of the triangle ABC is given by 25x  3x3 ,
x  0. (1)

c Hence find the largest area that the triangle can have. Give your answer in exact form.
(4)

d For the triangle with largest area, determine the following correct to three significant
figures:

i its perimeter

ii the size of its internal angles in degrees. (6)

10 The diagram shows the graph of the function f defined by f  x   3  x2 for 2  x  2 .

© Oxford University Press 2019 2


Mixed review

Consider the function g defined by g  x   sin ex   for 2  x  2 .

a On the same axes, sketch the graphs of f and g for 2  x  2 . (2)

b Solve f  x   g  x  in this interval. (2)

c Hence state the solutions to f  x   g  x  in this interval. (1)

d Find the area of the region enclosed by the graphs of f and g . (2)

1
11 Consider the function f defined by f  x   for   x   , x  0 .
sin x

a Use quotient rule to find an expression for f   x  , for   x   , x  0 . (2)

1  cos2 x
b Show that f   x   . (3)
sin3 x

c Hence show that the graph has exactly one maximum and one minimum point in the
domain given. (4)

d State the equations of the vertical asymptotes to the graph of f . (2)

e Hence sketch the graph of f . (3)

© Oxford University Press 2019 3


Mixed review

Answers

1 2
1 f x  x  sin x  3
2

2 a p  3; q  2; r  3

b 3

10
3 a v t   2 
t 1

b 0t 4

10
c v  t   a t   which is always greater than 0, therefore velocity is always increasing.
t  1
2

4 The dimensions are 45 m by 90 m with a maximum area of 4050 m2.

5 a -0.687 m

b 4.34 m

6 a 0  f  x   200

200
 
1
b f x   200 1  30e0.4 x
1  30e0.4 x
0.4 x
2400e0.4 x
   0.4  30e   200  12e
2
f   x   1  200 1  30e0.4 x 0.4 x

1  30e  1  30e 
2 2
0.4 x 0.4 x

c 8.50

 1 
7 a a  sin  2   1  sin0  0 M1A1
 2 

b y  x   2 cos 2x  1 M1A1

1
c y     2 cos 0  2 A1
2

1
m M1
2

1 1
y  x   A1
2 2

8 a i f   x   2e2 x A1

 
ii g  x   cos  x   . A1
 3

         
b h    f    g    f   g   M1
3
  3 3 3 3

2 2
 2e 3 sin0  e 3 cos 0 A1

2
 e3  0 A1AG

© Oxford University Press 2019 4


Mixed review

9 a  
A  x,25  3x2 , B x,25  3x2   A1A1

b f x 

2x 25  3x 2   25x  3x , x  0
3
M1AG
2

c f   x   25  9x2 A1

5
f  x  0  x   M1A1
3

 5  125 125 250


f    A1
3 3 9 9

2 2
5  50 
d i OA  OB       16.749... A1
3  3 

5
perimeter  2   2  16.749...  36.8 M1A1
3

5
^
1
ii A O B  2 arctan 50
3
 2 arctan  11.4º M1A1
3
10

^ ^
180  11.4212..
O AB  ABO   84.3º A1
2

10 a A1 for shape of g, A1 for domain of g

b f  x   g  x   x  1.68, x  1.80 A1A1

c 1.68  x  1.80 A1

1.799...
d   f  x   g  x 
1.678...
dx  5.68 M1A1A1

© Oxford University Press 2019 5


Mixed review

1  sin x  1   sin x  cos x


11 a f  x  2
 M1A1
sin x sin2 x

  cos x   sin2 x  cos x   sin2 x 


b f   x   M1
sin x 
2
2

sin x  sin2 x  2 sin x cos2 x


 A1A1
sin4 x

sin2 x  2 cos2 x 1  cos2 x


  A1AG
sin3 x sin3 x

cos x
c f  x  0    0  cos x  0 M1
sin2 x


x  A1
2

  1  0 
f      0  f has a minimum at R1
2
  1 2

   10 
f       0  f has a maximum at  R1
 2 1 2

d Vertical asymptotes: x   , x  0 A1A1

e A1A1 - for each branch; A1 for domain and asymptotes

© Oxford University Press 2019 6


End of chapter test

14 Valid comparisons and informed


decisions: probability distributions
Section A. A calculator is not allowed

1 The random variable, X, has the probability distribution

x 1 2 3 4 5

P(X = x) 0.05 0.4 a 0.2 0.1

a Find the value of a

b Find P(X > 3)

2 The discrete random variable, X, can only take the values 0,1, 2, 3 and
x −1
1
P (= ) k  2 
X x= .
 

Find the value of k.

3 A standard die (the singular of dice) is thrown and the score is squared.

a Copy and complete the table

x 1 4 9 25 36

P(X = x) 1 1 1 1 1
6 6 6 6 6

b Find the expected score.

4 A fund raising game at the school fair sells lottery tickets for $4. The possible prizes are $4, $5,
$10, $100 or you could not win anything. W is the amount won and is represented in this
distribution.

w 0 4 5 10 100

P(W = w) k 0.2 0.2 0.1 0.002

a Find the value of k

b Find how much money the fundraiser would expect to make if it sold 500 tickets.

Section B. A calculator is allowed

5 Find the probability of obtaining exactly 3 heads in 5 tosses of a fair coin.

6 The probability that Blake scores from a penalty in hockey is 0.8 .

Blake takes 6 penalties, what is the probability that Blake,

a does not score on his first shot

b scores exactly 3

c scores at least once.

© Oxford University Press 2019 End of chapter test 1


End of chapter test

7 In a baseball game, the probability that Eden will get on base safely when she comes to bat is
0.6 .

a What is the probability that she will get on base safely at least 3 out of 4 times?

If Eden gets to bat 50 times in a season

b What is the expected number of times that she will make first base?

c What is the variance of her number of bases?

8 The weights of Tuna sent for sushi follow a normal distribution with a mean of 150kg and a
standard deviation of 5kg.

a Find the probability that a tuna weighs less than 142kg.

b Fuji restaurant only takes tuna from the top 20%. How heavy is their lightest tuna?

9 The rice harvest in Thaibodia yields an average of 341kg per farmer and is normally distributed
with a standard deviation of 57kg.

It is known that 2.5% of the farmers had a weak season. Find how many kg of rice make a
weak season.

10 On a mathematics test, the scores were normally distributed with a mean of 74 and a standard
deviation of 7.

a Sketch this on a normal curve and shade the proportion of the class would be expected to
score between 60 and 80 points.

b Find the shaded area and state the percentage of the class expected to score between 60
and 80.

11 After Sage sprays the garden with a weed killer, the survival time of weeds in a field is
normally distributed where 15 % of the weeds survive greater than 9 days and 12 % of them
last less than 4 days. Find the mean and standard deviation for the survival time of the weeds
in sage’s garden.

12 A pack of coffee is sold as weighing 230gms but the manufacturer maintains a weight of 231
gms as the average so as not to cheat customers. Coffee packs are normally distributed with a
mean of 231 grams and a standard deviation of 1.5 grams.

A pack of coffee is considered to be underweight if it weighs less than 228 grams.

a What is the probability that a pack of coffee is underweight?

The manufacturer decides that the probability of a pack being underweight must be reduced to
0.002. He gives this problem to two junior executives, Eden and Haven

b Eden’s suggestion is to increase the mean and leave the standard deviation unchanged. Find
the value of the new mean.

c Haven’s suggestion is to reduce the standard deviation and leave the mean unchanged. Find
the value of the new standard deviation.

d After the probability of a pack of coffee being underweight has been reduced to 0.002, the
store sells 100 packs. Find the probability that at least two of the boxes are underweight.

© Oxford University Press 2019 2


End of chapter test

Answers

1 a 1 − ( 0.05 + 0.4 + 0.2 + 0.1) =


0.25

b 0.2 + 0.1 =
0.3

0 −1
1
2 P (X ) k  2  = 2k .
= 0=
 

1−1
1
P (X ) k  2  = k .
= 1=
 

2 −1
1 1
P (= ) k  2  = 2 k .
X x=
 

3 −1
1 1
P (= ) k  2  = 4 k .
X x=
 

1 1
2k + k + k+ k =
1
2 4

15
k =1
4

4
k =
15

3 a x 1 4 9 16 25 36

P(X = x) 1 1 1 1 1 1
6 6 6 6 6 6

 1  1  1  1  1  1  91 1
b E ( X ) = 1 ×  +  4 ×  +  9 ×  + 16 ×  +  25 ×  +  36 ×  = =15
 6   6   6   6   6   6  6 6

4 a k =1 − ( 0.2 + 0.2 + 0.1 + 0.002 ) =0.498

b E (W ) =(0 × 0.498) + ( 4 × 0.2) + (5 × 0.2) + (10 × 0.1) + (100 × 0.002) =$3


Profit = 500 × $1 = $500

5
P (3heads in5 tosses ) =  ( 0.5) ( 0.5)
3 2
5 = 0.3125
3
 

6
  ( 0.8 ) ( 0.2 ) = 0.08192 1 − ( 0.2 ) =
3 3 6
6 a 1 − 0.8 =
0.2 b c 0.999936
3

7 a P ( On base safely in at least 3 out of 4 at bats
= ) P (3) + P ( 4 )
 4  4
=   ( 0.6 ) ( 0.4 ) +   ( 0.6 ) ( 0.4 ) = 0.4752
3 1 4 0

3  4

b 50 × 0.6 =
30 c Var = npq =50 × 0.6 × 0.4 =12

8 a P (W < 142 ) =
0.0548

b P (V > w=
) 0.2, 154.2
x
= . Their lightest tuna weighs 154.2kg

© Oxford University Press 2019 3


End of chapter test

9 Using the GDC for inverse normal calculations, z = −1.96

x−µ x − 341
−1.96 = =
σ 57

x = 229kg

10 a

b Using the GDC P ( 60 ≤ s ≤ 80 ) =


78.2%

11 P( X   9)  0.15 
> = and P( X   4)  0.12
< =
Find the z scores from the GDC as 1 .036 and  –1.175
9−µ 4−µ
1.04 = , 1.18
− =
σ σ

Solve simultaneously to give


μ = 6.66, σ = 2.26

12 a (   .52
X  ~ N 231,1 )
P(X < 228) = 0.0228

b X ~ N(μ, 1.52 )
P(X < 228) = 0.002

228 − µ
−2.88 = .
1.5
µ   232 
= grams

c (
X  ~ N 231, σ 2 )
228 − 231
−2.88 = .
σ
σ  1
=  .04 grams

d X   ~  B (100, 0.002 )
P ( X   ≤1)=
    0.982...
P ( X=
≤ )  1
  2 P ( X=
   –   ≤1) 0.0174
   

The probability that 2 packs are underweight is 0.0174

© Oxford University Press 2019 4


Mixed review

14 Valid comparisons and informed


decisions: probability distributions
1 Consider a random variable X  B(n, p) .

n − x +1 p
Show that P( X =x) = × × P( X =x − 1) .
x 1− p

3xi − 2
2 We consider X a discrete random variable such that P(=
X x=
i) =, i 1,2,3, 4 .
22

a Complete the table:

xi

P(X = xi)

b Verify that P( X = xi ) is a probability function.

c Find E( X ) and Var ( X ) , giving your answers in simplified fractions.

3 A factory supervisor checks some machines to check if they work efficiently. If a machine works
efficiently, this machine “good”, if it is “bad”. The probability that a machine is “bad” is 0.1.

The supervisor checks a sample of four machines. Find the probability that:

a one machine is “bad”

b one machine is “good”

c at least one machine is “bad”.

4 A sample of four flashlights are chosen at random. Let X be a random variable that counts the
number of defunct flashlights in the sample. It is given that the random variable follows the
binomial distribution. The probability of picking zero defunct flashlights equals the probability
that all four are defunct.

a Find the probability that one flashlight will be defunct.

b Work out the probability that I will get 3 defunct flashlights out of the 4.

c Calculate E( X ) and Var ( X ) .

5 a The weight of each pupil in a school is measured. We consider as X the random variable
that measures each weight. If the random variable X follows the normal distribution with
mean µ = 60kg and variance σ 2 = 25kg2 , work out the probability P( X ≥ 70) .

b Let X be the time of waiting in a doctor’s office. We assume that X  N(12.56, 3.752 ) .
Work out the probability P(10 < X < 15) .

© Oxford University Press 2019 1


Mixed review

Exam-style questions

6 Find the value of each of the following, giving your answer as an integer:

1
a log10 (1)
100

b log10 25 + 2log10 2 (2)

c log9 (3log10 5 + 3log10 2 ) (3)

2
f (x)
7 Consider the rational function defined by = + q , where p, q ∈ , x ∈  .
x−p

a Write down the equations of the vertical and horizontal asymptotes of the graph of f in
terms of p and q . (2)

The domain of f is x ≠ 1 and its range is y ≠ 2 .

b Find the value of

i p

ii q (2)

c The point A (3, a) lies on the graph of f . Without finding an expression for the inverse of
f , show that A also lies on the graph of f −1 . (3)

d Find an expression for f −1 ( x ) . (4)

8 The graph of a quadratic function has y -intercept 4 and one of its zeros is 2.

Given that f ( x ) = ax 2 + bx + c , where a, b, c ∈ , x ∈  ,

a State the value of c . (1)

The equation of the axis of symmetry is x = 3 .

b Justify that a > 0 . (3)

c State the value of the other zero of f , and give a reason for your answer. (1)

d Hence find the value of each parameter a and b . (4)

2x
9 Consider the strictly increasing function defined by f ( x ) = , x ≥0.
x2 + 1

a Show that f ′ ( x ) =
(
2 1 − x2 ), x ≥0. (4)
(x )
2
2
+1

b Explain why the range of f is the interval 0,1 (4)

2x
c Find ∫x 2
+1
dx (2)

d Hence find the area enclosed by the graph of f and the x − axis between 0 and 1. (3)

© Oxford University Press 2019 2


Mixed review

) A + B sin (Cx ) for real constants A, B and C, and


10 The diagram shows part of the graph of f ( x=
x in radians.

a State the range of the function. (1)

b Using your answer to part a, justify that A = 1 . (1)

c Given that B > 0 , show that B = 4 . (2)

d Justify that the function has period π . (2)

e Hence, find the value of C and justify your answer. (1)

Let g(x ) = sin x .

f The graph of g can be transformed onto the graph of f .

Describe the sequence of transformations required to obtain the graph of f from the graph
of g , stating clearly the order in which they must be performed. (3)

© Oxford University Press 2019 3


Mixed review

Answers
n
1 P(= )   p x (1 − p)n − x since the random variable follows the binomial distribution.
X x=
x
 n  x −1  n  x −1
=P( X=
x − 1)  − p)n −( x −1) 
 p (1 =  p (1 − p)
n − x +1
.
 x − 1  x − 1
n − x +1 p  n  x −1 n − x +1
× ×  × p × (1 − p)
x 1 − p  x − 1
n − x +1 p n!
= × × × p x −1 × (1 − p)n − x +1
x 1 − p (x − 1)!(n − x + 1)!
(n − x + 1)n !
= × p x × (1 − p)n − x
x(x − 1)!(n − x + 1)!
n!
= × p x × (1 − p)n − x
x !(n − x )!
n
=   × p x × (1 − p)n − x
x
= P=( X x)
2 a Apply the various values of i to P( X = xi ) to get the results of the table.
3×1 − 2 1 3×2 −2 4
P( X
= 1)
= = P( X
= 2)
= =
22 22 22 22
3×3 −2 7 3 × 4 − 2 10
P( X
= 3)
= = P( X
= 4)
= =
22 22 22 22
xi 1 2 3 4
1 4 7 10
P ( X = xi )
22 22 22 22
1 4 7 10
b Since, P( X =
1) + P( X =2) + P( X =3) + P( X =4) = + + + 1,
=
22 22 22 22
then this is a probability function.
4
c=
E( X ) ∑
= x P( X
i =1
i xi ) and \ Var
= ( X ) E( X 2 ) − (E( X ))2

1 4 7 10 35
E( X ) =1× +2× + 3× + 4× = .
22 22 22 22 11
4
1 4 7 10 120
Also, E( X 2 ) =∑ xi2P( X =xi ) =1× + 4× +9× + 16 × = .
i =1 22 22 22 22 11
120 35 95
Therefore, Var ( X ) = − ( )2 = .
11 11 121
3 X  B(4, 0.1) .

 4 729  4 9
a P( X =1) =  × 0.11 × 0.94 −1 = . b P( X =3) =  × 0.13 × 0.94 −3 = .
1
  2500 3
  2500

 4 3439
c P( X ≥ 1) =1 − P( X < 1) =1 − P( X =0) .Therefore, P( X ≥ 1) =1 −   × 0.10 × 0.94 −0 = .
0
  10000
4 a P( X
= 0)
= P( X
= 4)

 4 0 4 −0  4 1
  p (1 − p) =   p4 (1 − p)4 − 4 ⇒ (1 − p)4 = p4 ⇒ 1 − p = p ⇒ p =
0
  4
  2

4 1 1
b P(= )   ( )x (1 − )4 − x
X x=
x
  2 2

 4 1 3 1 4 −3 1
Hence, P( X =
3) =
  ( ) (1 − ) =.
3
  2 2 4

© Oxford University Press 2019 4


Mixed review

c E( X ) = np and Var=
( X ) np(1 − p) .
1 1 1
Hence, E( X ) = 4 × = 2 and Var ( X ) = 4 × × (1 − ) = 1 .
2 2 2
X −µ
5 a Use standardization Z = and obtain that Z  N(0, 1) .
σ
70 − µ
P( X ≥ 70) =−
1 P( X < 70) =−
1 P (Z < )
σ
70 − 60
1 − P (Z <
= )=
1 − P(Z < 2) =
1 − 0.977 =
0.023
5
X −µ
b Use standardization Z = and obtain that Z  N(0, 1) .
σ
10 − 12.56 15 − 12.56
P(10 =
< X < 15) P( <Z < )
3.75 3.75
= P(−0.68 < Z < 0.65)
P(Z > −0.68) = 1 − P(Z < 0.68)
P(−0.68 < Z < 0.65) =P(Z < 0.65) − P(Z > −0.68) =P(Z < 0.65) − (1 − P(Z < 0.68))
Therefore: P(−0.68 < Z < 0.65)
= P(Z < 0.65) + P(Z < 0.68) − 1
= 0.7422 + 0.7517 − 1
= 0.4939
1
6 a log10 = −2 A1
100
b log10 25 + 2log=
10 2 log10 25 × =(
22 log10 100
= 2 ) M1A1

c log9 (3log10 5 + 3log10=


2 ) log9 (log10 125 + log10 8 ) M1

= log
= 9 (log10 1000 ) log9 3 A1

1
= A1
2
7 a=
x p=
,y q A1A1
b i p =1 A1
ii q=2 A1
2
c = )
a f (3= 2 3
+= M1A1
3 −1
A (3,3) lies on y = x , and since graphs of f and f −1 are reflections in the line y = x , hence
A also lies on the graph of f −1 R1AG
2
d=x +2 M1
y −1
Solve for y M1
2
=y +1 A1
x −2
2
1
f −= (x) x −2
+1 A1

8 a c =4 A1
b A quadratic function has only one turning point at the axis of symmetry. So f ( x ) has a
turning point at x = 3 R1
As the function decreases from x = 0 (where y = 4) to x = 2 (where y = 0), it must also
decrease until it reaches the turning point at x = 3. R1
So the turning point at x = 3 is a minimum, and hence a > 0 R1
c x=4 A1
since the curve is symmetrical about x = 3 R1

© Oxford University Press 2019 5


Mixed review

d f (2 ) = 0 ⇒ 4a + 2b + 4 = 0 and f ( 4 ) = 0 ⇒ 16a + 4b + 4 = 0 A1

Solve simultaneously M1
1
a= and b = −3 A1A1
2

(2x )′ ( x 2 + 1) − 2x ( x 2 + 1)′ ( )
2 x 2 + 1 − 2x ⋅ 2x
=9 a f ′(x) = M1A1A1
( ) (x )
2 2
x2 + 1 2
+1

=
2 − 2x 2
=
2 1 − x2 ( ) A1
( ) ( )
2 2
x2 + 1 x2 + 1

b f ( 0 ) = 0 and lim f ( x ) = 0 R1
x →∞

f ′(x) = 0 ⇒ x =1 A1

Since f ( x ) > 0 for x > 0 , so f has a maximum at x = 1 and f (1) = 1 A1

As f is strictly increasing, f ′ ( x ) > 0 ⇒ x < 1 R1

Therefore, the range of f is the interval 0,1 AG

2x
c By inspection, ∫x 2
+1
(
x ln x 2 + 1 + C
d= ) M1A1

1
2x
( )
1
d ∫x dx= ln x 2 + 1  = ln2 M1A1A1
2
+1  0
0

10 a −3 ≤ y ≤ 5 A1
5 + ( −3)
b A
= = 1 R1AG
2
5 − ( −3)
c B
= = 4 M1
2
As B > 0, so B =
4 R1AG
3π π π
d The difference between the x − coordinates of the maximum and minimum is − =.
4 4 2
M1
π
Therefore the period is 2 × π
= R1AG
2

e C = M1
period of f

C
So = = 2 A1
π
1
f Vertical stretch by factor 4 and horizontal stretch by factor (in any order) A1A1
2
Followed by a vertical translation of 1 unit upwards. A1

© Oxford University Press 2019 6


Exam practice

Exam practice: chapters 1 – 14

1 P2: In a geometric series, the second term is 24 and the fifth term is 1.536 .

a Find the common ratio for the series. (5)

b Find the first term in the series. (2)

c Find the sum to infinity of the series. (2)

2 P1: Triangle ABC has sides =


AB 4 2 − 3 , =
AC 4 2 + 3 and ∠BAC =
60

a Find the exact area of the triangle. (3)

b Find the exact length of BC . (4)

3 P2: A study was conducted to determine if there was any correlation between a person’s age
( x ) and their reaction time (T ) . A number of people were tested, and the mean reaction
time calculated for each age is shown in the table below.

Age x (to the


10 20 30 40 60 70 80
nearest 10 years)

Mean reaction
0.125 0.148 0.166 0.221 0.231 0.270 0.341
time (T seconds)

a Find Pearson’s product-moment correlation coefficient ( r ) for this data. (2)

b Use your result from part (a) to describe the strength of the correlation between x
and T. (1)

c Find the equation of the regression line of T on x . (2)

d Using the above data, determine an estimate for the reaction time of a 50 -year-old.
(2)

e Explain why would it be unwise to use your answer to part c) to determine the reaction
time of a 90-year-old. (1)

4 P1: Find the term in x 3 in the binomial expansion of (2 − x ) (3 + x )


5
(5)

© Oxford University Press 2019 1


Exam practice

5 P1: Sanju invests $500 in a savings account earning compound interest at a rate of 2.9%

At the same time, Pranav invests $800 in a different account, earning compound interest
at a rate of 2% . After N years, both Sanju and Pranav will have the same amount in
their respective accounts.

a Show that 1.075N = 1.6 (3)

b Hence find the least number of complete years they must wait for Sanju to have more
money that Pranav. (4)

6 P1: Two functions are given by

5x − 3
f (x) = , x ≠ −3 , x ∈  and g ( x ) = x , x >0 , x∈.
2x + 6
a Write down the equation of the vertical asymptote of the graph of y = f ( x ) . (1)
b Write down the equation of the horizontal asymptote of the graph of y = f ( x ) . (1)

c Sketch the graph of y = f ( x ) . On your sketch, mark the horizontal and vertical
asymptotes as dashed lines. (3)

d Write down the range of the function f ( x ) . (1)

e Write down an expression for the function g  f ( x ) . (1)

f Find the domain of g  f ( x ) . (2)

7 P1: Points O, A and B are given by the coordinates O ( 0, 0 ) , A ( −12,18 ) and B (18, −2 ) .
a Find the equation of the line AB , giving your answer in the form ax + by + c =
0 where
a, b and c are integers. (4)

b Hence find the area of the triangle bounded by the line AB and the coordinate axes.
(3)

8 P1: a Solve the inequality ( x + 4 ) (3 − x ) > 0 . (3)


b Solve the inequality 2 x 2 − 11x + 9 < 0 . (4)

c Hence, solve simultaneously the inequalities ( x + 4 ) (3 − x ) > 0 and


2 x 2 − 11x + 9 < 0 . (1)

9 P1: A quadratic function y = ax 2 + bx + c intersects the points with coordinates ( 0,18 ) ,

( −1,27) and (3,3) .


a Find the values of a, b and c . (5)

b Use the discriminant and your answer to part (a) to determine the number of solutions
to the equation ax 2 + bx + c =
0. (3)

c Express y in the form y = p ( x − q ) + r and hence write down the coordinates of the
2

vertex of the graph. (3)

45
10 P1: Solve the equation log
= 10 x − 12 . Give your answers as powers of 10. (5)
log10 x

© Oxford University Press 2019 2


Exam practice

2x + x
11 P1: Find the equation of the normal to the curve y = at the point on the curve
x x
where x = 4 .

0 where a, b, c are integers.


Give your answer in the form ax + by + c = (8)

12 P1: The line y = k is a tangent to the curve


= y e x (1 − x ) .

Determine the value of k . (6)


a
10
13 P1: Given ∫ 5x + 4 dx = 4ln2 , ( a > 2)
2
, find an exact value for a . (7)

14 P1: Solve the equation cos


= 2θ 3 cos θ − 2 , giving all solutions in the range 0 ≤ θ < 2π .
(7)

15 P2: Alison walks to school every day. The time she takes to walk to school is modelled as a
normal distribution, with mean 36 minutes and standard deviation 3.12 minutes.

a Find the probability that on any randomly selected day, Alison’s journey to school takes
longer than 40 minutes. (2)

b Find the probability that on any randomly selected day, Alison’s journey takes between
34 and 38 minutes. (4)

c If the probability that Alison walks for longer than M minutes is 0.015, find the value
of M . (3)

d Given that Alison walks to school on 195 days of the year, find the number of days on
which she can expect to reach school in under 30 minutes. (4)

16 P2: Approximately 4% of eggs produced and sold by a local farm are cracked.

Jerry buys 24 eggs from the farm.

a Find the probability that exactly two of Jerry’s eggs are cracked. (2)

b Find the probability that Jerry buys no more than four cracked eggs. (2)

c Find the probability that Jerry buys at least two cracked eggs . (2)

d Find the variance of the number of cracked eggs. (2)

17 P1: Two events A and B are independent. It is given that P ( A ) = 0.3 and P ( B ) = 0.8 .

a State, with a reason, whether events A and B are mutually exclusive. (2)

b Find the probability of the event:

i A∩B

ii A∪B

iii A | B′

iv A′ ∩ B (8)
π
3
sin2 x
18 P1: By using the substitution u= 1 + cos 2 x , show that ∫ 1 + cos 2x dx
0
= ln2 . (8)

© Oxford University Press 2019 3


Exam practice

19 P1: Consider the functions f ( x=


) x 2 − 25 ( x ≤ 0, x ∈  ) and g ( x =
) 4−x ( x ∈ ) .
a Find an expression for g −1 ( x ) , stating clearly the domain. (2)

b Find an expression for f −1 ( x ) , stating clearly the domain. (5)

c Solve the equation f  g ( x ) = 0 . (4)

20 P2: Over the course of a single December day in Limassol, Cyprus, the highest temperature
was found to be 22 C. The lowest temperature was 12 C, which occurred at 0300 hours.

If t is the number of hours since midnight, the temperature T may be modelled by the
equation
= ( )
T A sin B ( t − C ) + D .

a Find the values of A, B, C and D . (9)

b Hence, by using technology, find for how many hours the daily temperature is above
20 C. (4)

21 P1: The first four terms in an arithmetic series are given by

log2 343, log2 x, log2 y , log2 1331

Find the value of x (9)

22 P1: The velocity, v m s-1, of an object moving along a horizontal line at time t seconds is
given by
= v cos t − sin t where t is in radians, and 0 ≤ t ≤ π .

a Find the maximum speed of the object. Give your answer in exact form. (7)

b Find the displacement of the object after π seconds. (4)

c Find the exact value of the total distance travelled by the object after π seconds.

(7)

© Oxford University Press 2019 4


Exam practice

Answers
1 a ar = 24 A1
4
ar = 1.536 A1
Attempt to solve simultaneously M1
ar 4 1.536
=
ar 24
r 3 = 0.064 A1
r = 0.4 A1
24 24
b =a = = 60 M1A1
r 0.4
a
c S∞ = M1
1−r
60 60
= = = 100 A1
1 − 0.4 0.6
1
2 a Attempt to use bc sin A for area M1
2
1
( ) (
Area =× 4 2 − 3 × 4 2 + 3 × sin60
2
) A1

1 3
= × 23 ×
2 2

23 3
= A1
4
b Attempt to use cosine rule M1

(4 ) + (4 ) ( )( )
2 2
AC 2= 2 −3 2 +3 − 2 4 2 − 3 4 2 + 3 cos 60 A1

1
= 32 + 9 − 24 2 + 32 + 9 + 24 2 − 2 × 23 × A1
2
= 82 − 23 = 59

So AC = 59 A1
3 a Use of GDC to give M1
r = 0.9675 A1
b This is a strong positive correlation. R1
c Use of GDC to=
give: T 0.00277 x + 0.0919 M1A1
d =T 0.00277 × 50 + 0.0919 = 0.230 seconds M1A1
e 90 years lies outside the range of data we are given, which would therefore involve
extrapolation of data. R1
5 5
4 We require 2 ×   32 x 3 + ( − x ) ×   33 x 2 M1A1A1
2
  2
= 180 x 3 − 270 x 3 A1

= −90x 3 A1
5 a 500 × 1.029 = N
800 × 1.02 N
M1A1
N
1.029 800
N
= A1
1.02 500
N
 1.029  8
  =
 1.02  5

1.00882N = 1.6

© Oxford University Press 2019 5


Exam practice

b Attempts values for N (or use of logarithms) M1


1.0756 = 1.54 A1

1.0757 = 1.66 A1
So they must wait 7 years A1
6 a x = −3 A1
5
b y = A1
2
c A1 left-hand branch; A1 right-hand branch; A1 asymptotes

5
d f (x) ≠
2
(
, f (x) ∈  ) A1

5x − 3
e g  f (x) = A1
2x + 6
3
f x < −3 , x ≥ , ( x ∈ ) A1A1
5
18 − ( −2 ) 20 2
7 a m= = = − M1A1
−12 − 18 −30 3
2
Equation is y + 2 =−
3
( x − 18) M1A1

3y + 6 =−2 x + 36
2 x + 3y − 30 =
0 A1
b AB crosses axes at (15, 0 ) and ( 0,10 ) A1

1
Area is therefore × 15 × 10 =
75 units2. M1A1
2
8 a Using sketch of y =( x + 4 ) (3 − x ) M1

Chooses ‘inside values’ M1


Solution is −4 < x < 3 A1

b 2 x 2 − 11x + 9 < 0

(2x − 9) ( x − 1) < 0 M1

Using sketch of y =(2 x − 9 ) ( x − 1) M1

Chooses ‘inside values’ M1


9
Solution is 1 < x < A1
2
c Comparing answers from a and b gives: 1 < x < 3 A1

© Oxford University Press 2019 6


Exam practice

9 a c = 18 A1
27 = a − b + 18 M1
3 = 9a + 3b + 18
Attempt to solve simultaneously M1
a = 1 , b = −8 A1A1

b2 − 4ac =( −8 ) − 4 × 1 × 18 =−8
2
b M1A1

< 0 therefore no solutions R1


c y = x 2 − 8 x + 18

= ( x − 4 ) − 16 + 18
2
M1

=( x − 4) + 2
2
A1

Therefore vertex is at the point ( 4,2 ) A1

10 (log10 x=
) 45 − 12log10 x
2
M1

(log10 x )
2
+ 12log10 x − 45 =
0

(log10 x − 3) (log10 x + 15) =


0 (or other valid method to solve the quadratic) M1A1

log10 x − 3 =0 or log10 x + 15 =
0

log10 x = 3 log10 x = −15

x = 103 x = 10−15 A1
2 1
11=
y +
x x

5
x =4⇒y = A1
4
3
dy −
− x 2 − x −2
= M1A1
dx
dy 1 1 3
At x = 4 , =− − =− A1
dx 8 16 16
16
Required gradient is therefore M1
3
5 16
Equation is y − =
4 3
( x − 4) M1A1

So 64 x − 12y − 241 =
0 A1

12 y = k must intersect
= y e (1 − x ) at a maximum or minimum point.
x
R1

dy
=−e x + (1 − x ) e x =− xe x M1A1
dx
dy
=0 ⇒ x =0 M1A1
dx
At x = 0 , y = 1 A1
So k = 1

© Oxford University Press 2019 7


Exam practice

a
10 a
13 ∫ 5x=
2 +4
dx 2ln 5x + 4 
2
M1A1

= 2ln (5a + 4 ) − 2ln14 M1

 5a + 4 
= 2ln   M1
 14 
4ln2 = 2ln 4 A1
5a + 4
=4 M1
14
52
a= A1
5
14 2 cos2 θ=
− 1 3 cos θ − 2 M1

2 cos2 θ − 3 cos θ + 1 =0

(2 cos θ − 1) ( cos θ − 1) =
0 M1A1

1
cos θ = or cos θ = 1 A1
2
1 π 5π
cos θ = ⇒ θ = ,θ = A1A1
2 3 3
cos θ = 1 ⇒ θ = 0 A1

15 a (
X ~ N 36,3.122 )
 40 − 36 
P ( X > 40 ) = P  Z >  M1
 3.12 
= 0.1 A1
 34 − 36 38 − 36 
b < 38 ) P 
P (34 < X= <Z <  M1
 3.12 3.12 

= P ( −0.641 < Z < 0.641) A1

= P ( Z < 0.641) − P ( Z < −0.641) M1

= 0.739 − 0.261
= 0.478 A1
 M − 36 
c P Z > =0.015 M1
 3.12 

 M − 36 
P Z < =0.985
 3.12 

M − 36
= 2.170 A1
3.12
⇒M =
42.77
⇒M = 42 minutes, 46 seconds A1
 30 − 36 
d P ( X < 30 ) = P  Z <  M1
 3.12 

= P ( Z < −1.923)

= 0.027 A1
195 × 0.027 =
5.3 M1
Therefore the expected number of days is 5 . A1

© Oxford University Press 2019 8


Exam practice

16 a Let X represent the number of cracked eggs Jerry buys.


X ~ B (24, 0.04 )

P (X ) C2 ( 0.04 ) ( 0.96 )
2 22
= 2= 24
or uses GDC directly M1

= 0.180 A1
b P ( X ≤ 4) =
0.998 M1A1
c P ( X ≥ 2) =
0.249 M1A1
d ( X ) np (1 − p )
Var= M1

=24 × 0.04 × 0.96


= 0.9216 A1
17 a If they were mutually exclusive, then P ( A ∩ B ) =
0 , A1

but since they are independent, we have P ( A


= ∩ B ) P ( A) P ( B ) ≠ 0 .

Therefore we have a contradiction, and so A and B are not mutually exclusive. R1


b i P ( A ∩ B ) = P ( A ) P ( B ) = 0.3 × 0.8 = 0.24 M1A1

ii P ( A ∪ B ) = P ( A ) + P ( B ) − P ( A ∩ B ) = 0.3 + 0.8 − 0.24 = 0.86 M1A1

P ( A ∩ B′ ) P ( A ) − P ( A ∩ B ) 0.3 − 0.24 0.06


iii P ( A=
| B′ ) = = = = 0.3 M1A1
P ( B′ ) P ( B′ ) 0.2 0.2

iv P ( A′ ∩ B ) = P ( B ) − P ( A ∩ B ) = 0.8 − 0.24 = 0.56 M1A1

du
18 = −2 sin2 x A1
dx
π 1
x = ⇒u= and x = 0 ⇒ u = 2 A1
3 2
π 1
3
sin2 x 1 2 du
∫0 1 + cos 2x d x = −
2 ∫2 u
M1A1

1 1

= − ln u 22 A1
2
1 1 
−  ln − ln2 
= M1
2 2 
1
=−
2
( − ln2 − ln2) A1

1
=
2
(2ln2) A1

= ln2
19 a g −1 ( x =
) 4−x, x∈ A1A1
b y x − 25
= 2

Attempt to make x the subject M1


x2
y + 25 =

x =
± y + 25 A1

Since domain of f is x ≤ 0 , it follows that range of f −1 ( x ) is y ≤ 0 (R1)

and hence f −1 ( x ) =
− x + 25 , x ≥ −25 ( x ∈ ) A1A1

© Oxford University Press 2019 9


Exam practice

f  g ( x ) =( 4 − x ) − 25
2
c M1

Attempt to solve ( 4 − x ) − 25 =
2
0 M1

4−x =±5
Since domain of f is x ≤ 0 , it follows that domain of f  g ( x ) is 4 − x ≤ 0 R1
Hence 4 − x =−5 ⇒x =
9 A1
22 + 12
20 a D= = 17 M1A1
2
22 − 12
A= =5 M1A1
2
360
The period is = 24 M1
B
Therefore B = 15 A1

So
= (
T 5 sin 15 ( t − C ) + 17 )
At (3,12 ) = (
, 12 5 sin 15 ( t − C ) + 17 ) M1

= (
−1 sin 15 (3 − C ) )
15 (3 − C ) =
−90 A1

C =9 A1
Therefore
= (
T 5 sin 15 ( t − 9 ) + 17 )
b Solving
= ( )
T 5 sin 15 ( t − 9 ) + 17 and T = 20 by GDC M1

Solutions are T = 18.54 and T = 11.46 A1A1

18.54 − 11.46 = 7.08 hours (7 hours 5 minutes ) A1

21 a = log2 343 A1

a + 3d =
log2 1331 A1

Solve simultaneously to find d M1


log2 343 + 3d =
log2 1331

=3d log2 1331 − log2 343

 1331 
3d = log2  
 343 
1  1331 
d = log2   A1
3  343 
1
 1331  3
d = log2   M1
 343 

 11 
d = log2   A1
 7 
So log2 x= a + d M1

 11   11 
= log2 343 + log
= 2   log2  343 ×  A1
 7   7 

= log2 ( 49 × 11) = log2 539 A1

So x = 539

© Oxford University Press 2019 10


Exam practice

dv
22 a − sin t − cos t
= M1A1
dt
dv
Setting =0 M1
dt
tan t = −1 A1

t = A1
4
3π 3π
vMAX cos
= − sin
4 4

2 2
=
− −
2 2

= − 2 A1

So maximum speed is 2 m s-1 A1


π
b
= s ∫ ( cos t − sin t ) dt
0
M1

π
= sin t + cos t  0 A1

=−1 − 1 M1
= −2 A1
π
c v = 0 ⇒ tan t = 1 ⇒ t = M1A1
4
π
4 π
So total distance = ∫ ( cos t − sin t ) dt
0
+ ∫ ( cos t − sin t ) dt
π
M1A1
4

π π
= sin t + cos t  04 + sin t + cos t  π A1
4

= 2 − 1 + −1 − 2 M1A1

= 2 −1+1+ 2

=2 2 A1

© Oxford University Press 2019 11

You might also like